You are on page 1of 100

Merzy’s Notes Prelim Exam

Introduction: II. The Philippine Constitution


1. General Principles
1. Maloslos Constitution
2. The American Regime and organic acts
A. Political Law - branch of the public law which deals with
3. The 1935 Constitution
the organization and operation of the governmental organs 4. The Japanese Occupation
of the state and defines the relationship of the states with 5. The 1973 Constitution
the inhabitants of its territory.
Code:
Scope/ Division of Political law B. The 1987 Constitution
1. Constitutional Law.
1. Freedom of Constitution
The study of the maintenance of the proper balance 2. Adaptation of the Constitution
between authority as represented by the three inherent powers of 3. Effectivily of the 1987 Constitution
the State and
liberty as guaranteed by the Bill of Rights
Effectivity of statues
code:
Case in point : Tañada v. Tuvera G.R No: 1985
2. Administrative Law.
That branch of public law which fixes the organization
of government, determines the competence of the administrative We hold therefore that all statutes, including those of local
authorities who application and private laws, shall be published as a condition for
execute the law, and indicates to the individual remedies for the their effectivity, which shall begin fifteen days after publication
violation of his unless a different effectivity date is fixed by the legislature.
rights.
Covered by this rule are presidential decrees and executive orders
Code: promulgated by the President in the exercise of legislative powers
whenever the same are validly delegated by the legislature or, at
3.Law on Municipal Corporations. present, directly conferred by the Constitution. administrative rules
and regulations must a also be published if their purpose is to
4. Law of Public Officers. enforce or implement existing law pursuant also to a valid
delegation.
5. Election Laws.
Interpretative regulations and those merely internal in nature, that
is, regulating only the personnel of the administrative agency and

Constitutional Law 1 based on the syllabus of Atty. Remoroza 2018-2019 1


Merzy’s Notes Prelim Exam

not the public, need not be published. Neither is publication required As a general rule, an unconstitutional act is not a law; it confers
of the so-called letters of instructions issued by administrative no rights; it imposes no duties; it affords no protection; it
superiors concerning the rules or guidelines to be followed by their creates no office; it is inoperative as if it has not been passed at
subordinates in the performance of their duties. all. This rule, however, is not absolute. Under the doctrine of
operative facts, actions previous to the declaration of
Accordingly, even the charter of a city must be published unconstitutionality are legally recognized. They are not nullified.
notwithstanding that it applies to only a portion of the national This is essential in the interest of fair play.
territory and directly affects only the inhabitants of that place. All
presidential decrees must be published, including even, say, those The doctrine of operative fact, as an exception to the general
naming a public place after a favored individual or exempting him rule, only applies as a matter of equity and fair play. It nullifies
from certain prohibitions or requirements. The circulars issued by the the effects of an unconstitutional law by recognizing that the
Monetary Board must be published if they are meant not merely to existence of a statute prior to a determination of
interpret but to "fill in the details" of the Central Bank Act which that unconstitutionality is an operative fact and may have
body is supposed to enforce. consequences which cannot always be ignored. The past cannot
always be erased by a new judicial declaration. The doctrine is
However, no publication is required of the instructions issued by, applicable when a declaration of unconstitutionality will impose
say, the Minister of Social Welfare on the case studies to be made in an undue burden on those who have relied on the invalid law.
petitions for adoption or the rules laid down by the head of a Thus, it was applied to a criminal case when a declaration of
government agency on the assignments or workload of his personnel unconstitutionality would put the accused in double jeopardy or
or the wearing of office uniforms. Parenthetically, municipal would put in limbo the acts done by a municipality in reliance
ordinances are not covered by this rule but by the Local Government upon a law creating it.
Code.
Under the circumstances, the Court finds the exception applicable in
this case and holds that notwithstanding its finding of
We agree that publication must be in full or it is no publication at all
unconstitutionality in the current composition of the JBC, all its prior
since its purpose is to inform the public of the contents of the laws.
official actions are nonetheless valid.

Presumption of Constitutionality
C. Nature of the Constitution
Code:
1. Constitution defined
Case: Chavez v. JBC,
G.R no: 202242, April 16, 2013 That body of rules and maxims in accordance with
which the powers of sovereignty are habitually exercised
[Cooley, Constitutional

Constitutional Law 1 based on the syllabus of Atty. Remoroza 2018-2019 2


Merzy’s Notes Prelim Exam

Limitations, p. 4]. ARTICLE VI

Code: THE LEGISLATIVE DEPARTMENT


Section 1. The legislative power shall be vested in the Congress of
With particular reference to the Constitution of the Philippines: the Philippines which shall consist of a Senate and a House of
That written instrument enacted by direct action of the people by Representatives, except to the extent reserved to the people by the
which the provision on initiative and referendum.
fundamental powers of the government are established, limited and
defined, and Code:
by which those powers are distributed among the several
departments for their Executive Power
safe and useful exercise for the benefit of the body politic
[Malcolm, Philippine
Constitutional Law ARTICLE VII

EXECUTIVE DEPARTMENT
2. Purpose
Section 1. The executive power shall be vested in the President of
the Philippines.
To prescribe the permanent framework of a system of
Code:
government, to assign to the several departments their respective
powers and
Judicial Power
duties, and to establish certain first principles on which the
government is founded
ARTICLE VIII
code:
JUDICIAL DEPARTMENT
Section 1. The judicial power shall be vested in one Supreme Court
DEFINITION OF TERMS:
and in such lower courts as may be established by law.

Powers of the government Classification:


Classification
vis a vis Amendment Process

Legislative Power 1.Rigid Constitution:

Constitutional Law 1 based on the syllabus of Atty. Remoroza 2018-2019 3


Merzy’s Notes Prelim Exam

A rigid Constitution is one that can be amended onlyby a formal and


usually difficult process. 4.Cumulative or evolved:

2.Flexible Constitution: while a flexible Constitution is one that a cumulative constitution is the result of political evolution, not
inaugurated at any
can be changed by ordinary legislation
specific time but changing by accretion rather than by any
vis a vis Adaption systematic method
Written:
Code:
1. Written Constitution .
A written constitution is one whose precepts areembodied in one
document or set of documents; while an Our Constitution:
Code: 4. Qualities of a good written Constitution
2. Unwritten constitution Broad.
consists of rules which have not been integrated into a single,
concrete form but Not just because it provides for the organization of the entire
are scattered in various sources, such as statutes of a fundamental government and covers all persons and things within the territory of
character,
the State but because it must be comprehensive enough to provide
judicial decisions, commentaries of publicists, customs and
traditions, and certain for every contingency.
common law principles Code:

Brief:
code:
It must confine itself to basic principles to be implemented with
3. Enacted or conventional: legislative details more adjustable to change and easier to amend.

A conventional Code:
constitution is enacted, formally struck off at a definite time and
place following a
conscious or deliberate effort taken by a constituent body or ruler;
while a
Definite.

code: To prevent ambiguity in its provisions which could result in


Constitutional Law 1 based on the syllabus of Atty. Remoroza 2018-2019 4
Merzy’s Notes Prelim Exam

confusion and divisiveness among the people Art. XVI

code: Code:

4. Essential parts of a good written Constitution

a) Constitution of Liberty. 5. Interpretation/Construction of the Constitution

The series of prescriptions setting forth the


fundamental civil and political rights of the citizens and imposing 1. Supremacy of the Constitution:
limitations on the
powers of government as a means of securing the enjoyment of
those rights, e.g.,
Art. III. CASE IN POINT: Datu Michael Abas Kida v. Senate
G.R. No. 196271, October 18, 2011
Code:

b) Constitution of Government. Several laws pertaining to the Autonomous Region in Muslim


Mindanao (ARMM) were enacted by Congress. Republic Act (RA)
The series of provisions outlining the No. 6734 is the organic act that established the ARMM and
organization of the government, enumerating its powers, laying scheduled the first regular elections for the ARMM regional
down certain rules officials. RA No. 9054 amended the ARMM Charter and reset the
relative to its administration, and defining the electorate, e.g., regular elections for the ARMM regional officials to the second
Arts. VI, VII, VIII Monday of September 2001. RA No. 9140 further reset the first
and regular elections to November 26, 2001. RA No. 9333 reset for the
IX. third time the ARMM regional elections to the 2nd Monday of August
2005 and on the same date every 3 years thereafter.
Code:
Pursuant to RA No. 9333, the next ARMM regional elections
c) Constitution of Sovereignty. should have been held on August 8, 2011. COMELEC had begun
preparations for these elections and had accepted certificates of
The provisions pointing out the mode or candidacies for the various regional offices to be elected. But
procedure in accordance with which formal changes in the on June 30, 2011, RA No. 10153 was enacted, resetting the next
fundamental law may ARMM regular elections to May 2013 to coincide with the regular
be brought about, e.g., national and local elections of the country.

Constitutional Law 1 based on the syllabus of Atty. Remoroza 2018-2019 5


Merzy’s Notes Prelim Exam

may only interpret the mandate if an interpretation is appropriate


Issue: won R.A no. 10153 is unconstitutional and called for.54

In the case of the terms of local officials, their term has been fixed
Ruling: Since elective ARMM officials are local officials, they are clearly and unequivocally, allowing no room for any implementing
covered and bound by the three-year term limit prescribed by the legislation with respect to the fixed term itself and no vagueness that
Constitution; they cannot extend their term through a holdover. As would allow an interpretation from this Court. Thus, the term of
this Court put in Osmeña v. COMELEC:52 three years for local officials should stay at three (3) years as fixed
by the Constitution and cannot be extended by holdover by
It is not competent for the legislature to extend the term of officers Congress.
by providing that they shall hold over until their successors are
elected and qualified where the constitution has in effect or by clear
implication prescribed the term and when the Constitution fixes the
day on which the official term shall begin, there is no legislative
authority to continue the office beyond that period, even though the 2. Prospective in application:
successors fail to qualify within the time.

In American Jurisprudence it has been stated as follows: XPN:

"It has been broadly stated that the legislature cannot, by an act ARTICLE IV
postponing the election to fill an office the term of which is limited
by the Constitution, extend the term of the incumbent beyond the CITIZENSHIP
period as limited by the Constitution." [Emphasis ours.]
Section 2. Natural-born citizens are those who are citizens of the
Independently of the Osmeña ruling, the primacy of the Constitution Philippines from birth without having to perform any act to acquire
as the supreme law of the land dictates that where the Constitution or perfect their Philippine citizenship. Those who elect Philippine
has itself made a determination or given its mandate, then the citizenship in accordance with paragraph (3), Section 1 hereof shall
matters so determined or mandated should be respected until the be deemed natural-born citizens.
Constitution itself is changed by amendment or repeal through the
applicable constitutional process. A necessary corollary is that none
of the three branches of government can deviate from the 3. Verba Legis:
constitutional mandate except only as the Constitution itself may
allow.53 If at all, Congress may only pass legislation filing in details to
fully operationalize the constitutional command or to implement it by
legislation if it is non-self-executing; this Court, on the other hand,

Constitutional Law 1 based on the syllabus of Atty. Remoroza 2018-2019 6


Merzy’s Notes Prelim Exam

CASE IN POINT: Chavez v. JBC . It is very clear that the Framers were not keen on adjusting the
G.R. No. 202242, April 16, 2013 provision on congressional representation in the JBC because it was
not in the exercise of its primary function – to legislate. JBC was
created to support the executive power to appoint, and Congress, as
FACTS: one whole body, was merely assigned a contributory non-legislative
function.
In 1994, instead of having only seven members, an eighth member
was added to the JBC as two representatives from Congress began Domino v. Comelec
sitting in the JBC – one from the House of Representatives and one G.R. No. 134015. July 19, 1999
from the Senate, with each having one-half (1/2) of a vote. Then,
the JBC En Banc, in separate meetings held in 2000 and 2001,
decided to allow the representatives from the Senate and the House Facts:
of Representatives one full vote each. Senator Francis Joseph G.
Escudero and Congressman Niel C. Tupas, Jr. (respondents) Petitioner Domino filed his certificate of candidacy for the position of
simultaneously sit in the JBC as representatives of the legislature. It Representative of the lone legislative district of the Province of
is this practice that petitioner has questioned in this petition. Sarangani indicating that he has resided in the constituency where
he seeks to be elected for 1 year and 2 months. Private respondents
Ruling: filed a petition seeking to cancel the certificate of candidacy of
Domino, alleging that Domino, contrary to his declaration in the
The language used in the Constitution must be taken to have been certificate of candidacy, is not a resident, much less a registered
deliberately chosen for a definite purpose. Every word employed in voter, of the province of Sarangani where he seeks election.
the Constitution must be interpreted to exude its deliberate intent Thereafter, the COMELEC promulgated a resolution declaring
which must be maintained inviolate against disobedience and Domino disqualified as candidate for the position of representative of
defiance. What the Constitution clearly says, according to its text, the lone district of Sarangani in the May 11, 1998 polls for lack of
compels acceptance and bars modification even by the branch the one-year residency requirement and likewise ordered the
tasked to interpret it. cancellation of his certificate of candidacy based on his own Voter’s
Registration Record and his address indicated as 24 Bonifacio St.,
For this reason, the Court cannot accede to the argument of plain Ayala Hts., Old Balara, Quezon City.
oversight in order to justify constitutional construction. As stated in
the July 17, 2012 Decision, in opting to use the singular letter "a" to
describe "representative of Congress," the Filipino people through Issue: Whether or not petitioner has resided in Sarangani Province
the Framers intended that Congress be entitled to only one (1) seat for at least 1 year immediately preceding the May 11, 1998 elections
in the JBC. Had the intention been otherwise, the Constitution could
have, in no uncertain terms, so provided, as can be read in its other
provisions. Held: The term “residence,” as used in the law prescribing the
qualifications for suffrage and for elective office, means the same

Constitutional Law 1 based on the syllabus of Atty. Remoroza 2018-2019 7


Merzy’s Notes Prelim Exam

thing as “domicile,” which imports not only an intention to reside in a al., most of which petitions contend that the filing of the second
fixed place but also personal presence in that place, coupled with impeachment complaint is unconstitutional as it violates the
conduct indicative of such intention. “Domicile” denotes a fixed provision of Section 5 of Article XI of the Constitution that “[n]o
permanent residence to which, whenever absent for business, impeachment proceedings shall be initiated against the same official
pleasure, or some other reasons, one intends to return. more than once within a period of one year.”

A person’s domicile, once established, is considered to continue and Issues:


will not be deemed lost until a new one is established. To
successfully effect a change of domicile, one must demonstrate an 1. Whether or not the offenses alleged in the Second
actual removal or an actual change of domicile; a bona fide intention impeachment complaint constitute valid impeachable
of abandoning the former place of residence and establishing a new offenses under the Constitution.
one and definite acts which correspond with the purpose.
Held.
4. Ratio legis et anima: Interpretation according to spirit. The
the Supreme Court made reference to the use of well- settled
words of the Constitution should be interpreted in accordance with
principles of constitutional construction, namely: First, verba leais.
the intent of the framers. (Relate to number 2)
i. e., whenever possible, the words used in the Constitution must be
given their ordinary meaning except where technical terms are
employed. As the Constitution is not primarily a lawyer’s document,
Francisco v. House of Representatives
it being essential for the rule of law to obtain that it should ever be
G.R. No. 160261, November 10, 2003
present in the people’s consciousness, its language as much
as possible should be understood in the sense they have a common
use. Second, where there is ambiguity, ratio leqis et anima. The
Facts: words of the Constitution should
be interpreted in accordance with the intent of the framers.
former President Joseph E. Estrada filed an impeachment complaint
(first impeachment complaint) against Chief Justice Hilario G. Davide Civil Liberties Union v. Executive Secretary
Jr. and seven Associate Justices of the Supreme Court for “culpable 194 SCRA 317
violation of the Constitution, betrayal of the public trust and other
high crimes.” The complaint was endorsed by House FACTS: Consolidated petitions are being resolved jointly as both
Representatives, and was referred to the House Committee on seek for the declaration of the unconstitutionality of Executive Order
Justice on 5 August 2003 in accordance with Section 3(2) of Article No. 284 (EO No. 284) issued by former President Corazon C. Aquino
XI of the Constitution on July 25, 1987.
EO No. 284 allows members of the Cabinet, their Undersecretaries
Various petitions for certiorari, prohibition, and mandamus were filed and Assistant Secretaries to hold other than their government
with the Supreme Court against the House of Representatives, et. positions in addition to their primary positions

Constitutional Law 1 based on the syllabus of Atty. Remoroza 2018-2019 8


Merzy’s Notes Prelim Exam

likewise added that “considering that most of her government


ISSUE: Whether or not EO No. 284 is constitutional. records in the academe are more than 15 years old, it is reasonable
to consider it infeasible to retrieve all of those files,” and that the
HELD: The Court ruled in the negative. clearance issued by UP HRDO and CSC should be taken in her favor.
There was no record that the letter was deliberated upon. Despite
It has been held that in construing a Constitution should bear in this, on a report to the JBC, Sereno was said to have “complete
mind the object sought to be accomplished by its adoption, and the requirements.” On August 2012, Sereno was appointed Chief Justice.
evils, if any, sought to be prevented or remedied. A doubtful
provision will be examined in the light of the history of the times and On February 2018, Atty. Eligio Mallari wrote to the OSG, requesting
the condition and circumstances under which the Constitution was that the latter, in representation of the Republic, initiate a quo
framed. warranto proceeding against Sereno. The OSG, invoking the Court’s
original jurisdiction under Section 5(1), Article VIII of the
The legislative intent of both Constitutional provisions is to prevent Constitution in relation to the special civil action under Rule 66, the
government officials from holding multiple positions in the Republic, through the OSG filed the petition for the issuance of the
government for self enrichment which is a betrayal of public trust. extraordinary writ of quo warranto to declare as void Sereno’s
appointment as CJ of the SC and to oust and altogether exclude
Sereno therefrom.

Republic of the Philippines Vs. Maria Lourdes P. A. Sereno Issue: Whether the Court can assume jurisdiction and give due
G.R. No. 237428. May 11, 2018 course to the instant petition for quo warranto.

MAIN CASE: Held: . Quo warranto proceedings are essentially judicial in


character – it calls for the exercise of the Supreme Court’s
constitutional duty and power to decide cases and settle actual
On August 2010, Sereno was appointed as Associate Justice. On controversies. This constitutional duty cannot be abdicated or
2012, the position of Chief Justice was declared vacant, and the JBC transferred in favor of, or in deference to, any other branch of the
directed the applicants to submit documents, among which are “all government including the Congress, even as it acts as an
previous SALNs up to December 31, 2011” for those in the impeachment court through the Senate.
government and “SALN as of December 31, 2011” for those from the
private sector. The JBC announcement further provided that
To differentiate from impeachment, quo warranto involves a judicial
“applicants with incomplete or out-of-date documentary
determination of the eligibility or validity of the election or
requirements will not be interviewed or considered for nomination.”
appointment of a public official based on predetermined rules while
Sereno expressed in a letter to JBC that since she resigned from UP
impeachment is a political process to vindicate the violation of the
Law on 2006 and became a private practitioner, she was treated as
public’s trust. In quo warranto proceedings referring to offices filled
coming from the private sector and only submitted three (3) SALNs
by appointment, what is determined is the legality of the
or her SALNs from the time she became an Associate Justice. Sereno
appointment. The title to a public office may not be contested

Constitutional Law 1 based on the syllabus of Atty. Remoroza 2018-2019 9


Merzy’s Notes Prelim Exam

collaterally but only directly, by quo warranto proceedings. the Court’s finding that Sereno is ineligible, in the first place, to be a
usurpation of a public office is treated as a public wrong and carries candidate for the position of Chief Justice and to be nominated for
with it public interest, and as such, it shall be commenced by a said position follows as a matter of course. The Court has ample
verified petition brought in the name of the Republic of the jurisdiction to do so without the necessity of impleading the JBC as
Philippines through the Solicitor General or a public prosecutor. The the Court can take judicial notice of the explanations from the JBC
SolGen is given permissible latitude within his legal authority in members and the OEO. he Court, in a quo warranto proceeding,
actions for quo warranto, circumscribed only by the national interest maintains the power to issue such further judgment determining the
and the government policy on the matter at hand. respective rights in and to the public office, position or franchise of
all the parties to the action as justice requires.
Impeachment is not an exclusive remedy by which an invalidly
appointed or invalidly elected impeachable official may be removed Sereno is a de facto officer removable through quo warranto
from office.
The effect of a finding that a person appointed to an office is
The language of Section 2, Article XI of the Constitution does not ineligible therefor is that his presumably valid appointment will give
foreclose a quo warranto action against impeachable officers: him color of title that confers on him the status of a de facto officer.
“Section 2. The President, the Vice-President, the Members of the For lack of a Constitutional qualification, Sereno is ineligible to hold
Supreme Court, the Members of the Constitutional Commissions, and the position of Chief Justice and is merely holding a colorable right
the Ombudsman may be removed from office on impeachment or title thereto. As such, Sereno has never attained the status of an
for, and conviction of, culpable violation of the Constitution, treason, impeachable official and her removal from the office, other than by
bribery, graft and corruption, other high crimes, or betrayal of public impeachment, is justified. The remedy, therefore, of a quo warranto
trust.” The provision uses the permissive term “may” which denote at the instance of the State is proper to oust Sereno from the
discretion and cannot be construed as having a mandatory effect, appointive position of Chief Justice.
indicative of a mere possibility, an opportunity, or an option. In
American jurisprudence, it has been held that “the express provision
for removal by impeachment ought not to be taken as a tacit 5. The constitution has to be interpreted as a whole.
prohibition of removal by other methods when there are other
adequate reasons to account for this express provision.

CASES IN POINT: Civil Liberties Union v. Executive


Sereno’s ineligibility for lack of proven integrity cannot be cured by Secretary
her nomination and subsequent appointment as Chief Justice. 194 SCRA 317

Well-settled is the rule that qualifications for public office must be The Constitution, the fundamental law of the land, shall reign
possessed at the time of appointment and assumption of office and supreme over any other statute. When there is conflict, it shall be
also during the officer’s entire tenure as a continuing requirement. resolved in favor of the highest law of the land. Thus, the Court held
The voidance of the JBC nomination as a necessary consequence of that EO No. 284 is UNCONSTITUTIONAL.

Constitutional Law 1 based on the syllabus of Atty. Remoroza 2018-2019 10


Merzy’s Notes Prelim Exam

removal through impeachment and conviction was mandatory, as


opposed to “may”which
Again in Civil Liberties Union, supra., it was held should mean that it was an option to remove. Nor was “may ALSO
that while it is permissible to consult the debates and proceedings of be removed from office...” used, as it would clearly state the intent
the for processes other than impeachment and conviction to remove a
constitutional convention in order to arrive at the reason and sitting Chief Justice.
purpose of the resulting o
Constitution, resort thereto may be had only when other guides fail The phrase “may ONLYbe removed from office” was not also used.
as said But this should be interpreted within the context of the provision.
proceedings are powerless to vary the terms of the Constitution Specifically, the following must be taken into consideration:
when the meaning
is clear. We think it safer to construe the Constitution from what (i) that the Constitution reserves the process of removal through
“appears upon its impeachment
face”. The proper interpretation, therefore, depends more on how it and conviction to heads of Constitutional organs;
was understood
by the people adopting it than in the framers’ understanding thereof. (ii) that the process of removal is
deliberately cumbersome, such as the one year time bar to avoid
Republic of the Philippines Vs. Maria Lourdes P. A. Sereno G.R. No. harassment suits against
237428. May 11, 2018 impeachable officers and disruption of public service;

(iii) that the grounds for impeachment


Dissenting Opinion of Justice Leonen: are weighty and serious to shield impeachable officers
from malicious or bothersome suits.
The Constitution should be read as a singular, whole unit.
A verba legis or plain reading of Article Constitutional heads are expected to make difficult decisions. In this
XI, Section 2 of the Constitution is not proper. The words therein, light, the Constitution
“may be removed,”should be in accordance with the Constitutional should be read as to provide them incentive to do their duties. Thus,
framework and the subsequent jurisprudence over the text under “may be removed”
consideration. should be read in light of the principle that impeachment and
To focus on the word “may” precludes the importance of the entire conviction should be read as the only process to remove them from
document and provides a myopic and unhistorical view of the their respective office.
framework on which legal order rests.
o
The framers of the Constitution did not use “SHALL be removed” as
it communicates that

Constitutional Law 1 based on the syllabus of Atty. Remoroza 2018-2019 11


Merzy’s Notes Prelim Exam

6. DYNAMIC [Doctrine of Relative Constitutionality]: No doubt, the WTO Agreement was not yet in existence when the
Constitution was drafted and ratified in 1987. That does not mean
CASE IN POINT: Tañada vs Angara however that the Charter is necessarily flawed in the sense that its
G.R. No. 118295. May 2, 1997 framers might not have anticipated the advent of a borderless world
of business. By the same token, the United Nations was not yet in
existence when the 1935 Constitution became effective. Did that
Facts: necessarily mean that the then Constitution might not have
contemplated a diminution of the absoluteness of sovereignty when
This is a case petition by Sen. Wigberto Tanada, together with other the Philippines signed the UN Charter, thereby effectively
lawmakers, taxpayers, and various NGO’s to nullify the Philippine surrendering part of its control over its foreign relations to the
ratification of the World Trade Organization (WTO) Agreement. decisions of various UN organs like the Security Council?

Petitioners believe that this will be detrimental to the growth of our It is not difficult to answer this question. Constitutions are designed
National Economy and against to the “Filipino First” policy. The WTO to meet not only the vagaries of contemporary events. They should
opens access to foreign markets, especially its major trading be interpreted to cover even future and unknown circumstances. It
partners, through the reduction of tariffs on its exports, particularly is to the credit of its drafters that a Constitution can withstand the
agricultural and industrial products. Thus, provides new assaults of bigots and infidels but at the same time bend with the
opportunities for the service sector cost and uncertainty associated refreshing winds of change necessitated by unfolding events. As one
with exporting and more investment in the country. These are the eminent political law writer and respected jurist 38 explains:
predicted benefits as reflected in the agreement and as viewed by
the signatory Senators, a “free market” espoused by WTO. The Constitution must be quintessential rather than
superficial, the root and not the blossom, the base
Petitioners also contends that it is in conflict with the provisions of and frame-work only of the edifice that is yet to rise.
our constitution, since the said Agreement is an assault on the It is but the core of the dream that must take shape,
sovereign powers of the Philippines because it meant that Congress not in a twinkling by mandate of our delegates, but
could not pass legislation that would be good for national interest slowly "in the crucible of Filipino minds and hearts,"
and general welfare if such legislation would not conform to the where it will in time develop its sinews and gradually
WTO Agreement. gather its strength and finally achieve its substance.
In fine, the Constitution cannot, like the goddess
Athena, rise full-grown from the brow of the
Issue: won the provision WTO agreement is unconstitutional
Constitutional Convention, nor can it conjure by
mere fiat an instant Utopia. It must grow with the
Held:
society it seeks to re-structure and march apace
with the progress of the race, drawing from the
Constitution Designed to Meet vicissitudes of history the dynamism and vitality that
Future Events and Contingencies will keep it, far from becoming a petrified rule, a

Constitutional Law 1 based on the syllabus of Atty. Remoroza 2018-2019 12


Merzy’s Notes Prelim Exam

pulsing, living law attuned to the heartbeat of the A provision which lays down a general
nation. principle is usually not self-executing. But a provision which is
complete in itself and
Notwithstanding objections against possible limitations on national becomes operative without the aid of supplementary or enabling
sovereignty, the WTO remains as the only viable structure for legislation, or that
multilateral trading and the veritable forum for the development of which supplies a sufficient rule by means of which the right it grants
international trade law. The alternative to WTO is isolation, may be enjoyed
stagnation, if not economic self-destruction. Duly enriched with or protected, is self-executing
original membership, keenly aware of the advantages and
disadvantages of globalization with its on-line experience, and CASES IN POINT: Manila Prince Hotel v. GSIS
endowed with a vision of the future, the Philippines now straddles G.R. No. 122156, February 03, 1997
the crossroads of an international strategy for economic prosperity
and stability in the new millennium. Let the people, through their THE FACTS
duly authorized elected officers, make their free choice.
Pursuant to the privatization program of the Philippine
WHEREFORE, the petition is DISMISSED for lack of merit. Government, the GSIS sold in public auction its stake in Manila Hotel
Corporation (MHC). Only 2 bidders participated: petitioner Manila
Prince Hotel Corporation, a Filipino corporation, which offered to buy
51% of the MHC or 15,300,000 shares at P41.58 per share, and
7. GENERAL RULE: Provisions of the Constitution are deemed Renong Berhad, a Malaysian firm, with ITT-Sheraton as its hotel
self-executing operator, which bid for the same number of shares at P44.00 per
share, or P2.42 more than the bid of petitioner.

Self-executing/non-self-executing provisions Petitioner filed a petition before the Supreme Court to


compel the GSIS to allow it to match the bid of Renong Berhad. It
EXCEPTION: invoked the Filipino First Policy enshrined in §10, paragraph 2, Article
XII of the 1987 Constitution, which provides that “in the grant of
Article 2 (DECLARATION OF PRINCIPLES AND STATE
rights, privileges, and concessions covering the national economy
POLICIES PRINCIPLES).
and patrimony, the State shall give preference to qualified Filipinos.”
In case of doubt, the provisions should be considered selfexecuting;
mandatory rather than directory; and prospective rather than
Issue:
retroactive
Self-executing provisions.

Constitutional Law 1 based on the syllabus of Atty. Remoroza 2018-2019 13


Merzy’s Notes Prelim Exam

1. Whether §10, paragraph 2, Article XII of the 1987 Pamatong v. Comelec


Constitution is a self-executing provision and does not need G.R. No. 161872, April 13, 2004
implementing legislation to carry it into effect
Held: FACTS:
YES, paragraph 2, Article XII of the 1987
Constitution is a self-executing provision and does not need Petitioner Pamatong filed his Certificate of Candidacy (COC) for
implementing legislation to carry it into effect. President. Respondent COMELEC declared petitioner and 35 others
as nuisance candidates who could not wage a nationwide campaign
and/or are not nominated by a political party or are not supported by
a registered political party with a national constituency.
Sec. 10, second par., of Art XII is couched in such a way as
not to make it appear that it is non-self-executing but simply for ISSUE:
purposes of style. But, certainly, the legislature is not precluded
from enacting further laws to enforce the constitutional provision so Is there a constitutional right to run for or hold public office?
long as the contemplated statute squares with the
Constitution. Minor details may be left to the legislature without RULING:
impairing the self-executing nature of constitutional provisions.
the second paragraph can only be self-executing as it does No. What is recognized in Section 26, Article II of the Constitution is
merely a privilege subject to limitations imposed by law. It neither
not by its language require any legislation in order to give preference
bestows such a right nor elevates the privilege to the level of an
to qualified Filipinos in the grant of rights, privileges and concessions enforceable right. There is nothing in the plain language of the
covering the national economy and patrimony. A constitutional provision which suggests such a thrust or justifies an interpretation
provision may be self-executing in one part and non-self-executing in of the sort.
another.
Section 26, Article II of the Constitution neither bestows a right nor
Thus, a constitutional provision is self-executing if the nature and elevates the privilege to the level of an enforceable right. Like the
extent of the right conferred and the liability imposed are fixed by rest of the policies enumerated in Article II, the provision does not
the Constitution contain any judicially enforceable constitutional right but merely
itself, so that they can be determined by an examination and specifies a guideline for legislative or executive action. The disregard
construction of its of this provision does not give rise to any cause of action before the
terms, and there is no language indicating that the subject is courts
referred to the legislature for action.
What is the importance of knowing whether a provision in
the constitution is self-executing and not self-executing?

Constitutional Law 1 based on the syllabus of Atty. Remoroza 2018-2019 14


Merzy’s Notes Prelim Exam

Tondo Medical v. Court of Appeals aids or as guides in the exercise of its power of judicial review, and
527 SCRA 746 2007 by the legislature in its enactment of laws.

Facts: B. Amendment Amendment vs Revision

The Health Sector Reform Agenda (HSRA) was launched by the Lambino v. Comelec
Department of Health (DOH) in 1999, which provided five areas of G.R. No. 174153, October 25, 2006
general reform. One in particular was the provision of
fiscal autonomy to government hospitals that implements the Facts: Petitioners (Lambino group) commenced gathering
collection of socialized user fees and the corporate restructuring of signatures for an initiative petition to change the 1987 constitution,
government hospitals. The petitioners alleged that the they filed a petition with the COMELEC to hold a plebiscite that will
implementation of the aforementioned reform had resulted in ratify their initiative petition under RA 6735. Lambino group alleged
making free medicine and free medical services inaccessible to that the petition had the support of 6M individuals fulfilling what was
economically disadvantage Filipinos. Thus, they alleged that the provided by art 17 of the constitution. Their petition changes the
HSRA is void for violating the following constitutional provisions: 1987 constitution by modifying sections 1-7 of Art 6 and sections 1-4
Sections 5, 9, 10, 11, 13, 15, 18 of Article II, Section 1 of Article III, of Art 7 and by adding Art 18. the proposed changes will shift the
Sections 11 and 14 of Article XIII, and Sections 1 and 3(2) of Article present bicameral- presidential form of government to unicameral-
XV. On May 24, 1999, then President Joseph Ejercito Estrada issued parliamentary. COMELEC denied the petition due to lack of enabling
Executive Order No. 102, entitled “Redirecting the functions and law governing initiative petitions and invoked the Santiago Vs.
Operations of the Department of Health”, which provided for Comelec ruling that RA 6735 is inadequate to implement the
the changes in the roles, functions, and organizational processes of initiative petitions.
the DOH.
Issue:
Issues: . WON the HSRA is void for violating various provisions of
the Constitution Whether or Not the Lambino Group’s initiative petition complies with
Section 2, Article XVII of the Constitution on amendments to the
Constitution through a people’s initiative
No. As a general rule, the provisions of the Constitution are
considered self-executing, and do not require future legislation for Held:
their enforcement. For if they are not treated as self-executing, the
mandate of the fundamental law can be easily nullified by the The Initiative Petition Does Not Comply with Section 2, Article XVII
inaction of By its very title, Article II of the Constitution is a of the Constitution on Direct Proposal by the People
"declaration of principles and state policies." x x x. These principles
in Article II are not intended to be self-executing principles ready for The petitioners failed to show the court that the initiative signer
enforcement through the courts. They are used by the judiciary as must be informed at the time of the signing of the nature and effect,

Constitutional Law 1 based on the syllabus of Atty. Remoroza 2018-2019 15


Merzy’s Notes Prelim Exam

failure to do so is “deceptive and misleading” which renders the Modes: Congress, Con-con, People Congress and Con-con
initiative void. Examples
1. Extending limit of term 1. Shift to federal
In determining whether the Lambino proposal involves an 2. Voting age 2. Separation of powers
amendment or a revision, the Court considered the two-part test. 3. Change of policy mass- media 3. Check and balance
First, the
quantitative test asks whether the proposed change is so extensive
in its Tests to determine if the proposed change is an amendment
provisions as to change directly the “substance entirety” of the or a revision
Constitution by the
deletion or alteration of numerous provisions. The court examines QUANTITATIVE TEST
only the number
of provisions affected and does not consider the degree of the QUALITATIVE TEST
change. Second,
the qualitative test, which inquires into the qualitative effects of the
proposed 2. Constituent vs Legislative Power
change in the Constitution. The main inquiry is whether the change
will Imbong v. Comelec
“accomplish such far-reaching changes in the nature of our basic 35 SCRA 28
governmental
plan as to amount to a revision”. Facts:
ii) The Lambino proposal constituted a revision, not simply an
amendment, of the Constitution, because it involved a change in the These two separate but related petitions for declaratory relief were
form of filed pursuant to Sec. 19 of R.A. No. 6132 by petitioners Manuel B.
government, from presidential to parliamentary, and a shift from the Imbong and Raul M. Gonzales, both members of the Bar, taxpayers
present and interested in running as candidates for delegates to the
bicameral to a a unicameral legislature. Constitutional Convention. Both impugn the constitutionality of R.A.
No. 6132, claiming during the oral argument that it prejudices their
Let’s outline what we have learned from the case: rights as such candidates.

Amendments Revisions On March 16, 1967, Congress, acting as a Constituent Assembly


pursuant to Art. XV of the Constitution, passed Resolution No. 2
Purpose: Improved or change specific Examine entirely which among others called for a Constitutional Convention to
Provision propose constitutional amendments to be composed of two
Affected: Only specific provision Several Provisions delegates from each representative district who shall have the same
Substance: reduces w/o altering affects the basic principle qualifications as those of Congressmen, to be elected on the second

Constitutional Law 1 based on the syllabus of Atty. Remoroza 2018-2019 16


Merzy’s Notes Prelim Exam

Tuesday of November, 1970 in accordance with the Revised Election convention were passed by the required three-
Code. On June 17, 1969, Congress, also acting as a Constituent fourths vote.
Assembly, passed Resolution No. 4 amending the aforesaid
Resolution No. 2 of March 16, 1967 by providing that the convention 2. The grant to Congress as a Constituent Assembly
“shall be composed of 320 delegates apportioned among the existing of such plenary authority to call a constitutional
representative districts according to the number of their respective convention includes, by virtue of the doctrine of
inhabitants: Provided, that a representative district shall be entitled necessary implication, all other powers essential to
to at least two delegates, who shall have the same qualifications as the effective exercise of the principal power granted,
those required of members of the House of Representatives,” 1 “and such as the power to fix the qualifications, number,
that any other details relating to the specific apportionment of apportionment, and compensation of the delegates
delegates, election of delegates to, and the holding of, the as well as appropriation of funds to meet the
Constitutional Convention shall be embodied in an implementing expenses for the election of delegates and for the
legislation: Provided, that it shall not be inconsistent with the operation of the Constitutional Convention itself, as
provisions of this Resolution well as all other implementing details indispensable
to a fruitful convention. Resolutions Nos. 2 and 4
Issue: already embody the above-mentioned details,
except the appropriation of funds.
1. Whether the Congress has a right to call for Constitutional
Convention;
2. Whether the parameters set by such a call is constitutional. 3.Steps in Amendatory Process:

Decision: a. Proposal (Secs. 1-3, Art. XVII)

The Congress has the authority to call for a Constitutional Section 1. Any amendment to, or revision of, this Constitution may
Convention as a Constituent Assembly. Furthermore, specific be proposed by:
provisions assailed by the petitioners are deemed as constitutional. 1. The Congress, upon a vote of three-fourths of all its Members; or
2. A constitutional convention.
1. Congress, when acting as a Constituent Assembly
Section 2. Amendments to this Constitution may likewise be directly
pursuant to Art. XV of the Constitution, has full and
proposed by the people through initiative upon a petition of at least
plenary authority to propose Constitutional
twelve per centum of the total number of registered voters, of which
amendments or to call a convention for the purpose,
every legislative district must be represented by at least three per
by a three-fourths vote of each House in joint
centum of the registered voters therein. No amendment under this
session assembled but voting separately.
section shall be authorized within five years following the ratification
Resolutions Nos. 2 and 4 calling for a constitutional
of this Constitution nor oftener than once every five years thereafter.

Constitutional Law 1 based on the syllabus of Atty. Remoroza 2018-2019 17


Merzy’s Notes Prelim Exam

The Congress shall provide for the implementation of the exercise of 3. Whether or not the proposed amendments are so extensive
this right. in character that goes far beyond the limits of the authority
Section 3. The Congress may, by a vote of two-thirds of all its conferred on the Interim Batasang Pambansa.
Members, call a constitutional convention, or by a majority vote of all
its Members, submit to the electorate the question of calling such a HELD
convention.
1. No. The proposed amendments do not go beyond the limits of
Occena v. Comelec, 104 SCRA 1, 1981
the authority conferred on the Interim Batasang Pambansa.
FACTS: In Del Rosario v. Commission on Elections, Justice Makasiar,
dispose this contention: thus, "And whether the Constitutional
Petitioners Samuel Occena and Ramon A. Gonzales, both members Convention will only propose amendments to the Constitution or
of the Philippine Bar and former delegates to the 1971 Constitutional entirely overhaul the present Constitution and propose an entirely
Convention that framed the present Constitution, are suing as new Constitution based on an Ideology foreign to the democratic
taxpayers assailed against the validity of three Batasang Pambansa system, is of no moment; because the same will be submitted to
Resolutions (Resolution No. 1 proposing an amendment allowing a the people for ratification. Once ratified by the sovereign people,
natural-born citizen of the Philippines naturalized in a foreign country there can be no debate about the validity of the new
to own a limited area of land for residential purposes was approved Constitution. The fact that the present Constitution may be
by the vote of 122 to 5; Resolution No. 2 dealing with the revised and replaced with a new one ... is no argument against
Presidency, the Prime Minister and the Cabinet, and the National the validity of the law because 'amendment' includes the 'revision'
Assembly by a vote of 147 to 5 with 1 abstention; and Resolution or total overhaul of the entire Constitution. At any rate, whether
No. 3 on the amendment to the Article on the Commission on the Constitution is merely amended in part or revised or totally
Elections by a vote of 148 to 2 with 1 abstention) 1proposing changed would become immaterial the moment the same is
constitutional amendments. The petitioners asserted in rather ratified by the sovereign people." Therefore,
unorthodox aspect that the 1973 Constitution is not the fundamental the Interim Batasang Pambansa, sitting as a constituent body,
law, the Javellana 2 ruling to the contrary notwithstanding. can propose amendments, whether extensive in character, is
inconsequential once ratified.
ISSUES:

1. Whether or not the 1973 Constitution is the fundamental WHEREFORE, it is the ruling of the court that the petitions be
law. dismissed for lack of merit.
2. Whether or not the resolutions issued by the Interim
Batasang Pambansa is unconstitional or not unconstitutional.

Constitutional Law 1 based on the syllabus of Atty. Remoroza 2018-2019 18


Merzy’s Notes Prelim Exam

Lambino, supra 2. Statute


3. Local Legislation

i. Congress Section 2 (b), RA 6735 provides for:

Indirect Initiative :
which refers to a petition proposing
ii. Constitutional Convention amendments to the Constitution; initiative on statutes which refers
Three Theories on the position of a Constitutional to a petition
Convention vis-à-vis the regular the departments of proposing to enact a national legislation; and initiative on local
government: legislation which refers to a petition proposing to enact a regional,
provincial, city, municipal or bararigay law, resolution or ordinance
1. Theory of Conventional Sovereignty [Sec. 2(a), R.A. 6735
2. Convention is inferior to the other department
3. Independent of and co-equal to the other departments Direct Initiative

Indirect Initiative is
exercise of initiative by the people through a proposition sent to
iii. People only for Amendments Congress or the local legislative body for action

SANTIAGO VS. COMELEC 270 SCRA 106 (1997)


Republic Act No. 6735 - An Act Providing for a System of Initiative
and Referendum
FACTS:
Limitation: No amendment in this manner shall be authorized
within five years following the ratification of this Constitution nor Private respondent filed with public respondent Commission on
more often than once every five years thereafter. Elections (COMELEC) a “Petition to Amend the Constitution, to Lift
Term Limits of Elective Officials, by People’s Initiative” (Delfin
Initiative: People, through the power of initiative [Sec. 2, Art. Petition) wherein Delfin asked the COMELEC for an order (1) Fixing
XVI/]. the time and dates for signature gathering all over the country;
Requisite: A petition of at least 12% of the total number of (2) Causing the necessary publications of said Order and the
registered voters, of which every legislative district must be attached “Petition for Initiative on the 1987 Constitution, in
represented by at least 3% of the registered voters therein. newspapers of general and local circulation; and (3) Instructing
Municipal Election Registrars in all Regions of the Philippines, to
Three kinds of initiative: assist Petitioners and volunteers, in establishing signing stations at
1. Constitution the time and on the dates designated for the purpose. Delfin

Constitutional Law 1 based on the syllabus of Atty. Remoroza 2018-2019 19


Merzy’s Notes Prelim Exam

asserted that R.A. No. 6735 governs the conduct of initiative to only participation of the COMELEC or its personnel before the filing
amend the Constitution and COMELEC Resolution No. 2300 is a valid of such petition are (1) to prescribe the form of the petition; (2) to
exercise of delegated powers. Petitioners contend that R.A. No. 6375 issue through its Election Records and Statistics Office a certificate
failed to be an enabling law because of its deficiency and on the total number of registered voters in each legislative
inadequacy, and COMELEC Resolution No. 2300 is void. district; (3) to assist, through its election registrars, in the
establishment of signature stations; and (4) to verify, through its
ISSUE: election registrars, the signatures on the basis of the registry list of
voters, voters’ affidavits, and voters’ identification cards used in the
Whether or not (1) the absence of subtitle for such initiative is not immediately preceding election
fatal, (2) R.A. No. 6735 is adequate to cover the system of initiative
on amendment to the Constitution, and (3) COMELEC Resolution No.
2300 is valid. .
Procedure.
HELD:
The essence of amendments directly proposed
by the people through initiative upon a petition is that the entire
NO. Petition (for prohibition) was granted. The conspicuous silence
proposal on its face is a petition of the people. Thus, two essential
in subtitles simply means that the main thrust of the Act is initiative
elements must be present:
and referendum on national and local laws. R.A. No. 6735 failed to
provide sufficient standard for subordinate legislation. Provisions
COMELEC Resolution No. 2300 prescribing rules and regulations on
For a valid People’s initiative, there must be:
the conduct of initiative or amendments to the Constitution are
declared void.
Thus, two essential elements must be present:
RATIO: (1) The
people must author and sign the entire proposal; no agent or
Subtitles are intrinsic aids for construction and interpretation. R.A. representative can sign in their behalf.
No. 6735 failed to provide any subtitle on initiative on the
Constitution, unlike in the other modes of initiative, which are (2) As an initiative upon a petition, the proposal must be embodied
specifically provided for in Subtitle II and Subtitle III. This deliberate in the petition. The rationale for these requisites is that the signature
omission indicates that the matter of people’s initiative to amend the requirement would be meaningless if the person supplying the
Constitution was left to some future law. signature has not first seen what it is that he is signing, and more
importantly, a loose interpretation
The COMELEC acquires jurisdiction over a petition for initiative only of the subscription requirement would pose a significant potential for
after its filing. The petition then is the initiatory pleading. Nothing fraud. In Lambino, the great majority of the 6.3 million people who
before its filing is cognizable by the COMELEC, sitting en banc. The signed the signature sheets did not see the full text of the proposed

Constitutional Law 1 based on the syllabus of Atty. Remoroza 2018-2019 20


Merzy’s Notes Prelim Exam

certification by the Commission on Elections of the sufficiency of the


changes before signing; they were not apprised of the nature and petition.
effect of the proposed amendments. Failure to comply with these
requirements was fatal to the validity of the initiative petition How is ratification done?
[Lambino v. Comelec, supra.].

Gonzales v. Comelec, 21 SCRA 774

NOTE: People’s initiative applies only to an amendment, not a


revision, of the Constitution. Apeople’s inititiative can only propose In June 1967, Republic Act 4913 was passed. This law provided for
amendments the COMELEC to hold a plebiscite for the proposed amendments to
to the Constitution, inasmuch as the Constitution itself limits the Constitution. It was provided in the said law that the plebiscite
initiatives to shall be held on the same day that the general national elections
amendments, as shown by the deliberations of the Constitutional shall be held (November 14, 1967). This was questioned by Ramon
Commission. The Gonzales and other concerned groups as they argued that this was
Lambino initiative constituted a revision because it proposed to unlawful as there would be no proper submission of the proposals to
change the form the people who would be more interested in the issues involved in
of government from presidential to parliamentary and the bicameral the general election rather than in the issues involving the plebiscite.
to a
unicameral legislature. Thus, the people’s initiative as a mode to Gonzales also questioned the validity of the procedure adopted by
effect these Congress when they came up with their proposals to amend the
proposed amendments was invalid [Lambino v. Comelec, supra.]. Constitution (RA 4913). In this regard, the COMELEC and other
respondents interposed the defense that said act of Congress cannot
b. Ratification (Sec. 4, Art. XVII) be reviewed by the courts because it is a political question.

Issue: Whether or not a plebiscite may be held simultaneously with


Section 4.Any amendment to, or revision of, this Constitution under
a general election
Section 1 hereof shall be valid when ratified by a majority of the
votes cast in a plebiscite which shall be held not earlier than sixty
II. Yes. There is no prohibition to the effect that a plebiscite must
days nor later than ninety days after the approval of such
only be held on a special election. SC held that there is nothing in
amendment or revision.
this provision of the [1935] Constitution to indicate that the election
therein referred to is a special, not a general election. The
Any amendment under Section 2 hereof shall be valid when ratified
circumstance that the previous amendment to the Constitution had
by a majority of the votes cast in a plebiscite which shall be held not
been submitted to the people for ratification in special elections
earlier than sixty days nor later than ninety days after the
merely shows that Congress deemed it best to do so under the

Constitutional Law 1 based on the syllabus of Atty. Remoroza 2018-2019 21


Merzy’s Notes Prelim Exam

circumstances then obtaining. It does not negate its authority to as the resolution of the respondent COMELEC complying therewith
submit proposed amendments for ratification in general elections are declared null and void.

Tolentino v. Comelec, 41 SCRA 702 The condition and limitation that all the amendments to be proposed
by the same Convention must be submitted to the people in a single
Facts:
plebiscite pursuant to Section 1 Article XV of the Constitution. The
The Constitutional Convention of 1971 approved on 28 September part that the people play in its amendment becomes harder, when a
1971 Organic Resolution No 1, amending Section 1 Article V of the whole constitution is submitted to them, more or less they can
Constitution so as to lower the voting age to 18 and that the assumed its harmony as an integrated whole, and they can either
plebiscite for partial amendment to take place with the local accept or reject it in its entirety. When an amendment is submitted
elections on November 1971. President Diosdado Macapagal called to them that is to form part of the existing constitution, in like
upon the COMELEC to help the Convention implement the said fashion they can study with deliberation the proposed amendment in
resolution. On 30 September 1971 COMELEC resolved to inform the relation to the whole existing constitution and or any of its parts.
Convention that it will hold the plebiscite. Succeeding resolutions on
The use of the word “election" in the singular meant that
campaigning and confirming the authority of the President of the the entire Constitution must be submitted for ratification at one
Convention to implement the Organic Resolution were approved. plebiscite only;
Petitioner, Arturo Tolentino contended that under Section 1 Article furthermore, the people have to be given a “proper frame of
XV of the Constitution, the proposed amendment in question cannot reference” in arriving
be presented to the people for ratification separately from each and at their decision. Thus, submission for ratification of piece-meal
amendments by
all of the other amendments to be drafted and proposed by the
the Constitutional Convention (which is tasked to revise the
Convention. Constitution) was
disallowed since the people had, at that time, no idea yet of what
Issue: the rest of the
revised Constitution would be
Whether or not the Convention may call for a plebiscite on the sole
amendment contained in Organic Resolution 1 pursuant to Section 1 Do the Courts have the power to decide on the validity or
Article XV of the Constitution. invalidity of a proposed amendment?

Held: Petition is granted. Organic Resolution No. 1 and the


implementing acts and resolutions of the Convention, insofar as they
provide for the holding of a plebiscite on 08 November 1971, as well

Constitutional Law 1 based on the syllabus of Atty. Remoroza 2018-2019 22


Merzy’s Notes Prelim Exam

Sanidad v. Comelec, 78 SCRA 333 regular National Assembly in its active session, the power to propose
amendments becomes ipso facto the prerogative of the regular
Facts: National Assembly (Sec. 1, pars. 1 and 2 of Art. XVI, 1973
Constitution).
On 2 Sept 1976, Marcos issued PD No. 991 calling for a national
referendum on 16 Oct 1976 for the Citizens Assemblies Javellana v. Executive Secretary, 50 SCRA 50
(“barangays”) to resolve, among other things, the issues of martial
law, the interim assembly, its replacement, the powers of such Facts:
replacement, the period of its existence, the length of the period for
the exercise by the President of his present powers. Twenty days In 1973, Marcos ordered the immediate implementation of the new
after, the President issued another related decree, PD No. 1031, 1973 Constitution. Javellana, a Filipino and a registered voter sought
amending the previous PD No. 991, by declaring the provisions of PD to enjoin the Exec Sec and other cabinet secretaries from
No. 229 providing for the manner of voting and canvass of votes in implementing the said constitution. Javellana averred that the said
“barangays” applicable to the national referendum-plebiscite of Oct constitution is void because the same was initiated by the president.
16, 1976. Quite relevantly, PD No. 1031 repealed inter alia, Sec 4, of He argued that the President is w/o power to proclaim the
PD No. 991. On the same date of 22 Sept 1976, Marcos issued PD ratification by the Filipino people of the proposed constitution.
No. 1033, stating the questions to he submitted to the people in the Further, the election held to ratify such constitution is not a free
referendum-plebiscite on October 16, 1976. The Decree recites in its election there being intimidation and fraud.
“whereas” clauses that the people’s continued opposition to the
convening of the interim National Assembly evinces their desire to ISSUE: Whether or not the SC must give due course to the petition.
have such body abolished and replaced thru a constitutional
amendment, providing for a new interim legislative body, which will HELD: The SC ruled that they cannot rule upon the case at bar.
be submitted directly to the people in the referendum-plebiscite of Majority of the SC justices expressed the view that they were
October 16. concluded by the ascertainment made by the president of the
Philippines, in the exercise of his political prerogatives. Further, there
ISSUE: Whether or not Marcos can validly propose amendments to being no competent evidence to show such fraud and intimidation
the Constitution. during the election, it is to be assumed that the people had
acquiesced in or accepted the 1973 Constitution. The question of the
HELD: Yes. The amending process both as to proposal and validity of the 1973 Constitution is a political question which was left
ratification raises a judicial question. This is especially true in cases to the people in their sovereign capacity to answer. Their ratification
where the power of the Presidency to initiate the amending process of the same had shown such acquiescence.
by proposals of amendments, a function normally exercised by the
legislature, is seriously doubted. Under the terms of the 1973
Constitution, the power to propose amendments to the Constitution
resides in the interim National Assembly during the period of
transition (Sec. 15, Transitory Provisions). After that period, and the
Constitutional Law 1 based on the syllabus of Atty. Remoroza 2018-2019 23
Merzy’s Notes Prelim Exam

Issue: Whether the validity of Proclamation 1102 is a political or a


justiciable question

Judicial Review of Amendments. The question is now regarded


as subject
to judicial review, because invariably, the issue will boil down to
whether or not the
constitutional provisions had been followed
Javellana v. Executive Secretary, 50 SCRA 50],

Doctrine of proper submission.

Because the Constitution itself prescribes the time frame within


which the plebiscite is to be held, there can no longer be a question
on whether the time given to the people to determine the merits and
demerits of the proposed amendment is adequate.

Can you amend the constitution once every 5 years?


Can you revise the constitution more than once every 5
years?
What are the limitations on the power of the people to
propose changes in the Constitution?
1.
2.
3.

Constitutional Law 1 based on the syllabus of Atty. Remoroza 2018-2019 24


Merzy’s Notes Prelim Exam

E. The Power of Judicial Review departments, but merely an expression of the supremacy of the
Constitution
1. Judicial Review
Aquino v. Enrile, 59 SCRA 183
Angara v. Electoral Commission, 63 Phil. 139

FACTS: In the elections of Sept 17, 1935, Angara, and the FACTS: The petitioners having been arrested and held pursuant to
respondents, Pedro Ynsua et al. were candidates voted for the General Order No. 2 of the President (September 22, 1972), "for
position of member of the National Assembly for the first district of being participants or for having given aid and comfort in the
the Province of Tayabas. On Oct 7, 1935, Angara was proclaimed as conspiracy to seize political and state power in the country and to
member-elect of the NA for the said district. On November 15, 1935, take over the Government by force ...", filed the petitions for habeas
corpus. General Order No. 2 was issued by the President in the
he took his oath of office. On Dec 3, 1935, the NA in session
exercise of the powers he assumed by virtue of Proclamation No.
assembled, passed Resolution No. 8 confirming the election of the 1081 (September 21, 1972) placing the entire country under martial
members of the National Assembly against whom no protest had law.
thus far been filed. On Dec 8, 1935, Ynsua, filed before the Electoral
Commission a “Motion of Protest” against the election of Angara. ISSUE: Whether or not the validity of Proclamation No. 1081 is
subject to judicial inquiry
ISSUES: Whether or not the SC has jurisdiction over such matter.
Whether or not EC acted without or in excess of jurisdiction in taking HELD: The duty remains to assure that the supremacy of the
cognizance of the election protest. Constitution is upheld The power is inherent in the Judicial
Department, by virtue of the doctrine of separation of powers
HELD: The SC ruled in favor of Angara. The SC emphasized that in
cases of conflict between the several departments and among the Bondoc v. Pineda, 201 SCRA 792
agencies thereof, the judiciary, with the SC as the final arbiter, is the
Facts:
only constitutional mechanism devised finally to resolve the conflict
and allocate constitutional boundaries. That judicial supremacy is but
Emigdio Bondoc and Marciano Pineda were rivals for a Congressional
the power of judicial review in actual and appropriate cases and seat in the 4th District of Pampanga. Pineda was a member of the
controversies, and is the power and duty to see that no one branch Laban ng Demokratikong Pilipino (LDP). While Bondoc was a
or agency of the government transcends the Constitution, which is member of the Nacionalista Party (NP). Pineda won in that election.
the source of all authority. However, Bondoc contested the result in the HRET (House of
Representatives Electoral Tribunal). Bondoc won in the protest and
The power of the courts to test the validity of executive and he was subsequently declared as the winner by the HRET.
legislative acts in light of their conformity with the Constitution. This
is not an assertion of superiority by the courts over the other
Constitutional Law 1 based on the syllabus of Atty. Remoroza 2018-2019 25
Merzy’s Notes Prelim Exam

Pineda contends that the issue is already outside the jurisdiction of


the Supreme Court because Camasura’s removal is an official act of Ynot v. Intermediate Appellate Court, 148 SCRA
Congress and by virtue of the doctrine of separation of powers, the 659
judiciary may not interfere.

ISSUE: Whether or not the Supreme Court may inquire upon the Facts:
validity of the said act of the HRET without violating the doctrine of
separation of powers. There had been an existing law which prohibited the slaughtering of
carabaos (EO 626). To strengthen the law, Marcos issued EO 626-A
HELD: Yes. The SC can settle the controversy in the case at bar which not only banned the movement of carabaos from
without encroaching upon the function of the legislature particularly interprovinces but as well as the movement of carabeef. On 13 Jan
a part thereof, HRET. The issue here is a judicial question. It must 1984, Ynot was caught transporting 6 carabaos from Masbate to
be noted that what is being complained of is the act of HRET not the Iloilo. He was then charged in violation of EO 626-A. Ynot averred
act of Congress. In here, when Camasura was rescinded by the EO 626-A as unconstitutional for it violated his right to be heard or
tribunal, a decision has already been made, members of the tribunal his right to due process. He said that the authority provided by EO
have already voted regarding the electoral contest involving Pineda 626-A to outrightly confiscate carabaos even without being heard is
and Bondoc wherein Bondoc won. The LDP cannot withdraw their unconstitutional. The lower court ruled against Ynot ruling that the
representative from the HRET after the tribunal has already reached EO is a valid exercise of police power in order to promote general
a decision. They cannot hold the same election since the issue has welfare so as to curb down the indiscriminate slaughter of carabaos.
already become moot and academic. LDP is merely changing their
representative to change the outcome of the election. ISSUE: Whether or not the law is valid. Who may exercise the
power ?
That duty is part of the judicial power vested in the courts by an
express grant under Sec. 1, Art. VIII of the Constitution which
Held: yes. Sa EO is unconstitutional.
states: “Judicialpower includes the duty of the courts of justice to
settle actual controversiesinvolving rights which are legally
demandable and enforceable, and to determine whether or not there The challenged measure is an invalid exercise of Police power
has been a grave abuse of discretion amounting to lack or because the method employed to conserve the carabaos is not
excess of jurisdiction on the part of any branch or instrumentality of reasonably necessary to the purpose of the law and, worse, is
unduly oppressive. To justify the State in the imposition of its
Government.
authority in behalf of the public, it must be:
1) The interest of the public generally, as distinguished from those of
a particular class, require such interference;
2. Who may exercise the power?
Sec. 4(2), Art. VIII

Constitutional Law 1 based on the syllabus of Atty. Remoroza 2018-2019 26


Merzy’s Notes Prelim Exam

2) that the means employed are reasonably necessary for the Ongsuco v. Malones, 604 SCRA 499 (2009)
accomplishment of the purpose, and not unduly oppressive upon
individuals. Facts:

Petitioners are stall holders at the Maasin Public Market. After a


This Court has declared that while lower courts should observe a meeting with the stall holders, Sangguniang Bayan of Maasin
becoming modesty in examining constitutional questions, they are approvedMunicipal Ordinance No. 98-01, entitled "The Municipal
nonetheless not prevented from resolving the same whenever Revised Revenue Code."The Code contained a provision for
warranted, subject only to review by the highest tribunal. 6 We have
increased rentals for the stalls and the imposition of goodwill fees in
jurisdiction under the Constitution to "review, revise, reverse, modify
the amount of P20,000.00 andP15,000.00 for stalls located on the
or affirm on appeal or certiorari, as the law or rules of court may
provide," final judgments and orders of lower courts in, among first and second floors of the municipal public market, respectively.
others, all cases involving the constitutionality of certain measures. 7 The same Code authorizedrespondent to enter into lease contracts
This simply means that the resolution of such cases may be made in over the said market stalls, and incorporated a standard contract of
the first instance by these lower courts. lease for the stall holders at themunicipal public market.Sangguniang
Bayan of Maasin approved Resolution No. 68, series of 1998, moving
Judicial power authorizes this; and when the exercise is demanded, to have the meeting declared inoperative as a publichearing,
there should be no shirking of the task for fear of retaliation, or loss
because majority of the persons affected by the imposition of the
of favor, or popular censure, or any other similar inhibition unworthy
of the bench, especially this Court. goodwill fee failed to agree to the said measure.

Issues:
W/N there was a need for the exhaustion of administrative
remedies- NOW/N the imposition of the goodwill fees is valid- NO, it
is defective due to lack of public hearings
Commissioner of Internal Revenue v. Court of Tax
Appeals, 195 SCRA 444
Held:
it was held that the fact that the constitutional question was
properly raised by a party is not alone sufficient for the respondent Thus, a case where theissue raised is a purely legal question, well
court to pass upon the issue of constitutionality; every court should within the competence; and the jurisdiction of the court and not the
approach a constitutional question with grave care and considerable administrative agency,would clearly constitute an exception.There is
caution. no dispute herein that the notices sent to petitioners and other stall
holders at the municipal public market were sent out,informing th
them of the supposed "public hearing" to be held on 11 August
1998. Even assuming that petitioners received their notice,

Constitutional Law 1 based on the syllabus of Atty. Remoroza 2018-2019 27


Merzy’s Notes Prelim Exam

the"public hearing" was already scheduled, and actually conducted, and gambling lords, sought admission into the Government's
only five days later. "Witness Protection, Security and Benefit Program." Allegedly, he
gained first-hand information in his capacity as Chairman of the Task
3. Functions of Judicial Review Force Anti-Gambling (TFAG) during the term of former President
a. Checking Corazon C. Aquino until his resignation in 1989. He also revealed
b. Legitimating that he and members of his family were in danger of being
c. Symbolic liquidated, facing as he did the formidable world of corruption with a
Salonga v. Pano, 134 SCRA 438 well-entrenched hold on Philippine social, political and economic
systems.
4. Requisites of Judicial Review
REALS (Ripeness, Ealist opportunity, actual case or Issue: Whether or not a witness' testimony requires prior or
controversy, Lis Mota and Standing) simultaneous corroboration at the time he is admitted into
the witness protection, security and benefit program. 5
1. Actual Case or Controversy
2 Must be raise at the poper party Legal Standing/locus Held: No Actual Controversy
standi
3. Without going into the merits of the case, the Court finds the
4. petition fundamentally defective. The Constitution provides that
judicial power "includes the duty of the courts of justice to settle
actual controversies involving rights which are legally demandable
Guingona v. Court of Appeals, G. R. No. 125532, and enforceable." 6 According to Fr. Joaquin Bernas, a noted
July 10, 1998 constitutionalist, courts are mandated to settle disputes between real
conflicting parties through the application of the law. 7 Judicial
Facts: review, which is merely an aspect of judicial power, demands the
following: (1) there must be an actual case calling for the exercise of
Sometime in the last quarter of 1995, the National Bureau of judicial power; (2) the question must be ripe for adjudication; 8 and
Investigation (NBI) conducted an investigation on the alleged (3) the person challenging must have "standing"; that is, he has
participation and involvement of national and local government personal and substantial interest in the case, such that he has
officials in "jueteng" and other forms of illegal gambling. sustained or will sustain direct injury. 9

The case was also the subject of a legislative inquiry/investigation by The first requisite is that there must be before a court an actual case
both the Senate and the House of Representatives. calling for the exercise of judicial power. Courts have no authority to
pass upon issues through advisory opinions or to resolve
hypothetical or feigned problems 10 or friendly suits collusively
In November 1995, one Potenciano Roque, claiming to be an
arranged between parties without real adverse interests. 11 Courts
eyewitness to the networking of . . . national and local politicians
do not sit to adjudicate mere academic questions to satisfy scholarly
Constitutional Law 1 based on the syllabus of Atty. Remoroza 2018-2019 28
Merzy’s Notes Prelim Exam

interest, however intellectually challenging. 12 As a condition July 5, 1994, President Ramos issued proclamation no. 420 which
precedent to the exercise of judicial power, an actual controversy established a SEZ on a portion of Camp John Hay.
between litigants must first exist. 13
Issue: Whether Proclamation no. 420 is constitutional by providing
An actual case or controversy exists when there is a conflict of legal for national and local tax exemption within and granting other
rights or an assertion of opposite legal claims, which can be resolved economic incentives to the John Hay SEZ?
on the basis of existing law and jurisprudence. A justiciable
controversy is distinguished from a hypothetical or abstract Held
difference or dispute, in that the former involves a definite and
concrete dispute touching on the legal relations of parties having No. It is settled that when questions of constituional signifance are
adverse legal interests. A justiciable controversy admits of specific raised, the court can exercise its power of judicial review only if the
relief through a decree that is conclusive in character, whereas an
following requisites are present: (1) existence of actual and
opinion only advises what the law would be upon a hypothetical
state of facts.A conflict of legal rights, an assertion of opposite legal appropriate case; (2) person challenging the act must have the
claims which can be resolved on the basis of existing law and standing to question or have personal/substantial interest in the
jurisprudence case; (3) question of constitutionality must be raised at earliest
opportunit; (4) issue of constitutionality must be the very lis mota of
the case. There is none that have been mentioned in R.A 7227, a
grant of tax exemption to SEZ yet to be established in base areas,
John Hay People’s Alternative Coalition v. Lim,
unlike the grant under Section 12 which provides for tax exemption
G.R.
to the established Subic SEZ.
Facts:
it was held that the controversy must be definite and concrete,
bearing upon the legal relations of parties who are pitted against
During March 13, 1992, Republic Act 7227 were enacted. The R.A.
each other due to their adverse legal interests. It is not enough that
7227 is also known as “ Bases Conversion and Development Act of the controversy exists at the outset; to qualify for adjudication, it is
1992” . This grants Subic SEZ incentives which provides tax and duty necessary that the actual controversy be extant at all stages of the
free importations, exemption of business therein from local and review, not merely at the time the complaint is filed.
national taxes, to other hallmarks of liberated financial and bhsiness No. 119775, October 24, 2003
climate. This also gave authority to the President to create through
Perez v. Provincial Board, 113 SCRA 187
executive proclamation, subject to the concurrence of local
government units directly affected, other Special Economic Zones in
On May 10, 1972, President Ferdinand Marcos nominated petitioner
the areas covered respectively by the Clark Military reservation, the
Honorato Perez for appointment to the position of Provincial Fiscal of
Wallace Air Station in San Fernando, La Union and Camp John Hay.
Nueva Ecija. 2 It appears, however, that the nomination which was

Constitutional Law 1 based on the syllabus of Atty. Remoroza 2018-2019 29


Merzy’s Notes Prelim Exam

submitted to the Commission on Appointments for confirmation was offices they presently hold notwithstanding the filing of certificate of
by-passed upon adjournment sine die of Congress on May 18, 1972. candidacy, subject to the pleasure of the President of the Philippines.
On the following day, May 19, President Marcos designated (Emphasis supplied).
petitioner as acting provincial fiscal.
A petition instituted to establish petitioner's right to an appointive
Hence, the instant petition, petitioner raising the following legal office is rendered moot and academic where his right to said office
questions: has been forfeited by his filing of a certificate of candidacy to an
elective office.
1) Whether or not respondent Provincial Board has the power to
pass and enact a resolution not recognizing herein petitioner as ACCORDINGLY, this petition is hereby dismissed. No costs
acting provincial fiscal despite the fact that the latter has assumed
such office pursuant to a designation lawfully extended to him by the
President of the Philippines.
Lacson v. Perez, G.R. No. 147780, May 10, 2001
2) Whether or not respondent Provincial Board has the power to
defy and/or pass judgment on the validity of the said designation FACTS:
President Macapagal-Arroyo declared a State of Rebellion
and assumption.
(Proclamation No. 38) on May 1, 2001 as well as General Order No.
1 ordering the AFP and the PNP to suppress the rebellion in the NCR.
We deem it unnecessary to pass upon the issues raised, this petition Warrantless arrests of several alleged leaders and promoters of the
having become moot and academic. We take cognizance of the fact “rebellion” were thereafter effected. Petitioner filed for prohibition,
that petitioner Perez filed his certificate of candidacy for the office of injunction, mandamus and habeas corpus with an application for the
mayor of Cabanatuan City in the local elections of January 30, 1980. issuance of temporary restraining order and/or writ of preliminary
12
The mere filing of a certificate of candidacy constitutes forfeiture injunction. Petitioners assail the declaration of Proc. No. 38 and the
warrantless arrests allegedly effected by virtue thereof. Petitioners
of his right to the controverted office under Section 29 of the
furthermore pray that the appropriate court, wherein the information
Election Code of 1978 which provides: against them were filed, would desist arraignment and trial until this
instant petition is resolved. They also contend that they are allegedly
SEC. 29. Candidates holding appointive office or position.— Every faced with impending warrantless arrests and unlawful restraint
person holding a public appointive office or position, including active being that hold departure orders were issued against them.
members of the Armed Forces of the Philippines, and officers and
ISSUES: Whether or not Proclamation No. 38 is valid, along with
employees in government-owned or controlled corporations, shall
the warrantless arrests and hold departure orders allegedly effected
ipso facto cease in his office or position on the date he files his by the same.
certificate of candidacy. Members of the Cabinet shall continue in the
Constitutional Law 1 based on the syllabus of Atty. Remoroza 2018-2019 30
Merzy’s Notes Prelim Exam

ISSUE:
RULING: Whether or not the PP 1017 and G.O. No. 5 is constitutional.
President Macapagal-Arroyo ordered the lifting of Proc. No. 38 on
May 6, 2006, accordingly the instant petition has been rendered RULING:
moot and academic. Respondents have declared that the Justice
Department and the police authorities intend to obtain regular The Court holds that President Arroyo’s issuance of PP 1021 did not
render the present petitions moot and academic. During the eight
warrants of arrests from the courts for all acts committed prior to
(8) days that PP 1017 was operative, the police officers, according to
and until May 1, 2001. Under Section 5, Rule 113 of the Rules of petitioners, committed illegal acts in implementing it. Are PP 1017
Court, authorities may only resort to warrantless arrests of persons and G.O. No. 5 constitutional or valid? Do they justify these
suspected of rebellion in suppressing the rebellion if the alleged illegal acts? These are the vital issues that must be
circumstances so warrant, thus the warrantless arrests are not based resolved in the present petitions. It must be stressed that "an
on Proc. No. 38. Petitioner’s prayer for mandamus and prohibition is unconstitutional act is not a law, it confers no rights, it
improper at this time because an individual warrantlessly arrested imposes no duties, it affords no protection; it is in legal
contemplation, inoperative."30
has adequate remedies in law:

The "moot and academic" principle is not a magical formula that can
automatically dissuade the courts in resolving a case. Courts will
David v. Macapagal-Arroyo, G.R. No. 171396, May decide cases, otherwise moot and academic, if: first, there is a grave
violation of the Constitution;31 second, the exceptional character of
3, 2006
the situation and the paramount public interest is involved;32 third,
when constitutional issue raised requires formulation of controlling
David, et al. assailed PP 1017 on the grounds that (1) it encroaches
principles to guide the bench, the bar, and the public;33 and fourth,
on the emergency powers of Congress; (2) it is a subterfuge to avoid
the constitutional requirements for the imposition of martial law; and the case is capable of repetition yet evading review
(3) it violates the constitutional guarantees of freedom of the press,
of speech and of assembly. They alleged “direct injury” resulting All the foregoing exceptions are present here and justify this Court’s
from “illegal arrest” and “unlawful search” committed by police assumption of jurisdiction over the instant petitions. Petitioners
operatives pursuant to PP 1017. alleged that the issuance of PP 1017 and G.O. No. 5 violates the
Constitution. There is no question that the issues being raised affect
During the hearing, the Solicitor General argued that the issuance of the public’s interest, involving as they do the people’s basic rights to
PP 1017 and GO 5 have factual basis, and contended that the intent freedom of expression, of assembly and of the press. Moreover, the
of the Constitution is to give full discretionary powers to the Court has the duty to formulate guiding and controlling constitutional
President in determining the necessity of calling out the armed precepts, doctrines or rules. It has the symbolic function of
forces. The petitioners did not contend the facts stated b the educating the bench and the bar, and in the present petitions, the
Solicitor General. military and the police, on the extent of the protection given by
constitutional guarantees.35 And lastly, respondents’ contested

Constitutional Law 1 based on the syllabus of Atty. Remoroza 2018-2019 31


Merzy’s Notes Prelim Exam

actions are capable of repetition. Certainly, the petitions are subject President has full discretionary power to call out the armed forces
to judicial review. and to determine the necessity for the exercise of such power.

Note:

Saniakas v. Executive Secretary, G.R. No. 159085, iii) However, the moot and academic principle is not a magical
February 3, 2004 formula that can automatically dissuade the courts from resolving a
case. In David v. Macapagal-Arroyo, supra., it was held that courts
Facts: During the wee hours of July 27, 2003, some three-hundred will still decide cases otherwise moot and academic if: (a) there is a
junior officers and enlisted men of the AFP, acting upon instigation, grave violation of the Constitution [Province of Batangas v. Romulo,
command and direction of known and unknown leaders have seized supra.]; (b) there is an exceptional character of the situation and
the Oakwood Building in Makati. Publicly, they complained of the paramount public interest is involved [Lacson v. Perez, supra.]
corruption in the AFP and declared their withdrawal of support for (c) the constitutional issues raised require formulation of controlling
the government, demanding the resignation of the President, principles to guide the bench, the bar and the public [Salonga v.
Secretary of Defense and the PNP Chief. These acts constitute a Pano, supra.]; and (d) the case is capable of repetition yet evasive
violation of Article 134 of the Revised Penal Code, and by virtue of of review [Saniakas v. Executive Secretary,
Proclamation No. 427 and General Order No. 4, the Philippines was .
declared under the State of Rebellion. Negotiations took place and
the officers went back to their barracks in the evening of the same
day. On August 1, 2003, both the Proclamation and General Orders
were lifted, and Proclamation No. 435, declaring the Cessation of the Salonga v. Pano, 134 SCRA 438
State of Rebellion was issued.
Issue: Facts:

Whether or Not Proclamation No. 427 and General Order No. 4 are The petitioner invokes the constitutionally protected right to life and
constitutional? liberty guaranteed by the due process clause, alleging that no prima
facie case has been established to warrant the filing of an
Whether or Not the petitioners have a legal standing or locus standi information for subversion against him. Petitioner asks the Court to
to bring suit? prohibit and prevent the respondents from using the iron arm of the
law to harass, oppress, and persecute him, a member of the
democratic opposition in the Philippines.
Held: The Court rendered that the both the Proclamation No. 427 The case roots backs to the rash of bombings which occurred in the
and General Order No. 4 are constitutional. Section 18, Article VII Metro Manila area in the months of August, September and October
does not expressly prohibit declaring state or rebellion. The of 1980. Victor Burns Lovely, Jr, one of the victims of the bombing,
President in addition to its Commander-in-Chief Powers is conferred implicated petitioner Salonga as one of those responsible.
by the Constitution executive powers. It is not disputed that the

Constitutional Law 1 based on the syllabus of Atty. Remoroza 2018-2019 32


Merzy’s Notes Prelim Exam

Held: 1. No. The Court had already deliberated on this case, a predicted benefits as reflected in the agreement and as viewed by
consensus on the Court’s judgment had been arrived at, and a the signatory Senators, a “free market” espoused by WTO.
draft ponencia was circulating for concurrences and separate
opinions, if any, when on January 18, 1985, respondent Judge Petitioners on the other hand viewed the WTO agreement as one
Rodolfo Ortiz granted the motion of respondent City Fiscal Sergio that limits, restricts and impair Philippine economic sovereignty and
Apostol to drop the subversion case against the petitioner. Pursuant
legislative power. That the Filipino First policy of the Constitution
to instructions of the Minister of Justice, the prosecution restudied its
evidence and decided to seek the exclusion of petitioner Jovito was taken for granted as it gives foreign trading intervention.
Salonga as one of the accused in the information filed under the
Held: on the challenge posed by the
questioned resolution.
petitioners that the concurrence of the Senate in the WTO
moot and academic nature, it has on several occasions rendered
Agreement violated the Constitution, particularly Sec. 19, Art. II
elaborate decisions in similar cases where mootness was clearly
(which mandates the development of a self-reliant and independent
apparent.
national economy), the Supreme Court held that this was a
justiciable controversy, because where an action of the Legislature is
The Court also has the duty to formulate guiding and controlling alleged to have infringed the Constitution, it becomes not only the
constitutional principles, precepts, doctrines, or rules. It has the right but the duty of the Judiciary to settle the dispute.
symbolic function of educating bench and bar on the extent of
protection given by constitutional guarantees.
People v. Vera, 65 Phil. 56
In dela Camara vs Enage (41 SCRA 1), the court ruled that:
Facts: Mariano Cu Unjieng was convicted by the trial court in
“The fact that the case is moot and academic should not preclude
this Tribunal from setting forth in language clear and unmistakable, Manila. He filed for reconsideration and four motions for new trial
the obligation of fidelity on the part of lower court judges to the but all were denied. He then elevated to the Supreme Court and the
unequivocal command of the Constitution that excessive bail shall Supreme Court remanded the appeal to the lower court for a new
not be required.” trial. While awaiting new trial, he appealed for probation alleging
that the he is innocent of the crime he was convicted of. The Judge
of the Manila CFI directed the appeal to the Insular Probation Office.
Tanada v. Angara, 272 SCRA 18
The IPO denied the application. However, Judge Vera upon another
The WTO opens access to foreign markets, especially its major request by petitioner allowed the petition to be set for hearing. The
trading partners, through the reduction of tariffs on its exports, City Prosecutor countered alleging that Vera has no power to place
particularly agricultural and industrial products. Thus, provides new Cu Unjieng under probation because it is in violation of Sec. 11 Act
opportunities for the service sector cost and uncertainty associated No. 4221 which provides that the act of Legislature granting
with exporting and more investment in the country. These are the provincial boards the power to provide a system of probation to
convicted person.

Constitutional Law 1 based on the syllabus of Atty. Remoroza 2018-2019 33


Merzy’s Notes Prelim Exam

Held communicated to the Senate that the executive and AFP officials
would not be able to attend the meeting since the President has not
which states that a person who impugns the yet given her consent. Despite the lack of consent, Col. Balutan and
validity of a statute must have a personal and substantial interest in Brig. Gen. Gudani, among all the AFP officials invited, attended the
the case such that he has sustained or will sustain direct injury as a investigation. Both faced court marshal for such attendance.
result.
To this enumeration may be added the ruling in People v. Hence, these petitions.
Vera, supra., where the Supreme Court declared that the
Government of the Philippines is a proper party to question the ISSUES:
validity of its own laws, because more than any one, it should be
concerned with the constitutionality of its acts. In that case, it was  Whether or not EO 464 contravenes the power of inquiry
held that the government has substantial interest in having the vested in Congress
Probation Law declared as unconstitutional, because more than the  Whether or not EO 464 violates the right of the people to
damage caused by the illegal expenditure of public funds is the information on matters of public concern
mortal would inflicted upon  Whether or not respondents have committed grave abuse of
the fundamental law by the enforcement of an invalid statute. discretion when they implemented EO 464 prior to its
publication in a newspaper of general circulation

RULING:
Senate v. Executive Secretary, G.R. No. 169777,
April 20, 2006 ESSENTIAL REQUISITES OF JUDICIAL REVIEW:

The Senate Committees sent invitations to various officials of the 1. there must be an actual case or controversy calling for the
Executive Department and AFP officials for them to appear before exercise of judicial power;
Senate on Sept. 29, 2005. Before said date arrived, Executive Sec. 2. the person challenging the act must have standing to
Ermita sent a letter to Senate President Drilon, requesting for a challenge the validity of the subject act or issuance;
postponement of the hearing on Sept. 29 in order to “afford said otherwise stated, he must have a personal and substantial
officials ample time and opportunity to study and prepare for the interest in the case such that he has sustained, or will
various issues so that they may better enlighten the Senate sustain, direct injury as a result of its enforcement;
Committee on its investigation.” Senate refused the request. 3. the question of constitutionality must be raised as the
earliest opportunity; and
On Sept. 28, 2005, the President issued EO 464, effective 4. the issue of constitutionality must be the very lis mota of the
immediately, which, among others, mandated that “all heads of case.
departments of the Executive Branch of the government shall secure
the consent of the President prior to appearing before either House
of Congress.” Pursuant to this Order, Executive Sec. Ermita
LEGAL STANDING
Constitutional Law 1 based on the syllabus of Atty. Remoroza 2018-2019 34
Merzy’s Notes Prelim Exam

has not withheld her consent or prohibited the


Standing of the Senate appearance of the officials concerned immaterial in
determining the existence of an actual case or
That the Senate of the Philippines has a fundamental right essential
controversy insofar as EO 464 is concerned. For EO 464
not only for intelligent public decision-making in a democratic
system, but more especially for sound legislation is not disputed. EO does not require either a deliberative withholding of
464, however, allegedly stifles the ability of the members of consent or an express prohibition issuing from the
Congress to access information that is crucial to law-making. Verily, President in order to bar officials from appearing before
the Senate, including its individual members, has a substantial and Congress.
direct interest over the outcome of the controversy and is the proper
party to assail the constitutionality of EO 464. Indeed, legislators As the implementation of the challenged order has
have standing to maintain inviolate the prerogative, powers and
already resulted in the absence of officials invited to the
privileges vested by the Constitution in their office and are allowed
to sue to question the validity of any official action which they claim hearings of petitioner Senate of the Philippines, it would
infringes their prerogatives as legislators. make no sense to wait for any further event before
considering the present case ripe for adjudication.
Standing of an ordinary citizen Indeed, it would be sheer abandonment of duty if this
Court would now refrain from passing on the
It is well-settled that when suing as a citizen, the interest of the constitutionality of EO 464.
petitioner in assailing the constitutionality of laws, presidential
decrees, orders and other regulations must be direct and personal.
In Francisco v. House of Representatives, this Court held that when
The power of inquiry
the proceeding involves the assertion of a public right, the mere fact
that he is a citizen satisfies the requirement of personal interest.
The Congress power of inquiry is expressly recognized in
Requisites for transcendental importance: Establish (1) the character Sec. 21, Art. VI. But as early as 1950 (the 1935
of the funds (that it is public) or other assets involved in the case, Constitution did not contain a similar provision) in
(2) the presence of a clear case of disregard of a constitutional or Arnault v. Nazareno, the Court already recognized that
statutory prohibition by the public respondent agency or
the power of inquiry is inherent in the power to legislate.
instrumentality of the government, and (3) the lack of any party with
xxx
a more direct and specific interest in raising the questions being
raised.
That this power of inquiry is broad enough to cover
ACTUAL CASE/CONTROVERSY officials of the executive branch may be deduced from
the same case. The power of inquiry...is co-extensive
The Court finds respondents’ assertion that the President with the power to legislate. The matters which may be a

Constitutional Law 1 based on the syllabus of Atty. Remoroza 2018-2019 35


Merzy’s Notes Prelim Exam

proper subject of legislation and those which may be a speculation on the part of the person invited on whether
proper subject of investigation are one. It follows that the inquiry is in aid of legislation.
the operation of government, being a legitimate subject
for legislation, is a proper subject for investigation. Sec. 21, Art. VI likewise establishes crucial safeguards
that proscribe the legislative power of inquiry. The
xxx the power of inquiry, “with process to enforce it,” is provision requires that the inquiry be done in accordance
grounded on the necessity of the information in the with the Senate or House’s duly published rules of
legislative process. If the information possessed by procedure, necessarily implying the constitutional
executive officials on the operation of their offices is infirmity of an inquiry conducted without duly published
necessary for wise legislation on that subject, by parity rules of procedure. Sec. 21 also mandates that the rights
of reasoning, Congress has the right to that information of persons appearing in or affected by such inquiries be
and the power to compel the disclosure thereof. respected, an imposition that obligates Congress to
adhere to the guarantees in the Bill of Rights.
The power of inquiry is subject to judicial review

xxx the right of Congress to conduct inquiries in aid of Automotive Industry Workers Alliance v. Romulo,
legislation is, in theory, no less susceptible to abuse than G.R. No. 157509, January 18,2005
executive or judicial power. It may thus be subjected to
judicial review pursuant to the Court’s certiorari powers
under Sec. 1, Art. VIII.
During the time of President Corazon C. Aquino, and while she was
endowed with legislative functions after EDSA I, Executive Order 292
For one...the inquiry itself might not properly be in aid of
was issued whereby the NLRC became an agency attached to the
legislation, and thus beyond the constitutional power of
DOLE for policy and program coordination and for administrative
Congress. Such inquiry could not usurp judicial
supervision. On 2 March 1989, Article 213 of the Labor Code was
functions. Parenthetically, one possible way for Congress
expressly amended by Republic Act 6715 declaring that the NLRC
to avoid such a result...is to indicate in its invitations to
was to be attached to the DOLE for program and policy coordination
the public officials concerned, or to any person for that
only while the administrative supervision over the NLRC, its regional
matter, the possible needed statute which prompted the
branches and personnel, was turned over to the NLRC Chairman.
need for the inquiry. Given such statement in its
The subject EO 185, in authorizing the Secretary of Labor to exercise
investigations, along with the usual indication of the
administrative supervision over the NLRC, its regional branches and
subject of inquiry and the questions relative to and in
personnel, allegedly reverted to the pre-RA 6715 set-up, amending
furtherance thereof, there would be less room for
Constitutional Law 1 based on the syllabus of Atty. Remoroza 2018-2019 36
Merzy’s Notes Prelim Exam

the latter law which only Congress can do. Alberto Romulo (in his system on the basis of serious moral and ethical considerations. It
capacity as Executive Secretary) and Patricia Sto. Tomas (in her submitted that said contract of lease violated Section 1 of R. A. No.
capacity as Secretary of Labor and Employment), as represented by 1169, as amended by B. P. Blg. 42.
the Office of the Solicitor General, opposed the petition on
procedural and substantive grounds. Respondents contended, among others, that, the contract does not
violate the Foreign Investment Act of 1991; that the issues of
ISSUE: Whether the Unions -- which contend that they are suing for wisdom, morality and propriety of acts of the executive department
and in behalf of their members (more or less 50,000 workers) –-- are beyond the ambit of judicial reviews; and that the petitioners
has the requisite standing. have no standing to maintain the instant suit.

HELD: NO. Legal standing or locus standi is defined as a “personal ISSUES:


and substantial interest in the case such that the party has sustained 1. Whether or not petitioners have the legal standing to file the
or will sustain direct injury as a result of the governmental act that is instant petition.
being challenged.” For a citizen to have standing, he must establish 2. Whether or not the contract of lease is legal and valid.
that he has suffered some actual or threatened injury as a result of
the allegedly illegal conduct of the government; the injury is fairly RULING: As to the preliminary issue, the Court resolved to set aside
traceable to the challenged action; and the injury is likely to be the procedural technicality in view of the importance of the issues
redressed by a favorable action. Herein, the Unions have not shown raised. The Court adopted the liberal policy on locus standi to allow
that they have sustained or are in danger of sustaining any personal the ordinary taxpayers, members of Congress, and even association
injury attributable to the enactment of EO 185. of planters, and non-profit civic organizations to initiate and
prosecute actions to question the validity or constitutionality of laws,
acts, decisions, or rulings of various government agencies or
Kilosbayan v. Guingona, 232 SCRA 110 instrumentalities.

Facts: This is a special civil action for prohibition and injunction,


As to the substantive issue, the Court agrees with the petitioners
with a prayer for a temporary restraining order and preliminary
whether the contract in question is one of lease or whether the
injunction which seeks to prohibit and restrain the implementation of
PGMC is merely an independent contractor should not be decided on
the Contract of Lease executed by the PCSO and the Philippine
the basis of the title or designation of the contract but by the intent
Gaming Management Corporation in connection with the on-line
of the parties, which may be gathered from the provisions of the
lottery system, also know as lotto.
contract itself. Animus homini est anima scripti. The intention of the
party is the soul of the instrument.
Petitioners strongly opposed the setting up of the on-line lottery

Constitutional Law 1 based on the syllabus of Atty. Remoroza 2018-2019 37


Merzy’s Notes Prelim Exam

Therefore the instant petition is granted and the challenged Contract President Fidel V. Ramos, through then Executive Secretary Ruben
of Lease is hereby declared contrary to law and invalid. Torres, approved the JVA.

Issue: Won petitioner has a locus standi


Chavez v. Public Estates Authority and Amari,
G.R. No. 133250, July 09, 2002 Held: 1. We rule that the signing and of the Amended JVA by PEA
and AMARI and its approval by the President cannot operate to moot
Facts: On February 4, 1977, then President Ferdinand E. Marcos the petition and divest the Court of its jurisdiction.
issued Presidential Decree No. 1084 creating PEA. PD No. 1084
tasked PEA "to reclaim land, including foreshore and submerged PEA and AMARI have still to implement the Amended JVA. The
areas," and "to develop, improve, acquire, lease and sell any and all prayer to enjoin the signing of the Amended JVA on constitutional
kinds of lands." On the same date, then President Marcos issued grounds necessarily includes preventing its implementation if in the
Presidential Decree No. 1085 transferring to PEA the "lands meantime PEA and AMARI have signed one in violation of the
reclaimed in the foreshore and offshore of the Manila Bay" under the Constitution. Petitioner's principal basis in assailing the renegotiation
Manila-Cavite Coastal Road and Reclamation Project (MCCRRP). of the JVA is its violation of the Section 3, Article XII of the
Constitution, which prohibits the government from alienating lands
On January 19, 1988, then President Corazon C. Aquino issued of the public domain to private corporations. The Amended JVA is
Special Patent No. 3517, granting and transferring to PEA "the not an ordinary commercial contract but one which seeks to transfer
parcels of land so reclaimed under the Manila-Cavite Coastal Road title and ownership to 367.5 hectares of reclaimed lands and
and Reclamation Project (MCCRRP) containing a total area of one submerged areas of Manila Bay to a single private corporation.
million nine hundred fifteen thousand eight hundred ninety four
(1,915,894) square meters." Subsequently, on April 9, 1988, the Ordinary taxpayers have a right to initiate and prosecute actions
Register of Deeds of the Municipality of Parañaque issued Transfer questioning the validity of acts or orders of government agencies or
Certificates of Title Nos. 7309, 7311, and 7312, in the name of PEA, instrumentalities, if the issues raised are of 'paramount public
covering the three reclaimed islands known as the "Freedom Islands" interest,' and if they 'immediately affect the social, economic and
located at the southern portion of the Manila-Cavite Coastal Road, moral well being of the people.'
Parañaque City.
We rule that since the instant petition, brought by a citizen,
PEA and AMARI entered into the JVA through negotiation without involves the enforcement of constitutional rights — to information
public bidding. On April 28, 1995, the Board of Directors of PEA, in and to the equitable diffusion of natural resources — matters of
its Resolution No. 1245, confirmed the JVA. On June 8, 1995, then transcendental public importance, the petitioner has the requisite
locus standi.

Constitutional Law 1 based on the syllabus of Atty. Remoroza 2018-2019 38


Merzy’s Notes Prelim Exam

Matibag v. Benipayo, G.R. No. 149036, April 2, subject to confirmation by the Commission on Appointments does
2002 not alter its permanent character. The Constitution itself makes an
ad interim appointment permanent in character by making it
effective until disapproved by the Commission on Appointments or
until the next adjournment of Congress.
In his capacity as COMELEC Chairman, Benipayo issued a
Memorandum dated April 11, 2001 addressed to petitioner as
It was held that the earliest opportunity to raise a constitutional
Director IV of the EID and to Cinco as Director III also of the EID,
issue is to raise it In the pleadings before a
designating Cinco Officer-in-Charge of the EID and reassigning competent court that can resolve the same, such that, If not raised
petitioner to the Law Department. COMELEC EID Commissioner-in- in the pleadings, it cannot be considered at the trial and, if not
Charge Mehol K. Sadain objected to petitioner’s reassignment in a considered in the trial, it cannot be considered on appeal
Memorandum dated April 14, 2001 addressed to the COMELEC en
banc. Specifically, Commissioner Sadain questioned Benipayo’s
failure to consult the Commissioner-in-Charge of the EID in the
Estarija v. Ranada, G.R. No. 159314, June 26,
reassignment of petitioner.
2006
During the pendency of her complaint before the Law Department,
petitioner filed the instant petition questioning the appointment and Facts:
the right to remain in office of Benipayo, Borra and Tuason, as
Chairman and Commissioners of the COMELEC, respectively. On August 31, 2000, the Ombudsman rendered a decision in the
Petitioner claims that the ad interim appointments of Benipayo, administrative case, finding Estarija guilty of dishonesty and grave
Borra and Tuason violate the constitutional provisions on the misconduct. Estarija seasonably filed a motion for reconsideration.
independence of the COMELEC. Estarija claimed that dismissal was unconstitutional since the
Ombudsman did not have direct and immediate power to remove
ISSUES: Whether or not the assumption of office by Benipayo, government officials, whether elective or appointive, who are not
Borra and Tuason on the basis of the ad interim appointments issued removable by impeachment. He maintains that under the 1987
by the President amounts to a temporary appointment prohibited by Constitution, the Ombudsman’s administrative authority is merely
Section 1 (2), Article IX-C of the Constitution. recommendatory, and that Republic Act No. 6770, otherwise known
as "The Ombudsman Act of 1989", is unconstitutional because it
RULING: We find petitioner’s argument without merit. An ad gives the Office of the Ombudsman additional powers that are not
interim appointment is a permanent appointment because it takes provided for in the Constitution
effect immediately and can no longer be withdrawn by the President
once the appointee has qualified into office. The fact that it is Held:

Constitutional Law 1 based on the syllabus of Atty. Remoroza 2018-2019 39


Merzy’s Notes Prelim Exam

where it upheld the constitutionality of Sections 15, 21 and 25 of RTC and CA favored Fertiphil holding that it is an exercise of the
R.A. 6770, and ruled that the Ombudsman has the constitutional power of taxation ad is as such because it is NOT for public purpose
power to directly remove from governmentservice an erring public as PPI is a private corporation
official, other than a Member of Congress or of the Judiciary.
Held:

Planters Products v. Fertiphll Corporation, G.R.


We shall first tackle the procedural issues of locus standi and the
No. 166006, March 14, 2008
jurisdiction of the RTC to resolve constitutional issues.
FACTS:
Fertiphil has locus standi because it suffered direct injury; doctrine of
standing is a mere procedural technicality which may be waived.
President Ferdinand Marcos, exercising his legislative powers, issued
LOI No. 1465 which provided, among others, for the imposition of a PPI argues that Fertiphil has no locus standi to question the
capital recovery component (CRC) on the domestic sale of all grades constitutionality of LOI No. 1465 because it does not have a
of fertilizers which resulted in having Fertiphil paying P 10/bag sold "personal and substantial interest in the case or will sustain direct
to the Fertilizer and Perticide Authority (FPA). injury as a result of its enforcement."21 It asserts that Fertiphil did
not suffer any damage from the CRC imposition because "incidence
FPA remits its collection to Far East Bank and Trust Company who of the levy fell on the ultimate consumer or the farmers themselves,
not on the seller fertilizer company."22
applies to the payment of corporate debts of Planters Products Inc.
(PPI)
We cannot agree. The doctrine of locus standi or the right of
appearance in a court of justice has been adequately discussed by
After the Edsa Revolution, FPA voluntarily stopped the imposition of
this Court in a catena of cases. Succinctly put, the doctrine requires
the P10 levy. Upon return of democracy, Fertiphil demanded a
a litigant to have a material interest in the outcome of a case. In
refund but PPI refused. Fertiphil filed a complaint for collection and
private suits, locus standi requires a litigant to be a "real party in
damages against FPA and PPI with the RTC on the ground that LOI
interest," which is defined as "the party who stands to be benefited
No. 1465 is unjust, unreaonable oppressive, invalid and unlawful
or injured by the judgment in the suit or the party entitled to the
resulting to denial of due process of law.
avails of the suit."
FPA answered that it is a valid exercise of the police power of the
state in ensuring the stability of the fertilizing industry in the country Judicial review of official acts on the ground of unconstitutionality
and that Fertiphil did NOT sustain damages since the burden may be sought or availed of through any of the actions cognizable
imposed fell on the ultimate consumers. by courts of justice, not necessarily in a suit for declaratory relief.
Such review may be had in criminal actions, as in People v. Ferrer 35
involving the constitutionality of the now defunct Anti-Subversion
Constitutional Law 1 based on the syllabus of Atty. Remoroza 2018-2019 40
Merzy’s Notes Prelim Exam

law, or in ordinary actions, as in Krivenko v. Register of Deeds 36 BSP. An appointment to the said position is not among the
involving the constitutionality of laws prohibiting aliens from appointments which have to be confirmed by the COA under Section
acquiring public lands. The constitutional issue, however, (a) must 16 of Article 7 of the Constitution. Congress cannot by law expand
be properly raised and presented in the case, and (b) its resolution is the confirmation powers of the Commission on Appointments and
necessary to a determination of the case, i.e., the issue of require confirmation of appointment of other government officials
constitutionality must be the very lis mota presented not expressly mentioned in the first sentence of Section 16 of Article
7 of the Constitution.

the Courtrefrained from passing upon the constitutionality of the


Tarrosa v. Singson, 232 SCRA 553
assailed provision in R.A,7653 (which provided that the appointment
of the Governor of the Bangko Sentral
FACTS: Gabriel C. Singson was appointed Governor of the Bangko
ng Pilipinas should be confirmed by the Commission on
Sentral by President Fidel V. Ramos in 1993. Jesus Armando Tarrosa, Appointments) because of the principle that bars judicial inquiry into
as a "taxpayer", filed a petition for prohibition questioning the a constitutional question unless the resolution thereof is
appointment of Singson for not having been confirmed by the indispensable to the determination of the case.
Commission on Appointments as required by the provisions of
Section 6 of R.A. No. 7653, which established the Bangko Sentral as
the Central Monetary Authority of the Philippines. The Secretary of
Budget and Management was impleaded for disbursing public funds
5. Political Questions / Justiciable Questions
in payment of the salaries and emoluments of respondent
Singson. In their comment, respondents claim that Congress
exceeded its legislative powers in requiring the confirmation by the
CA of the appointment of the Governor of the Bangko Sentral. They Velarde v. SJS, 428 SCRA 283 (2004)
contend that an appointment to the said position is not among the
Facts:
appointments which have to be confirmed by the CA, citing Section
16 of Article VI of the Constitution. On January 28, 2003, SJS filed a Petition for Declaratory Relief
before the RTC-Manila against Velarde and his co-respondents
ISSUE: Whether or not the Governor of the BSP is subject to COA’s Eminence, Jaime Cardinal Sin, Executive Minister Eraño Manalo,
confirmation. Brother Eddie Villanueva and Brother Eliseo F. Soriano.
SJS, a registered political party, sought the interpretation of several
HELD: No. Congress exceeded its legislative powers in requiring the constitutional provisions, specifically on the separation of church and
state; and a declaratory judgment on the constitutionality of the acts
confirmation by the COA of the appointment of the Governor of the
of religious leaders endorsing a candidate for an elective office, or

Constitutional Law 1 based on the syllabus of Atty. Remoroza 2018-2019 41


Merzy’s Notes Prelim Exam

urging or requiring the members of their flock to vote for a specified The plaintiffs in this case are all minors duly represented and joined
candidate. by their parents. The first complaint was filed as a taxpayer's class
-The petitioner filed a Motion to dismiss before the trial court suit at the Branch 66 (Makati, Metro Manila), of the Regional Trial
owing to the fact that alleged that the questioned SJS Petition did Court, National capital Judicial Region against defendant
not state a cause of action and that there was no justiciable (respondent) Secretary of the Department of Environment and
controversy. Natural Reasources (DENR). Plaintiffs alleged that they are entitled
to the full benefit, use and enjoyment of the natural resource
Held: treasure that is the country's virgin tropical forests. They further
. NO. A justiciable controversy to an existing case asseverate that they represent their generation as well as
or controversy that is appropriate or ripe for judicial generations yet unborn and asserted that continued deforestation
determination, not one that is conjectural or merely have caused a distortion and disturbance of the ecological balance
anticipatory. A petition filed with the trial court should contain a and have resulted in a host of environmental tragedies.
plain, concise and direct statement of the ultimate facts on
which the party pleading relies for his claim. Plaintiffs prayed that judgement be rendered ordering the
respondent, his agents, representatives and other persons acting in
The SJS Petition fell short of the requirements to constitutue a his behalf to cancel all existing Timber License Agreement (TLA) in
jusiciable controversy. Why? the country and to cease and desist from receiving, accepting,
a. It stated no ultimate facts. The petition simply theorized that processing, renewing or approving new TLAs.
the people elected who were endorsed by these religious leaders
might become beholden to the latter. ISSUES:
b. It did not sufficiently state a declaration of its
rights and duties, what specific legal right of the petitioner was (1) Whether or not the plaintiffs have a cause of action.
violated by the respondents therein, and what particular act or acts (2) Whether or not the complaint raises a political issue.
of the latter were in breach of its rights, the law or the constitution, (3) Whether or not the original prayer of the plaintiffs result in the
c. The petition did not pray for a stoppage of violated impairment of contracts.
rights (duh, wala ngang rights na sinabi eh). It merely sought an
opinion of the trial court. However, courts are proscribed from RULING:
rendering an advisory opinion. (tantamount to making laws,
remember the questionability of justice panganiban’s guidelines for First Issue: Cause of Action.
article 36 of the family code)
Respondents aver that the petitioners failed to allege in their
Oposa v. Factoran, 224 SCRA 792 (1993) complaint a specific legal right violated by the respondent Secretary
for which any relief is provided by law. The Court did not agree with
Facts: this. The complaint focuses on one fundamental legal right -- the
right to a balanced and healthful ecology which is incorporated in
Section 16 Article II of the Constitution. The said right carries with it

Constitutional Law 1 based on the syllabus of Atty. Remoroza 2018-2019 42


Merzy’s Notes Prelim Exam

the duty to refrain from impairing the environment and implies,


among many other things, the judicious management and Third Issue: Violation of the non-impairment clause.
conservation of the country's forests. Section 4 of E.O. 192 expressly
mandates the DENR to be the primary government agency The Court held that the Timber License Agreement is an instrument
responsible for the governing and supervising the exploration, by which the state regulates the utilization and disposition of forest
utilization, development and conservation of the country's natural resources to the end that public welfare is promoted. It is not a
resources. The policy declaration of E.O. 192 is also substantially re- contract within the purview of the due process clause thus, the non-
stated in Title XIV Book IV of the Administrative Code of 1987. Both impairment clause cannot be invoked. It can be validly withdraw
E.O. 192 and Administrative Code of 1987 have set the objectives whenever dictated by public interest or public welfare as in this case.
which will serve as the bases for policy formation, and have defined The granting of license does not create irrevocable rights, neither is
the powers and functions of the DENR. Thus, right of the petitioners it property or property rights.
(and all those they represent) to a balanced and healthful ecology is
as clear as DENR's duty to protect and advance the said right.

A denial or violation of that right by the other who has the Vinuya v. Romulo, 619 SCRA 533 (2010)
correlative duty or obligation to respect or protect or respect the
same gives rise to a cause of action. Petitioners maintain that the
Defensor-Santiago v. Guingona, G.R. No. 134577,
granting of the TLA, which they claim was done with grave abuse of
discretion, violated their right to a balance and healthful ecology. November 18, 1998
Hence, the full protection thereof requires that no further TLAs
Facts: During the first regular session of the eleventh Congress,
should be renewed or granted.
Senator Fernan was declared the duly elected President of the
After careful examination of the petitioners' complaint, the Court Senate by a vote of 20 to 2. Senator Tatad manifested that, with the
finds it to be adequate enough to show, prima facie, the claimed agreement of Senator Santiago, allegedly the only other member of
violation of their rights. the minority, he was assuming the position of minority leader. He
explained that those who had voted for Senator Fernan comprised
the majority, while only those who had voted for him, the losing
Second Issue: Political Issue.
nominee, belonged to the minority. Senator Flavier manifested that
Second paragraph, Section 1 of Article VIII of the constitution the senators belonging to the Lakas-NUCD-UMDP Party numbering 7
provides for the expanded jurisdiction vested upon the Supreme and, thus, also a minority had chosen Senator Guingona as the
Court. It allows the Court to rule upon even on the wisdom of the minority leader. Thereafter, the majority leader informed the body
decision of the Executive and Legislature and to declare their acts as that he was in receipt of a letter signed by the 7 Lakas-NUCD-UMDP
invalid for lack or excess of jurisdiction because it is tainted with senators, stating that they had elected Senator Guingona as the
grave abuse of discretion.
minority leader. By virtue thereof, the Senate President formally
recognized Senator Guingona as the minority leader of the Senate.
Constitutional Law 1 based on the syllabus of Atty. Remoroza 2018-2019 43
Merzy’s Notes Prelim Exam

Senators Santiago and Tatad filed a petition for quo warranto, imposes no duties; it affords no protection; it creates no office; it is
alleging that Senator Guingona had been usurping, unlawfully inoperative, as if it had not been passed at all. See Art. 7, Civil Code
holding and exercising the position of Senate minority leader, a of the Philippines.
position that, according to them, rightfully belonged to Senator
b. Modern view
Tatad
Courts simply refuse to recognize the law and determine the rights
Issue: (2) Whether or not there is an actual violation of the
of the parties as if the statute had no existence
Constitution

Held: where Senator Defensor-Santiago questioned the Serrano de Agbayani v. PNB, 35 SCRA 429
election of Senator Guingona as Minority Floor Leader, the Supreme
Court said that it “has no authority to interfere and unilaterally FACTS: Plaintiff obtained a loan from PNB dated July 19, 1939,
intrude into that exclusive realm, without running afoul of maturing on July 19, 1944, secured by real estate mortgage. On July
constitutional principles that it is bound to protect and uphold --- the 13 1959 or 15 years after maturity of the loan, defendant instituted
very duty that justifies the Court’s being. Constitutional respect extra-judicial foreclosure proceedings for the recovery of the balance
and a becoming regard for the sovereign acts of a co-equal branch of the loan remaining unpaid. Plaintiff countered with his suit against
prevent this Court from prying into the internal workings of the both alleging that the mortgage sought to be foreclosed had long
Senate. To repeat, this Court will be neither a tyrant nor a wimp; prescribed, fifteen years having elapsed from the date of maturity.
rather, it will remain steadfast and judicious in upholding the rule PNB on the other hand claims that the defense of prescription would
and the majesty of the law.” not be available if the period from March 10, 1945, when Executive
Order No. 32 1 was issued, to July 26, 1948, when the subsequent
legislative act 2 extending the period of moratorium was declared
6. Presumption of Constitutionality invalid, were to be deducted from the computation of the time
during which the bank took no legal steps for the recovery of the
Perez v. People, 544 SCRA 532 (2008) loan. The lower court did not find such contention persuasive and
decided the suit in favor of plaintiff.

ISSUE: W/N the period of the effectivity of EO 32 and the Act


extending the Moratorium Law before the same were declared
7. Effects of Declaration of Unconstitutionality
invalid tolled the period of prescription (Effect of the declaration of
a. Orthodox view
Unconstitutionality of a law)
Art. 7, Civil Code of the Philippines

Orthodox view: An unconstitutional act is not a law; it confers no HELD: YES. In the language of an American Supreme Court
rights; it decision: “The actual existence of a statute, prior to such a
determination [of unconstitutionality], is an operative fact and may

Constitutional Law 1 based on the syllabus of Atty. Remoroza 2018-2019 44


Merzy’s Notes Prelim Exam

have consequences which cannot justly be ignored. The past cannot NGO. Several petitions were lodged before the Court similarly
always be erased by a new judicial declaration. The effect of the seeking that the "Pork Barrel System" be declared unconstitutional
subsequent ruling as to invalidity may have to be considered in UDK-14951 – A Petition filed seeking that the PDAF be declared
various aspects, with respect to particular relations, individual and unconstitutional, and a cease and desist order be issued restraining
corporate, and particular conduct, private and official.” 4 President Benigno Simeon S. Aquino III (President Aquino) and
Secretary Abad from releasing such funds to Members of Congress
The now prevailing principle is that the existence of a statute or
executive order prior to its being adjudged void is an operative fact
to which legal consequences are attached. Precisely because of the HELD:
judicial recognition that moratorium was a valid governmental Yes, the PDAF article is unconstitutional. The post-enactment
response to the plight of the debtors who were war sufferers, this measures which govern the areas of project identification, fund
Court has made clear its view in a series of cases impressive in their release and fund realignment are not related to functions of
number and unanimity that during the eight-year period that congressional oversight and, hence, allow legislators to intervene
Executive Order No. 32 and Republic Act No. 342 were in force, and/or assume duties that properly belong to the sphere of budget
prescription did not run. execution. This violates the principle of separation of powers.
Congress‘role must be confined to mere oversight that must be
confined to: (1) scrutiny and (2) investigation and monitoring of the
implementation of laws. Any action or step beyond that will
Belgica v. Ochoa, 710 SCRA 1 (2013) undermine the separation of powers guaranteed by the constitution.

FACTS: Thus, the court declares the 2013 pdaf article as well as all other
The NBI Investigation was spawned by sworn affidavits of six (6) provisions of law which similarly allow legislators to wield any form
whistle-blowers who declared that JLN Corporation (Janet Lim of post-enactment authority in the implementation or enforcement of
Napoles) had swindled billions of pesos from the public coffers for the budget, unrelated to congressional oversight, as violative of the
"ghost projects" using dummy NGOs. Thus, Criminal complaints were separation of powers principle and thus unconstitutional.
filed before the Office of the Ombudsman, charging five (5)
lawmakers for Plunder, and three (3) other lawmakers for
Malversation, Direct Bribery, and Violation of the Anti-Graft and Araullo v. Aquino III, G.R. No. 209287, July 1,
Corrupt Practices Act. Also recommended to be charged in the 2014
complaints are some of the lawmakers’ chiefs -of-staff or
representatives, the heads and other officials of three (3)
When President Benigno Aquino III took office, his administration
implementing agencies, and the several presidents of the NGOs set
noticed the sluggish growth of the economy. The World Bank
up by Napoles.
advised that the economy needed a stimulus plan. Budget Secretary
Whistle-blowers alleged that" at least P900 Million from royalties in
Florencio “Butch” Abad then came up with a program called the
the operation of the Malampaya gas project off Palawan province
Disbursement Acceleration Program (DAP).
intended for agrarian reform beneficiaries has gone into a dummy

Constitutional Law 1 based on the syllabus of Atty. Remoroza 2018-2019 45


Merzy’s Notes Prelim Exam

The DAP was seen as a remedy to speed up the funding of applicable to the authors, implementers, and proponents of the DAP
government projects. DAP enables the Executive to realign funds if it is so found in the appropriate tribunals (civil, criminal, or
from slow moving projects to priority projects instead of waiting for administrative) that they have not acted in good faith.
next year’s appropriation. So what happens under the DAP was that
if a certain government project is being undertaken slowly by a
certain executive agency, the funds allotted therefor will be
withdrawn by the Executive. Once withdrawn, these funds are Cocofed v. Republic, 663 SCRA 514 (2012)
declared as “savings” by the Executive and said funds will then be
reallotted to other priority projects. The DAP program did work to
stimulate the economy as economic growth was in fact reported and Facts:
portion of such growth was attributed to the DAP (as noted by the In 1971, RA 6260 created the Coconut Investment Company (CIC) to
Supreme Court). administer the Coconut Investment Fund, a fund to be sourced from
levy on the sale of copra. The copra seller was, or ought to be,
Issue: Whether or not the Doctrine of Operative Fact is applicable. issued COCOFUND receipts. The fund was placed at the disposition
of COCOFED, the national association of coconut producers having
HELD: the largest membership.
When martial law started in 1972, several presidential decrees were
I. , the DAP did not violate Section 29(1), Art. VI of the Constitution. issued to improve the coconut industry through the collection and
DAP was merely a program by the Executive and is not a fund nor is use of the coconut levy fund. Then came P.D. No. 755 in July 1975,
it an appropriation. It is a program for prioritizing government providing under its Section 1 the policy to provide readily available
spending. As such, it did not violate the Constitutional provision cited credit facilities to the coconut farmers at preferential rates. Towards
in Section 29(1), Art. VI of the Constitution. In DAP no additional
achieving this, Section 2 of PD 755 authorized PCA to utilize the
funds were withdrawn from the Treasury otherwise, an appropriation
made by law would have been required. Funds, which were already CCSF and the CIDF collections to acquire a commercial bank and
appropriated for by the GAA, were merely being realigned via the deposit the CCSF levy collections in said bank, interest free, the
DAP. deposit withdrawable only when the bank has attained a certain level
of sufficiency in its equity capital. It also decreed that all levies PCA
Yes. The Doctrine of Operative Fact, which recognizes the legal is authorized to collect shall not be considered as special and/or
effects of an act prior to it being declared as unconstitutional by the fiduciary funds or form part of the general funds of the government.
Supreme Court, is applicable. The DAP has definitely helped
Both P.D. Nos. 961 and 1468 also provide that the CCSF shall not be
stimulate the economy. It has funded numerous projects. If the
construed by any law as a special and/or trust fund, the stated
Executive is ordered to reverse all actions under the DAP, then it
may cause more harm than good. The DAP effects can no longer be intention being that actual ownership of the said fund shall pertain to
undone. The beneficiaries of the DAP cannot be asked to return coconut farmers in their private capacities.
what they received especially so that they relied on the validity of
the DAP. However, the Doctrine of Operative Fact may not be Held:
Constitutional Law 1 based on the syllabus of Atty. Remoroza 2018-2019 46
Merzy’s Notes Prelim Exam

In rendering the assailed PSJs and thereafter refusing to proceed to 14, 1997, she only earned a total of NT$9,000.15 According to her,
trial on the merits, on the mere say-so of the respondent Republic, Wacoal deducted NT$3,000 to cover her plane ticket to Manila.
the Sandiganbayan committed gross and irreversible error, gravely
abused its judicial discretion and flagrantly exceeded its jurisdiction ISSUE: Whether or not Cabiles was entitled to the unexpired portion
of her salary due to illegal dismissal.
as it effectively sanctioned the taking of COCOFED, et al.’s property
by the respondent Republic without due process of law and through
HELD:
retroactive application of the declaration of unconstitutionality of the
coconut levy laws, an act that is not only illegal and violative of the
YES. The Court held that the award of the three-month equivalent
settled Operative Fact Doctrine but, more importantly, inequitable to of respondent’s salary should be increased to the amount equivalent
the coconut farmers whose only possible mistake, offense or to the unexpired term of the employment contract
misfortune was to follow the law.
A statute or provision which was declared unconstitutional is not a
law. It “confers no rights; it imposes no duties; it affords no
Sameer Overseas v. Cabilles, G.R. No.170139, protection; it creates no office; it is inoperative as if it has not been
August 5, 2014 passed at all.”

The Court said that they are aware that the clause “or for three (3)
months for every year of the unexpired term, whichever is less” was
FACTS:
reinstated in Republic Act No. 8042 upon promulgation of Republic
Act No. 10022 in 2010. Thus, when a law or a provision of law is null
Petitioner, Sameer Overseas Placement Agency, Inc., is a because it is inconsistent with the Constitution, the nullity cannot be
recruitment and placement agency. cured by reincorporation or reenactment of the same or a similar law
or provision. A law or provision of law that was already declared
Respondent Joy Cabiles was hired thus signed a one-year unconstitutional remains as such unless circumstances have so
employment contract for a monthly salary of NT$15,360.00. Joy was changed as to warrant a reverse conclusion.
deployed to work for Taiwan Wacoal, Co. Ltd. (Wacoal) on June 26,
1997. She alleged that in her employment contract, she agreed to
work as quality control for one year. In Taiwan, she was asked to
work as a cutter. 8. Partial unconstitutionality

Sameer claims that on July 14, 1997, a certain Mr. Huwang from Partial Unconstitutionality. Requisites:
Wacoal informed Joy, without prior notice, that she was terminated a) The Legislature must be willing to retain the valid portion(s),
and that “she should immediately report to their office to get her usually
salary and passport.” She was asked to “prepare for immediate shown by the presence of a separability clause in the law; and
repatriation.” Joy claims that she was told that from June 26 to July
Constitutional Law 1 based on the syllabus of Atty. Remoroza 2018-2019 47
Merzy’s Notes Prelim Exam

b) The valid portion can stand independently as law. See: In Re: indisputably a judicial function and responsibility. We have said that
in the judicial system from which ours has been derived, the
In Re: Cunanan, 94 Phil. 534 admission, suspension, disbarment or reinstatement of attorneys at
law in the practice of the profession is concededly judicial.

Facts: Congress passed Republic Act Number 972, commonly known


as the “Bar Flunkers’ Act of 1953.” In accordance with the said law, Salazar v. Achacoso, 183 SCRA 145
the Supreme Court then passed and admitted to the bar those
candidates who had obtained an average of 72 per cent by raising it Facts: Rosalie Tesoro of Pasay City in a sworn statement filed with
to 75 percent. the POEA, charged petitioner with illegal recruitment. Public
respondent Atty. Ferdinand Marquez sent petitioner a telegram
After its approval, many of the unsuccessful postwar candidates filed directing him to appear to the POEA regarding the complaint against
petitions for admission to the bar invoking its provisions, while other him. On the same day, after knowing that petitioner had no license
motions for the revision of their examination papers were still to operate a recruitment agency, public respondent Administrator
pending also invoked the aforesaid law as an additional ground for Tomas Achacoso issued a Closure and Seizure Order No. 1205 to
admission. There are also others who have sought simply the petitioner.
reconsideration of their grades without, however, invoking the law in
Issue: Whether or Not the Philippine Overseas Employment
question. To avoid injustice to individual petitioners, the court first
Administration (or the Secretary of Labor) can validly issue warrants
reviewed the motions for reconsideration, irrespective of whether or
of search and seizure (or arrest) under Article 38 of the Labor Code
not they had invoked Republic Act No. 972.

Held: Under the new Constitution, “. . . no search warrant or


Issue: Whether or Not RA No. 972 is constitutional and valid.
warrant of arrest shall issue except upon probable cause to be
determined personally by the judge after examination under oath or
affirmation of the complainant and the witnesses he may produce,
Held: RA No. 972 has for its object, according to its author, to admit
and particularly describing the place to be searched and the persons
to the Bar, those candidates who suffered from insufficiency of
or things to be seized”. Mayors and prosecuting officers cannot issue
reading materials and inadequate preparation.
warrants of seizure or arrest. The Closure and Seizure Order was
based on Article 38 of the Labor Code. The Supreme Court held, “We
In the judicial system from which ours has been evolved, the
reiterate that the Secretary of Labor, not being a judge, may no
admission, suspension, disbarment and reinstatement of attorneys at
law in the practice of the profession and their supervision have been

Constitutional Law 1 based on the syllabus of Atty. Remoroza 2018-2019 48


Merzy’s Notes Prelim Exam

longer issue search or arrest warrants. Hence, the authorities must government of that foreign power which, although not an
go through the judicial process. international person in the sense of international law, does not
impose transfer or death upon intangible person properties of our
citizens not residing therein, or whose law allows a similar exemption
from such taxes. It is, therefore, not necessary that Tangier should
I. THE PHILIPPINES AS A STATE have been recognized by our Government order to entitle the
petitioner to the exemption benefits of the proviso of Section 122 of
A. Definition of a State our Tax. Code."

A community of persons, more or less numerous, ISSUE: Whether the exemption is valid.
permanently occupying a definite portion of territory, independent of
external control, and possessing a government to which a great RULING:
body of inhabitants render habitual obedience. YES. The controlling legal provision as noted is a proviso in Section
122 of the National Internal Revenue Code. It reads thus: "That no
Collector v. Campos Rueda, 42 SCRA 23 (1971)
tax shall be collected under this Title in respect of intangible
Facts: personal property (a) if the decedent at the time of his death was a
resident of a foreign country which at the time of his death did not
Antonio Campos Rueda is the administrator of the estate of the impose a transfer tax or death tax of any character in respect of
deceased Maria Cerdeira. Cerdeira is a Spanish national, by reason intangible person property of the Philippines not residing in that
of her marriage to a Spanish citizen and was a resident of Tangier, foreign country, or (b) if the laws of the foreign country of which the
Morocco up to her death. At the time of her demise she left, among decedent was a resident at the time of his death allow a similar
others, intangible personal properties in the Philippines. The CIR exemption from transfer taxes or death taxes of every character in
then issued an assessment for state and inheritance taxes of respect of intangible personal property owned by citizens of the
P369,383.96. Rueda filed an amended return stating that intangible Philippines not residing in that foreign country."
personal properties worth P396,308.90 should be exempted from
taxes. The CIR denied the request on the ground that the law of It does not admit of doubt that if a foreign country is to be identified
Tangier is not reciprocal to Section 122 (now Section 104) of the with a state, it is required in line with Pound's formulation that it be
National Internal Revenue Code. a politically organized sovereign community independent of outside
control bound by penalties of nationhood, legally supreme within its
The case was elevated to the CTA which sided with Rueda. The CTA territory, acting through a government functioning under a regime
stated that the foreign country mentioned in Section 122 "refers to a of law. A foreign country is thus a sovereign person with the people
composing it viewed as an organized corporate society under a
Constitutional Law 1 based on the syllabus of Atty. Remoroza 2018-2019 49
Merzy’s Notes Prelim Exam

government with the legal competence to exact obedience to its i. Other territories over which the Philippines exercises
commands. jurisdiction
j. Archipelagic Doctrine – Sec. 1, Article II
Distinguished from Nation. State is a legal or juristic concept, while iii. Straight baseline method
nation is an ethnic or racial concept. iv. UN Convention on the Law of the Sea

Distinguished from Government. Government is merely an a) The National Territory: “The national territory comprises the
instrumentality of the State through which the will of the State is Philippine archipelago, with all the islands and waters embraced
implemented and realized. therein, and all other territories over which the Philippines has
sovereignty or jurisdiction, consisting of its terrestrial, fluvial and
B. Elements of a State aerial domains, including its territorial sea,
the seabed, the subsoil, the insular shelves, and other submarine
1. People areas” [Sec. 1, Art. !].

a) Different meanings as used in the Constitution: (i) Inhabitants b) Components: Terrestrial, Fluvial, Maritime and Aerial domains.
[Sec.
2, Art. Ill; Sec. 1, Art. XIII]; (ii) Citizens [Preamble; Secs. 1 & 4, Art. c) The Philippine Archipelago: (i) Treaty of Paris, December 10, 1898
II; Sec. 7, Art. (Cession of the Philippine Islands by Spain to the United States); (ii)
Ill]; (iii) Electors [Sec. 4, Art. VII]. Treatybetween Spain and US at Washington, November 7, 1900
(Cagayan, Sulu &
b) As requisite for Statehood: Adequate number for self-sufficiency Sibuto); (iii) Treaty between US and Great Britain, January 2, 1930
and defense; of both sexes for perpetuity. (Turtle &
Mangsee Islands).
1. Territory - Art. I; R.A. 3046; R.A. 5446
d) Other territories over which the Philippines exercises jurisdiction.
a. The National Territory
b. Components
(i)Batanes [1935 Constitution]; (ii) Those contemplated in Art. I,
c. The Philippine Archipelago
1973 Constitution
d. Other territories over which the Philippines exercises
[belonging to the Philippines by historic right or legal title]; (iii) PD
jurisdiction
1596, June 11,
e. Archipelagic Doctrine – Sec. 1, Article II
1978.
i. Straight baseline method
ii. UN Convention on the Law of the Sea
e) Archipelago Doctrine: “The waters around, between and
connecting
f. The National Territory
g. Components
h. The Philippine Archipelago
Constitutional Law 1 based on the syllabus of Atty. Remoroza 2018-2019 50
Merzy’s Notes Prelim Exam

the islands of the archipelago, regardless of their breadth and entity through which the functions of government are exercised
dimensions, form part of the internal waters of the Philippines” [2nd throughout the Philippines, including, save as the contrary appears
sentence, Sec. 1, Art II from
the context, the various arms through which political authority is
i) This articulates the archipelagic doctrine of national territory, made
based on the principle that an archipelago, which consists of a effective in the Philippines, whether pertaining to the autonomous
number of islands separated by bodies of water, should be treated regions, the provincial, city, municipal or barangay subdivisions or
as one integral unit. other
forms of local government" [Sec. 2 (1), Administrative Code of
ii) Straight baseline method: Imaginary straight lines are drawn 1987].
joining the outermost points of outermost islands of the archipelago,
enclosing an area the ratio of which should not be more than 9:1 b. Functions
(water to land); provided that the drawing of baselines shall not
depart, to any appreciable extent, from the general configuration of Traditionally, the functions of government have been classified
the archipelago. The waters within the baselines shall be into constituent, which are mandatory for the Government to
considered internal waters; while the breadth of the territorial sea perform because they
shall then be measured from the baselines. constitute the very bonds of society, such as the maintenance of
peace and order,
iii) UN Convention on the Law of the Sea [April 30,1982; ratified by regulation of property and property rights, the administration of
the Philippines in August, 1983] provides (i) Contiguous Zone of 12 justice, etc; and ministrant, those intended to promote the welfare,
miles; (ii) Exclusive Economic Zone of 200 miles. Although the progress and prosperity of the people, and which are merely optional
contiguous zone and most of for Government to perform.
the exclusive economic zone may not, technically, be part of the
territory of the Romualdez-Yap v. Civil Service Commission,
.State, nonetheless, the coastal State enjoys preferential rights over 225 SCRA 285
the marine
resources found within these zones. See also P.D. 1599, June 11,
1978 FACTS:

2. Government Petitioner Conchita Romualdez-Yap started working with the


Philippine National Bank (PNB) on September 20, 1972. After several
a. Definition promotions, she was appointed in 1983 as a Senior Vice President
assigned to the Fund Transfer Department. The case at bar is a
The agency or instrumentality through which the will of the State is special civil action for certiorari assailing Res. No. 92-201 of the
formulated, expressed and realized. respondent which upheld the petitioner’s separation from PNB in
Government of the Philippines is “the corporate governmental light of EO 80 or the Revised Charter of PNB. Petitioner contends
that there is an existence of bad faith in its reorganization and that
Constitutional Law 1 based on the syllabus of Atty. Remoroza 2018-2019 51
Merzy’s Notes Prelim Exam

there is an erroneous application of the one year prescriptive period But a reorganization whether in a government bureau performing
for quo warranto proceedings in her case. constituent functions or in agovernment-owned or controlled
corporation performing ministrant functions must meet a
ISSUE: commontest, the test of good faith.

Is the reorganization of PNB, a government-owned or controlled The decision to abolish the Fund Transfer Department (FTD) was a
corporation performing ministrant functions, valid? business judgment made ingood faith. Due to the restructuring, PNB
became once more a viable banking institution. After fouryears, FTD
HELD: was restored and its functions were transferred to the International
Department. Thesecan be attributed to the bank's growth after
Ministrant functions are those undertaken by way of advancing the reorganizations, thereby negating bad faith in that organization.
general interests of society and are merely optional. Commercial or
universal banking is, ideally, not a governmental but a private sector
endeavor, an optional function of the government. There are
functions of the government which it may exercise to promote Shipside, Inc. v. Court of Appeals, G.R. No.
merely the welfare, progress, and prosperity of the people. Thus, 143377, February 20,2001
reorganization of such corporations like PNB is valid so long as they
are done in good faith as prescribed in the Dario v. Mison doctrine.
On April 11, 1960, Lots No. 1 and 4 were conveyed by Rafael Galvez
Accordingly, the reorganization of PNB is found to be done in good
in favor of Filipina Mamaril, Cleopatra Llana, Regina Bustos, and
faith by the Court.
Erlinda Balatbat in a deed of sale which was inscribed as Entry No.
9115 OCT No.0-381 on August 10, 1960. Consequently, Transfer
ISSUE: Certificate No. T-4304 was issued in favor of the buyers covering
Whether there was an existence of bad faith in the reorganization of Lots No. 1 and 4.
the Philippine NationalBank resulting in the separation from the
service of petitioner?
n August 16, 1960, Mamaril, et al. sold Lots No. 1 and 4 to Lepanto
Consolidated Mining Company. The deed of sale covering the
HELD:
aforesaid property was inscribed as Entry No. 9173 on TCT No. T-
The Court made a distinction in this ruling on the validity of
4304. Subsequently, Transfer Certificate No. T-4314 was issued in
reorganization between agovernment bureau or office performing
the name of Lepanto Consolidated Mining Company as owner of Lots
constituent functions (like the Customs) and a government-owned or
No. 1 and 4.
controlled corporation performing ministrant functions (like the PNB).
Constituent
Held:

it was held that the Bases Conversion Development Authority


(BCDA),

Constitutional Law 1 based on the syllabus of Atty. Remoroza 2018-2019 52


Merzy’s Notes Prelim Exam

created under R.A. 7227, performs functions which are basically government of paramount force.
proprietary in nature. The promotion of economic and social
development of Central Luzon, in particular, and the country’s goal ii) Presidential vs. parliamentary government. The principal
for enhancement, in general, do not make BCDA distinction is that in a presidential government, there is separation of
equivalent to Government. Other corporations, such as SSS, GSIS, executive and legislative powers (the first is lodged in the President,
NIA, although performing functions aimed at promoting public while the second is vested in Congress); while in a parliamentary
interest and public welfare, are not invested with government government, there is fusion of both executive and legislative powers
attributes. [Thus, with the transfer to BCDA of Camp Wallace, the in Parliament, although the actual exercise of the executive powers
government no longer had a right or interest to protect; the real is vested in a Prime Minister who is chosen by, and accountable
party in interest to recover the property is, thus, the BCDA, not the to, Parliament.
Republic of the
Philippines. iii) Unitary vs. federal government. A unitary government is a
single, centralized government, exercising powers over both the
internal and
c. Doctrine of Parens Patriae external affairs of the State; while a federal government consists of
autonomous state (local) government units merged into a single
Literally, parent of the people. As such, the Government may act as State, with the national government exercising a limited degree of
guardian of the rights of people who may be disadvantaged or power over the domestic affairs but generally full direction of the
suffering from some disability or misfortune external affairs of the State.

d. Classification 3. Sovereignty

i. De jure vs de facto a. Definition


ii. Presidential vs parliamentary The supreme and uncontrollable power inherent in a
iii. Unitary vs federal government State by which that State is governed

De jure vs. De facto. See: Co Kim Chan v. Tan Keh, 75 Phil. b. Kinds
113; Lawyers League for a Better Philippines v. Aquino, supra
i) Legal, which is the power to issue final commands; or Political,
ia) Kinds of de facto government: That which takes which is the sum total of all the influences which lie behind the law.
possession or control of, or usurps, by force or by the voice of the
majority, the rightful legal government and maintains itself against ii) Internal, or the supreme power over everything within its
the will of the latter; that which is established by the inhabitants of a territory; or External, also known as independence, which is freedom
territory who rise in insurrection against the parent state; and that from external control.
which is established by the invading forces of an enemy who occupy
a territory in the course of war. The last is denominated a de facto c. Characteristics

Constitutional Law 1 based on the syllabus of Atty. Remoroza 2018-2019 53


Merzy’s Notes Prelim Exam

representatives, and consuls to a certain degree;


permanence, exclusiveness, comprehensiveness, (b) Foreign state property, including embassies, consulates, and
absoluteness, indivisibility, inalienability, imprescriptibility public vessels engaged in non-commercial activities;
(c) Acts of state;
d. Effects of change in sovereignty (d) Foreign merchant vessels exercising the rights of innocent
passage or involuntary entry, such as arrival under stress;
Political laws are abrogated People v. Perfecto, 43 Phil. 887; (e) Foreign armies passing through or stationed in its territory with
Macariola v. Asuncion, 114 SCRA 77]; municipal its permission; and
laws remain in force [Vilas v. City of Manila, 229 US 345]. (f) Such other persons or property, including organizations like the
United Nations, over which it may, by agreement, waive jurisdiction
e. Effects of belligerent occupation
ii) Personal: power of the State over its nationals, which may be
No change in sovereignty. See: Peralta v. Director of Prisons, 75 Phil. exercised by the State even if the individual is outside the territory of
285; Alcantara v. Director of Prisons, 75 the State.
Phil. 749;Ruffyv. Chief of Staff, 75 Phil. 875.
i) However, political laws, except the law on treason, are iii) Extraterritorial: power exercised by the State beyond its territory
suspended [Laurel v. Misa, 77 Phil. 856]; municipal laws remain in in the following cases: (a) Assertion of its personal jurisdiction over
force unless repealed by the belligerent occupant. At the end of the its nationals abroad; or the exercise of its rights to punish certain
belligerent occupation, when the occupant is ousted from the offenses committed outside
territory, the political laws which had been suspended during the its territory against its national interests even if the offenders are
occupation shall automatically become effective again, under the non-resident aliens;
doctrine of jus postliminium (b) By virtue of its relations with other states or territories, as when
it establishes a colonial protectorate, or a condominium, or
f. Dominium vs imperium administers a trust territory, or occupies enemy territory in the
course of war;
Dominium refers to the capacity to own (c) When the local state waives its jurisdiction over persons and
or acquire property, including lands held by the State in its things within its territory, as when a foreign army stationed therein
proprietary capacity; while Imperium is the authority possessed by remains under the jurisdiction of the sending state;
the State embraced in the concept of sovereignty. (d) By the principle of exterritoriality, as illustrated by the
immunities of the head of state in a foreign country;
(e) Through enjoyment of easements or servitudes, such as
g. Jurisdiction the easement of innocent passage or arrival under stress;
(f) The exercise of jurisdiction by the state in the high seas over its
i) Territorial: power of the State over persons and things within vessels; over pirates; in the exercise of the right to visit and search;
its territory. Exempt are: and under the doctrine of hot pursuit;
(a) Foreign states, heads of state, diplomatic

Constitutional Law 1 based on the syllabus of Atty. Remoroza 2018-2019 54


Merzy’s Notes Prelim Exam

(g) The exercise of limited jurisdiction over the contiguous zone and
the patrimonial Minucher v. Court of Appeals, G.R. No.
sea, to prevent infringement of its customs, fiscal, immigration or 142396, February 11, 2003
sanitary regulations..
FACTS
C. State Immunity from Suit
Sometime in May 1986, an information for violation of the
1. Basis Dangerous Drugs Act was filed against petitioner Khosrow Minucher
with the RTC. The criminal charge followed a "buy-bust operation"
There can be no legal right against the authority which makes the concluded by the Philippine police narcotic agent in the house if
law on which the right depends [Republic v. Villasor, 54 SCRA 83], Minucher where a quantity of heroin, a prohibited drug, was said to
However, it may be sued if it gives consent, whether express or have been seized. The narcotic agents were accompanied by private
implied. The doctrine is also known as the Royal Prerogative of respondent Arthur Scalzo who would, in due time, become one of
Dishonesty. the principal witnesses for the prosecution. On January 1988,
Presiding Judge Migrino rendered a decision acquitting the accused.
2. Par in parem non habet imperium Minucher filed Civil Case before the RTC for damages on account of
what he claimed to have been trumped-up charges of drug
Immunity is enjoyed by other States, consonant with the public trafficking made by Arthur Scalzo.
international law principle of par in parem non habet imperium. The
Head of State, who is deemed the personification of the State, is
inviolable, and thus, enjoys immunity from suit. ISSUES

a) The State’s diplomatic agents, including consuls to a 1. Whether or not Arthur Scalzo is entitled to diplomatic immunity
certain extent, are also exempt from the jurisdiction of
local courts and admiinistraive tribunals. [See PUBLIC 2. Whether the Doctrine of State Immunity from suit is applicable
INTERNATIONAL LAW, infra herein

a. State’s diplomatic agents


RULING
A foreign agent, operating within a territory, can be cloaked with
immunity from suit but only as long as it can be established that he 1. Scalzo contends that the Vienna Convention on Diplomatic
is acting within the directives of the sending State. The cloak of Relations, to which the Philippines is a signatory, grants him
protection is removed the moment the foreign agent is sued in his absolute immunity from suit being an agent of the US Drugs
individual capacity, as when he is sought to be made liable for Enforcement Agency. However, the main yardstick in ascertaining
whatever damage he may have caused by his act done with malice whether a person is a diplomat entitled to immunity is the
or in bad faith or beyond the scope of his authority or jurisdiction determination of whether or not he performs duties of diplomatic

Constitutional Law 1 based on the syllabus of Atty. Remoroza 2018-2019 55


Merzy’s Notes Prelim Exam

nature. The Vienna Convention lists the classes of heads of are likewise beyond the jurisdiction of local courts [Convention on
diplomatic missions to include (a) ambassadors or nuncios accredited Privileges and Immunities of the United Nations; Convention on
to the heads of state, (b) envoys, ministers or inter nuncios Privileges and Immunities of Specialized Agencies of the United
accredited to the head of states, and (c) charges d' affairs accredited Nations; World Health Organization v. Aquino, supra.
to the ministers of foreign affairs. The Convention defines
"diplomatic agents" as the heads of missions or members of the c. Other international organizations/agencies
diplomatic staff, thus impliedly withholding the same privileges from
all others. Scalzo asserted that he was an Assistant Attache of the
US diplomatic mission. Attaches assist a chief of mission in his duties SEAFDEC-Aquaculture v. NLRC, 206 SCRA 283
and are administratively under him. These officials are not generally
regarded as members of the diplomatic mission, nor they normally
designated as having diplomatic rank. FACTS: SEAFDEC-AQD is a department of an international
organization, the Southeast Asian Fisheries Development Center,
2. While the diplomatic immunity of Scalzo might thus remain organized through an agreement in 1967 by the governments of
contentions, it was sufficiently established that, indeed, he worked Malaysia, Singapore, Thailand, Vietnam, Indonesia and the
for the USDEA. If it should be ascertained that Scalzo was acting Philippines with Japan as the sponsoring country.
well within his assigned functions when he committed the acts
allegedly complained of, the present controversy could then be Juvenal Lazaga was employed as a Research Associate on a
resolved under the related doctrine of State Immunity from Suit. probationary basis by SEAFDEC-AQD. Lacanilao in his capacity as
While the doctrine appears to prohibit only suits against against the Chief of SEAFDEC-AQD sent a notice of termination
State without its consent, it is also applicable to complaints filed to Lazaga informing him that due to the financial constraints being
against officials of the State for acts allegedly performed by them in experienced by the department, his services shall be terminated.
the discharge of their duties. The official exchanges of SEAFDEC-AQD's failure to pay Lazaga his separation pay forced him
communication, certifications from officials, as well as participation to file a case with the NLRC. The Labor Arbiter and NLRC ruled in
of members of the Philippine Narcotics Command may be inadequate favor of Lazaga. Thus SEAFDEC-AQD appealed, claiming that the
to support to support the diplomatic status of Scalzo but they give NLRC has no jurisdiction over the case since it is immune from suit
enough indication that the Philippine government has given its owing to its international character and the complaint is in effect a
imprimatur to the activities of Scalzo. It can hardly be said that he suit against the State which cannot be maintained without its
acted beyond the scope of his official function or duties. All told, consent.
Scalzo is entitled to the defense os state immunity from suit.
ISSUES:

1. Does the NLRC have jurisdiction over SEAFDEC-AQD?


b. The United Nations
2. Is SEAFDEC-AQD estopped for its failure to raise the issue of
The United Nations, as well as its organs and specialized agencies, jurisdiction at the first instance?

Constitutional Law 1 based on the syllabus of Atty. Remoroza 2018-2019 56


Merzy’s Notes Prelim Exam

HELD: 16, 1968. The purpose of the Center is to contribute to the


promotion of fisheries development in Southeast Asia by mutual
1. SEAFDEC-AQD is an international agency beyond the jurisdiction cooperation among the member governments of the Center.
The invocation by private respondents of the doctrine of estoppel is
of public respondent NLRC. Being an intergovernmental organization,
unavailing, because estoppel does not confer jurisdiction
SEAFDEC including its Departments (AQD), enjoys functional
independence and freedom from control of the state in whose
territory its office is located. Callado v. IRRI, 244 SCRA 210

Permanent international commissions and administrative bodies have Facts: Ernesto Callado, petitioner, was employed as a driver at the
been created by the agreement of a considerable number of States IRRI. One day while driving an IRRI vehicle on an official trip to the
for a variety of international purposes, economic or social and mainly NAIA and back to the IRRI, petitioner figured in an accident.
non-political. In so far as they are autonomous and beyond the
Petitioner was informed of the findings of a preliminary investigation
control of any one State, they have a distinct juridical personality conducted by the IRRI's Human Resource Development Department
independent of the municipal law of the State where they are Manager. In view of the findings, he was charged with:
situated. As such, according to one leading authority "they must be (1) Driving an institute vehicle while on official duty under the
deemed to possess a species of international personality of their influence of liquor; (2) Serious misconduct consisting of failure to
own." report to supervisors the failure of the vehicle to start because of a
problem with the car battery, and(3) Gross and habitual neglect of
duties
One of the basic immunities of an international organization is
immunity from local jurisdiction, i.e., that it is immune from the legal Issue: Did the (IRRI) waive its immunity from suit in this dispute
writs and processes issued by the tribunals of the country where it is which arose from an employer-employee relationship?
found. The obvious reason for this is that the subjection of such an
organization to the authority of the local courts would afford a Held: No.
convenient medium thru which the host government may interfere in
P.D. No. 1620, Article 3 provides:
there operations or even influence or control its policies and
Art. 3. Immunity from Legal Process. The Institute shall enjoy
decisions of the organization; besides, such subjection to local immunity from any penal, civil and administrative proceedings,
jurisdiction would impair the capacity of such body to discharge its except insofar as that immunity has been expressly waived by the
responsibilities impartially on behalf of its member-states Director-General of the Institute or his authorized representatives.

it was held that SEAFDEC, as an international agency, enjoys The SC upholds the constitutionality of the aforequoted law. There is
diplomatic immunity. It was established through an international in this case "a categorical recognition by the Executive Branch of the
agreement to which the Philippines became a signatory on January Government that IRRI enjoys immunities accorded to international

Constitutional Law 1 based on the syllabus of Atty. Remoroza 2018-2019 57


Merzy’s Notes Prelim Exam

organizations, which determination has been held to be a political The Bureau of Forestry issued Notice 2087 advertising for public
question conclusive upon the Courts in order not to embarass a hearing a certain tract of public forest land. Petitioner submitted his
political department of Government. application in due form after paying the necessary fees and posting
It is a recognized principle of international law and under our system the required bond. Later that year, President Carlos Garcia issued a
of separation of powers that diplomatic immunity is essentially a directive to the Director of the Bureau of Forestry to convert the land
political question and courts should refuse to look beyond a into a forest reserve for watershed purposes. The Agriculture and
determination by the executive branch of the government, and Natural Resources issued General Memorandum No. 46 granting
where the plea of diplomatic immunity is recognized and affirmed by ordinary timber license where the area is not more than 3,000
the executive branch of the government as in the case at bar, it is hectares. This was subsequently revoked by General Memorandum
then the duty of the courts to accept the claim of immunity upon No. 60.
appropriate suggestion by the principal law officer of the
government or other officer acting under his direction. ISSUE(S):
Whether or not the doctrine of State immunity applies in this case.
The raison d'etre for these immunities is the assurance of
unimpeded performance of their functions by the agencies HELD:
concerned. YES. Petitioners not only failed to exhaust his administrative
remedies, but also failed to note that his action is a suit against the
The grant of immunity to IRRI is clear and unequivocal and an State which cannot prosper unless the State give its consent.
express waiver by its Director-General is the only way by which it
may relinquish or abandon this immunity.
the Supreme Court said that State immunity from suit may be
invoked as long as the suit really affects the property, rights or
In cases involving dismissed employees, the Institute may waive its
interests of the State and not merely those of the officers nominally
immunity, signifying that such waiver is discretionary on its part
made party defendants. In this case, the Court said that the
promotion of public welfare and the protection of the inhabitants
near the public forest areproperty rights and interests of the State.
3. Test to determine if the suit is against the State

On the assumption that decision is rendered against the public


officer or agency impleaded, will the enforcement thereof require an Veterans Manpower and Protective Services, Inc.
affirmative act from the State, such as the appropriation of the v. Court of Appeals, 214 SCRA 286
needed amount to satisfy the judgment? If so, then it is a suit
against the State
FACTS:
Tan v. Director of Forestry, 125 SCRA 302
Veterans Manpower and Protective Services, Inc. (VMPSI) alleges
that the provisions under Section 4 and 17 of Republic Act No. 5487

Constitutional Law 1 based on the syllabus of Atty. Remoroza 2018-2019 58


Merzy’s Notes Prelim Exam

or the Private Security Agency Law violate the 1987 Constitution statutory authority, hence, a legislative act, not from a mere
against monopolies, unfair competition and combinations in restraint memorandum. Without such consent, the trial court did not acquired
of trade, and tend to favor and institutionalize the Philippine jurisdiction over the public respondents. Petition for review is denied
Association of Detective and Protective Agency Operators, Inc. and the judgment appealed from is affirmed in toto.
(PADPAO) which is monopolistic because it has an interest in more
than one security agency.

Respondent VMPSI likewise questions the validity of paragraph 3, 4. Suits against Government Agencies
subparagraph (g) of the Modifying Regulations on the Issuance of
License to Operate and Private Security Licenses and Specifying a. Incorporated
Regulations for the Operation of PADPAO issued by then PC Chief Lt.
Gen. Fidel V. Ramos, through Col. Sabas V. Edades, requiring that If the charter provides that the agency can sue and be
“all private security agencies/company security forces must register sued, then suit will lie, including one for tort. The provision in the
as members of any PADPAO Chapter organized within the Region charter constitutes express consent on the part of the State to be
where their main offices are located...”. As such membership sued.
requirement in PADPAO is compulsory in nature, it allegedly violates
legal and constitutional provisions against monopolies, unfair SSS v. Court of Appeals, 120 SCRA 707
competition and combinations in restraint of trade.

ISSUE: Facts:

Whether or not VMPSI’s complaint against the PC Chief and PC- Spouses David and Socorro Cruz, applied and granted a real estate
SUSIA is a suit against the State without its consent. loan by the SSS withresidential lot located at Pateros, Rizal as
collateral. The spouses Cruz complied with their monthlypayments.
HELD:
When delayed were incurred in their monthly payments SSS filed a
Yes. A public official may sometimes be held liable in his personal or petition for foreclosure of their real estate mortgage executed by the
private capacity if he acts in bad faith, or beyond the scope of his spouses Cruz on the ground that the spouses Cruz defaultedin
authority or jurisdiction, however, since the acts for which the PC payment, Pursuant for these application for foreclosure notices were
Chief and PC-SUSIA are being called to account in this case, were published on the second noticethe counsel for spouses Cruz sent a
performed as part of their official duties, without malice, gross letter to SSS informing the latter that his clients are up to date
negligence, or bad faith, no recovery may be had against them in
intheir payment of the monthly amortization and the SSS should
their private capacities. Furthermore, the Supreme Court agrees with
the Court of Appeals that the Memorandum of Agreement dated May discontinued the publication of thenotices of foreclosure. This
12, 1986 does not constitute an implied consent by the State to be request remain unheaded, this spouses Cruz filed an action for
sued. The consent of the State to be sued must emanate from damagesagainst SSS before RTC in Rizal. SSS invoking its immunity

Constitutional Law 1 based on the syllabus of Atty. Remoroza 2018-2019 59


Merzy’s Notes Prelim Exam

from suit being an agency of the government performing suability of the State. After trial, the court ruled in favor of the
government function plaintiffs and ordered Municipality and Bislig to pay jointly and
severally the heirs of Baniña.
ISSUE: Whether or not SSS is immune from suit.
ISSUES:
HELD:
1. Are municipal corporations suable?
Negative.. The SSS has a distinct legal personality and it can be sued
for damages. The SSS doesnot enjoy immunity from suit by express 2. Is the Municipality liable for the torts committed by its employee
statutory consent.It has corporated power separate and distinct from who was then engaged in the discharge of governmental functions?
the government. SSS own organic act
specifically provides that it can sue and be sued in court. These HELD:
words “sue and be sued” embrace all
civil process incident to a legal action. So that even assuming that 1. Municipal corporations, like provinces and cities, are agencies of
the SSS, as it claims, enjoys immunityfrom suit as an entity the State when they are engaged in governmental functions and
performing governmental function, by virtue of the explicit provision therefore should enjoy the sovereign immunity from suit.
of theaforecited enabling law, the government must be deemed to Nevertheless, they are subject to suit even in the performance of
have waived immunity in respect of theSSS, although it does not such functions because their charter provided that they can sue and
thereby concede its liability that statutory law has given to the be sued.
private citizen aremedy for the enforcement and protection of his
rights. The SSS thereby has been required to submitto the 2. Municipal corporations are suable because their charters grant
jurisdiction of the court; subject to its right to interpose any lawful them the competence to sue and be sued. Nevertheless, they are
defense generally not liable for torts committed by them in the discharge of
governmental functions and can be held answerable only if it can be
Municipality of San Fernando, La Union v.
shown that they were acting in a proprietary capacity. In permitting
Judge Firme, 195 SCRA 692
such entities to be sued, the State merely gives the claimant the
right to show that the defendant was not acting in its governmental
FACTS: A passenger jeepney, a sand truck and a dump truck of the capacity when the injury was committed or that the case comes
Municipality of San Fernando, La Union collided. Due to the impact, under the exceptions recognized by law. Failing this, the claimant
several passengers of the jeepney including Laureano Baniña Sr. cannot recover.
died. The heirs of Baniña filed a complaint for damages against the
owner and driver of the jeepney, who, in turn, filed a Third Party In this case, the driver of the dump truck of the municipality insists
Complaint against the Municipality and its dump truck driver, Alfredo that "he was on his way to the Naguilian river to get a load of sand
Bislig. Municipality filed its answer and raised the defense of non-
Constitutional Law 1 based on the syllabus of Atty. Remoroza 2018-2019 60
Merzy’s Notes Prelim Exam

and gravel for the repair of San Fernando's municipal streets." In the Petitioner Department of Agriculture and Sultan Security Agency
absence of any evidence to the contrary, the regularity of the entered into a contract for security services to be provided by the
performance of official duty is presumed. Hence, the driver of the latter to the said governmental entity. Pursuant to their
arrangements, guards were deployed by Sultan Security Agency in
dump truck was performing duties or tasks pertaining to his office.
the various premises of the DA. Thereafter, several guards filed a
complaint for underpayment of wages, non-payment of 13th month
pay, uniform allowances, night shift differential pay, holiday pay, and
overtime pay, as well as for damages against the DA and the
National Irrigation Administration v. Court of security agency.
Appeals, 214 SCRA 35
The Labor Arbiter rendered a decision finding the DA jointly and
the Supreme Court reiterated that NIAis a corporate body severally liable with the security agency for the payment of money
performing claims of the complainant security guards. The DA and the security
proprietary functions, whose charter, P.D. 552, provides that it may agency did not appeal the decision. Thus, the decision became final
sue and be and executory. The Labor Arbiter issued a writ of execution to
sued. enforce and execute the judgment against the property of the DA
and the security agency. Thereafter, the City Sheriff levied on
execution the motor vehicles of the DA.

Philippine National Railways v. Intermediate The private respondents, on the other hand, argue that the
Appellate Court, 217 SCRA 401 petitioner has impliedly waived its immunity from suit by concluding
a service contract with Sultan Security Agency.

it was held that although the charter of PNR is silent on whether it Issues: Whether or not the doctrine of non-suability of the State
may sue or be sued, it had already been ruled in Malong v. PNR, 185 applies in the case.
SCRA 63, that the PNR “is not performing any governmental
function” and may, therefore, be sued. Discussions:

b. Unincorporated Act No. 3083, aforecited, gives the consent of the State to be “sued
upon any moneyed claim involving liability arising from contract,
Department of Agriculture v. NLRC, 227 SCRA express or implied. However, the money claim should first be
693 brought to the Commission on Audit. Act 3083 stands as the general
law waiving the State’s immunity from suit, subject to its general
Facts: limitation expressed in Section 7 thereof that ‘no execution shall
issue upon any judgment rendered by any Court against the

Constitutional Law 1 based on the syllabus of Atty. Remoroza 2018-2019 61


Merzy’s Notes Prelim Exam

Government of the (Philippines), and that the conditions provided in Sanders v. Veridiano, 162 SCRA 88
Commonwealth Act 327 for filing money claims against the
Government must be strictly observed.
FACTS:
Rulings: Petitioner Dale Sanders was the special services director of the US
Naval Station (NAVSTA) in Olongapo City. Private respondents,
No. The rule does not say that the State may not be sued under any Anthony Rossi and Ralph Wyers, are American citizens permanently
circumstances. The State may at times be sued. The general law residing in the Philippines and were employed as game room
waiving the immunity of the state from suit is found in Act No. 3083, attendants in the special services department of NAVSTA. On
where the Philippine government “consents and submits to be sued October 3, 1975, the respondents were advised that their
upon any money claims involving liability arising from contract, employment had been converted from permanent full-time to
express or implied, which could serve as a basis of civil action permanent part-time. In a letter addressed to petitioner Moreau,
between private parties.” Sanders disagreed with the hearing officer’s report of the
reinstatement of private respondents to permanent part-time plus
In this case, The DA has not pretended to have assumed a capacity back wages. Respondents allege that the letters contained libelous
apart from its being a governmental entity when it entered into the imputations which caused them to be ridiculed and, thus, filed for
questioned contract; nor that it could have, in fact, performed any damages against petitioners.
act proprietary in character. But the claims of the complainant
security guards clearly constitute money claims. ISSUE:
Whether the petitioners were performing their official duties when
they did the acts for which they have been sued for damages.

HELD:
Civil Aeronautics Administration v. Court of It is abundantly clear in the present case that the acts for which the
Appeals, 167 SCRA 28 petitioners are being called to account were performed by them in
the discharge of their official duties. Sanders, as director of the
special services department of NAVSTA, undoubtedly had supervision
over its personnel and had a hand in their employment, work
If proprietary: suit will lie^ because when the State engages in
assignments, discipline, dismissal and other related matters. The
principally proprietary functions, then it descends to the level of a
same can be said for Moreau. Given the official character of the
private individual, and may, therefore, be vulnerable to suit
above-described letters, it can be concluded that the petitioners
were being sued as officers of the United States government. There
5. Suit against Public Officers should be no question by now that such complaint cannot prosper
unless the government sought to be held ultimately liable has given
its consent to be sued. The private respondents must pursue their
claim against the petitioners in accordance with the laws of the
Constitutional Law 1 based on the syllabus of Atty. Remoroza 2018-2019 62
Merzy’s Notes Prelim Exam

Unites States of which they are all citizens and under whose Issue: WON private respondents are immune from suit being officers
jurisdiction the alleged offenses were committed for the Philippine of the US Armed Forces
courts have no jurisdiction over the case
Held:
No they are not immune.
Shauf v. Court of Appeals, 191 SCRA 713 WHEREFORE, the challenged decision and resolution of respondent
Court of Appeals in CA-G.R. CV No. 17932 are hereby ANNULLED
and SET ASIDE. Private respondents are hereby ORDERED, jointly
and severally, to pay petitioners the sum of P100,000.00 as moral
Facts:
damages, P20,000.00 as and for attorney's fees, and the costs of
suit.
 Loida Shauf, a Filipino by origin and married to an American
who is a member of the US Air Force, was rejected for a Ratio:
position of Guidance Counselor in the Base Education Office
at Clark Air Base, for which she is eminently qualified.
 They state that the doctrine of immunity from suit will not
 By reason of her non-selection, she filed a complaint for
apply and may not be invoked where the public official is
damages and an equal employment opportunity complaint
being sued in his private and personal capacity as an
against private respondents, Don Detwiler (civillian
ordinary citizen. The cloak of protection afforded the
personnel officer) and Anthony Persi (Education Director),
officers and agents of the government is removed the
for alleged discrimination by reason of her nationality and
moment they are sued in their individual capacity. This
sex.
situation usually arises where the public official acts without
 Shauf was offered a temporary position as a temporary
authority or in excess of the powers vested in him.
Assistant Education Adviser for a 180-day period with the
o It is a well-settled principle of law that a public
condition that if a vacancy occurs, she will be automatically
official may be liable in his personal private capacity
selected to fill the vacancy. But if no vacancy occurs after
for whatever damage he may have caused by his act
180 days, she will be released but will be selected to fill a
done with malice and in bad faith, or beyond the
future vacancy if she’s available. Shauf accepted the offer.
scope of his authority or jurisdiction
During that time, Mrs. Mary Abalateo’s was about to vacate
her position. But Mrs. Abalateo’s appointment was extended
thus, Shauf was never appointed to said position. She claims The rationale for this ruling is that the doctrine of state
that the Abalateo’s stay was extended indefinitely to deny immunity cannot be used as an instrument for perpetrating
her the appointment as retaliation for the complaint that she an injustice
filed against Persi. Persi denies this allegation. He claims it
was a joint decision of the management & it was in
accordance of with the applicable regulation.

Constitutional Law 1 based on the syllabus of Atty. Remoroza 2018-2019 63


Merzy’s Notes Prelim Exam

Republic v. Sandiganbayan, G.R. No. 142476, not qualify as suit against the State.While the Republic in
March 20, 2001 this case is sued by name, the ultimate liability does
not pertain to the government.The military officials are held
Thus, the PCGG or any of its members, may be held civilly liable (for liable for the damages for their official functions ceased the
the sale of an aircraft to Fuller Aircraft,which was void) if they did moment they haveexceeded to their authority. They were
not act with good faith and within the scope of their authority in the deployed to ensure that the rally would be peaceful and orderly
performance of official duties andshould guarantee the safety of the people. The court has
made it quite clear that even a “high position in thegovernment
Republic v. Hon. Edilberto Sandoval, 220 SCRA 124 does not confer a license to persecute or recklessly injure
another.” The court rules that there isn o r e v e r s i b l e e r r o r
 The petitioner (CaylaoGroup) filed a suit against the
and no grave abuse of dicretion commited by
State that for them the State has waived its
t h e r e s p o n d e n t J u d g e i n i s s u i n g t h e questioned
immunity when the Mendiola Commission recommended
orderS.
the government to indemnify the victims of the Mendiola
incident and the acts andutterances of President Aquino
which is sympathetic to the cause is indicative of State's
waiver of immunityand therefore, the government
should also be liable and should be compensated by Lansang v. Court of Appeals, G.R. No. 102667,
the government . The case has been dismissed that State February 23, 2000
has not waived its immunity. On the other hand, the Military
Officer filed apetition for certiorari to review the orders of
the Regional Trial Court, Branch 9. Facts: Private respondents General Assembly of the Blind, Inc.
(GABI) and Jose Iglesias were allegedly awarded a verbal contract of
 ISSUE: lease in 1970 to occupy a portion of Rizal Park by the National Parks
 Whether or not the State has waived its immunity from suit
Development Committee (NPDC), a government initiated civic body
and therefore should the State be liablefor the incident?
engaged in the development of national parks. Private respondents
 HELD: were allegedly given office and library space as well as kiosks area
No. The recommendation made by the Mendiola selling food and drinks. Private respondent GABI was to remit to
Commission regarding the indemnification of theheirs NPDC 40% of the profits derived from operating the kiosks. After the
of the deceased and the victims of the incident does EDSA Revolution, petitioner Lansang, the new Chairman of the
not in any way mean liability authomatically a t t a c h e s NPDC, sought to clean up Rizal Park. Petitioner terminated the so-
to the State. The purpose of which i s to
called verbal agreement with GABI and demanded that the latter
investigate of the disorders that took place and
t h e recommendation it makes cannot in any way bind the State. vacate the premises and the kiosks it ran privately within the public
The acts and utterances of President Aquino doesnot mean park. On the day of the supposed eviction, GABI filed an action for
admission of the State of its liability. Moreover, the case does damages and injunction against petitioner.

Constitutional Law 1 based on the syllabus of Atty. Remoroza 2018-2019 64


Merzy’s Notes Prelim Exam

In order that suit may lie against the state, there must be consent,
either express or implied. Where no consent is shown, state
Issue: Whether or not the complaint filed against the petitioner is in immunity
from suit may be invoked as a defense by the courts sua sponte at
reality a complaint against the State, which could not prosper
any stage of the proceedings, because waiver of immunity, being in
without the State’s consent derogation of sovereignty, will not be inferred lightly and must be
construed in strictissimi juris. Accordingly, the complaint (or
counterclaim) against the State must allege the existence of such
Held: The doctrine of state immunity from suit applies to complaints consent (and where the same is found), otherwise, the complaint
filed against public officials for acts done in the performance of their may be
duties. The rule is that the suit must be regarded as one against the dismissed
a. Express consent
state where satisfaction of the judgment against the public official
concerned will require the state itself to perform a positive act, such Express consent. Express consent can be given only by an act of the
as appropriation of the amount necessary to pay the damages legislative body [Republic v. Feliciano, supra.], in a general or a
awarded to the plaintiff. special law.
General Law. An example of a general law granting
The rule does not apply where the public official is charged in his consent is CA327, as amended by PD 1445, which requires that all
official capacity for acts that are unlawful and injurious to the rights money claims against the government must first be filed with the
Commission on Audit before suit is instituted in court. See: Sayson v.
of others. Public officials are not exempt, in their personal capacity,
Singzon, 54 SCRA 282. of Agriculture may be sued for money claims
from liability arising from acts committed in bad faith. Neither does based on a contract entered into in its governmental capacity,
its apply where the public official is clearly being sued not in his because of the express consent contained in Act No. 3038,provided
official capacity but in his personal capacity, although the acts that the claim be first brought to the Commission on Audit in
complained of may have been committed while he occupied a public accordance
position. In the case, the petitioner is being sued not in his capacity with CA 327, as amended [Department of Agriculture v. NLRC
as NPDC chairman but in his personal capacity. It is also evident the
petitioner is sued allegedly for having personal motives in ordering Department of Agriculture v. NLRC, 227 SCRA
the ejectment of GABI from Rizal Park. 693

Facts: Petitioner Department of Agriculture (DA) and Sultan Security


Agency entered into a contract for security services to be provided
6. Need for consent by the latter to the said governmental entity. Pursuant to their
arrangements, guards were deployed by Sultan Security Agency in
the various premises of the DA. Thereafter, several guards filed a
Constitutional Law 1 based on the syllabus of Atty. Remoroza 2018-2019 65
Merzy’s Notes Prelim Exam

complaint for underpayment of wages, nonpayment of 13th month law. Implied consent, on the other hand, is conceded when the State
pay, uniform allowances, night shift differential pay, holiday pay, and itself commences litigation, thus opening itself to a counterclaim, or
overtime pay, as well as for damages against the DA and the when it enters into a contract. In this situation, the government is
security agency. deemed to have descended to the level of the other contracting
party and to have divested itself of its sovereign immunity.
The Labor Arbiter rendered a decision finding the DA jointly and
severally liable with the security agency for the payment of money In the case, the DA has not pretended to have assumed a capacity
claims of the complainant security guards. The DA and the security apart from its being a governmental entity when it entered into the
agency did not appeal the decision. Thus, the decision became final questioned contract; nor that it could have, in fact, performed any
and executory. The Labor Arbiter issued a writ of execution to act proprietary in character.
enforce and execute the judgment against the property of the DA
and the security agency. Thereafter, the City Sheriff levied on But, be that as it may, the claims of the complainant security guards
execution the motor vehicles of the DA. clearly constitute money claims. Act No. 3083 gives the consent of
the State to be sued upon any moneyed claim involving liability
arising from contract, express or implied. Pursuant, however, to
Issue: Whether or not the doctrine of non-suability of the State Commonwealth Act 327, as amended by PD 1145, the money claim
applies in the case must first be brought to the Commission on Audit.

Held: The basic postulate enshrined in the Constitution that “the Amigable v. Cuenca, 43 SCRA 360
State may not be sued without its consent” reflects nothing less than
a recognition of the sovereign character of the State and an express FACTS: Victoria Amigable rightfully owned a lot in Cebu City which
affirmation of the unwritten rule effectively insulating it from the was used by the government for Mango and Gorordo Avenues
without her permission and without proper negotiation of sales.
jurisdiction of courts. It is based on the very essence of sovereignty.
Because of this, she filed a case in CFI Cebu.
A sovereign is exempt from suit based on the logical and practical
ground that there can be no legal right as against the authority that Defendants argue that 1) Action was premature; 2) Right of action
makes the law on which the right depends. has already been prescribed; 3) Government cannot be sued without
its consent and; 4) Cebu already agreed to use the land as such.
The rule is not really absolute for it does not say that the State may
not be sued under any circumstances. The State may at times be CFI rendered a decision which states that Amigable cannot restore
and recover her ownership and possession of the said land and thus
sued. The State’s consent may be given expressly or impliedly.
Express consent may be made through a general law or a special
Constitutional Law 1 based on the syllabus of Atty. Remoroza 2018-2019 66
Merzy’s Notes Prelim Exam

dismissed the complaint on grounds that state may not be sued a n d assurance of then DPWH Undersecretary Canlas, they undertook
without its consent. additional constructionsf o r t h e c o m p l e t i o n o f t h e p r o j e c t ,
but said additional constructions were not
ISSUE: Whether or not petitioner Amigable may rightfully sue the
i s s u e d payment by DPWH.W i t h a favorable
government without its consent.
recommendation from the DPWH Asst. Secretary for
HELD: In the case of Ministerio vs Court of First Instance of Cebu, it L e g a l Affairs, the petitioners sent a demend letter to the DPWH
was held that when the government takes away property from a Secretary. The DPWH Auditor did not object to the payment subject
private landowner for public use without going through the legal to whatever action COA may adopt.(1992) Through the request of
process of expropriation or negotiated sale, the aggrieved party may then DPWH Secretary De Jesus, the DBM releasedt h e a m o u n t
properly maintain a suit against the government without violating for payment but (1996) respondent DPWH Secreatry
the doctrine of governmental immunity from suit without its consent.
V i g i l a r d e n i e d t h e money claims prompting petitioners to
In the case at bar, since no annotation in favor of the government file a petition for mandamus before the RTC which said trial
appears at the back of the certificate of title and plaintiff has not court denied. Hence, this petition.A m o n g others,
executed any deed of conveyance of any portion of the lot to the respondent-secretary argues that the state may not
government, then she remains as the rightful owner of the lot. b e s u e d invoking the constitutional doctrine of Non-suability of the
State also known as the RoyalPrerogative of Dishonesty.
She could then bring an action to recover possession of the land
anytime, because possession is one of the attributes of ownership. ISSUE: Whether or not the Principle of State Immunity is applicable in
However, since such action is not feasible at this time since the lot the case at bar.
has been used for other purposes, the only relief left is for the
government to make due compensation of the exact amount, price HELD:
or value of the lot at the time of the taking.
The principle of state immunity finds no application
Petition is partly GRANTED
i n t h i s c a s e . U n d e r t h e circumstances, respondent may
EPG Construction v. Secretary Vigilar, G.R. No. not validly invoke the Royal Prerogative of Dishonesty a n d
131544, March 16, 2001 hide under the state’s cloak of invincibility against
s u i t . C o n s i d e r i n g t h a t t h i s principle yields to certain settled
1983) The herein petitioners-contractors, exceptions. The rule is not absolute for it does not saythat the
under contracts with D P W H , constructed 145 state may not be sued under any circumstance. The
housing units but coverage of construction and funding doctrine of governmentali m m u n i t y f r o m s u i t c a n n o t
under the saidc o n t r a c t s w a s o n l y f o r 2 / 3 o f e a c h serve as an instrument for perpetrating an injustice
housing unit. Through the verbal request on acitizen. It is just as important that there be

Constitutional Law 1 based on the syllabus of Atty. Remoroza 2018-2019 67


Merzy’s Notes Prelim Exam

f i d e l i t y t o l e g a l n o r m s o n t h e p a r t o f officialdom if the the installments, the Shipping Commission tool possession of said
rule of law is to be maintained. The ends of justice would be vessel and considered the contract of sale cancelled. The Shipping
subvertedif we were to uphold, in this instance, the state’s immunity Commission chartered and delivered said vessel to the defendant-
appellant Pan Oriental Shipping Co. subject to the approval of the
from suit.This court - as the staunch guardian of the
President of the Philippines. Plaintiff appealed the action of the
citizen’s rights and welfare - cannots a n c t i o n a n i n j u s t i c e Shipping Commission to the President of the Philippines and, in its
so patent on its face, and allow itself to be an meeting the Cabinet restored him to all his rights under his original
instrument of perpetration thereof. Justice and contract with the Shipping Commission. Plaintiff had repeatedly
equity sternly demand that the state’s cloak demanded from the Pan Oriental Shipping Co. the possession of the
of invincibility against suit be shred in this vessel in question but the latter refused to do so.
p a r t i c u l a r i n s t a n c e a n d t h a t p e t i t i o n e r s - contractors be
duly compensated , on the basis of quantum meruit, for construction Pan Oriental protested to this restoration of Plaintiff ‘s rights under
doneon the public works housing projectPetition GRANTted. the contract of sale, for the reason that when the vessel was
delivered to it, the Shipping Administration had authority to dispose
of said authority to the property, Plaintiff having already relinquished
whatever rights he may have thereon. Plaintiff paid the required
b. Implied consent cash of P10,000.00 and as Pan Oriental refused to surrender
possession of the vessel, he filed an action to recover possession
When the State commences litigation, it becomes vulnerable to
a counterclaim. When the State enters into a business contract thereof and have him declared the rightful owner of said property.
The Republic of the Philippines was allowed to intervene in said civil
case praying for the possession of the in order that the chattel
mortgage constituted thereon may be foreclosed.
Froilan v. Pan Oriental Shipping, G.R. No. L-
6060, Sept. 30, 1950 Issues:

Facts: Whether or not the Court has jurisdiction over the intervenor with
regard to the counterclaim.
Plaintiff, Fernando Froilan filed a complaint against the defendant-
appellant, Pan Oriental Shipping Co., alleging that he purchased Discussions:
from the Shipping Commission the vessel for P200,000, paying
P50,000 down and agreeing to pay the balance in instalments. To When the government enters into a contract, for the State is then
secure the payment of the balance of the purchase price, he deem to have divested itself of the mantle of sovereign immunity
executed a chattel mortgage of said vessel in favor of the Shipping and descended to the level of the ordinary individual. Having done
Commission. For various reasons, among them the non-payment of so, it becomes subject to judicial action and processes.

Constitutional Law 1 based on the syllabus of Atty. Remoroza 2018-2019 68


Merzy’s Notes Prelim Exam

Rulings: restrain the defendants from entering into contracts with


third parties for work on the projects.
Yes. The Supreme Court held that the government impliedly allowed  Hence the instant petition which seeks to restrain
itself to be sued when it filed a complaint in intervention for the perpetually the proceedings in Civil Case No. 779-M for lack
purpose of asserting claim for affirmative relief against the plaintiff of jurisdiction on the part of the trial court.
to the recovery of the vessel. The immunity of the state from suits
does not deprive it of the right to sue private parties in its own Issue/s:
courts. The state as plaintiff may avail itself of the different forms of
actions open to private litigants. In short, by taking the initiative in
an action against a private party, the state surrenders its privileged  WON the US naval base in bidding for said contracts exercise
position and comes down to the level of the defendant. The latter governmental functions to be able to invoke state immunity
automatically acquires, within certain limits, the right to set up
whatever claims and other defenses he might have against the state
Held:
WHEREFORE, the petition is granted; the questioned orders of the
U.S. v. Ruiz, 136 SCRA 487 respondent judge are set aside and Civil Case No. is dismissed. Costs
against the private respondent.
 US invited the submission of bids for Repair offender system
and Repair typhoon damages. Eligio de Guzman & Co., Inc. Ratio:
responded to the invitation, submitted bids and complied
with the requests based on the letters received from the US.  The traditional rule of State immunity exempts a State from
 In June 1972, a letter was received by the Eligio De Guzman being sued in the courts of another State without its consent
& Co indicating that the company did not qualify to receive or waiver. This rule is a necessary consequence of the
an award for the projects because of its previous principles of independence and equality of States. However,
unsatisfactory performance rating on a repair contract for the rules of International Law are not petrified; they are
the sea wall at the boat landings of the U.S. Naval Station in constantly developing and evolving. And because the
Subic Bay. activities of states have multiplied, it has been necessary to
 The company sued the United States of America and Messrs. distinguish them-between sovereign and governmental acts
James E. Galloway, William I. Collins and Robert Gohier all (jure imperii) and private, commercial and proprietary acts
members of the Engineering Command of the U.S. Navy. (jure gestionis). The result is that State immunity now
The complaint is to order the defendants to allow the extends only to acts jure imperil (sovereign & governmental
plaintiff to perform the work on the projects and, in the acts)
event that specific performance was no longer possible, to  The restrictive application of State immunity is proper only
order the defendants to pay damages. The company also when the proceedings arise out of commercial transactions
asked for the issuance of a writ of preliminary injunction to of the foreign sovereign, its commercial activities or
economic affairs. Stated differently, a State may be said to
Constitutional Law 1 based on the syllabus of Atty. Remoroza 2018-2019 69
Merzy’s Notes Prelim Exam

have descended to the level of an individual and can thus be sequestration on the 227 NOGCCI shares, among other Benedicto’s
deemed to have tacitly given its consent to be sued only properties, petitioner Republic acknowledging that it was within
when it enters into business contracts. It does not apply private respondent Benedicto’s capacity to acquire the same shares
where the contract relates to the exercise of its sovereign out of his income from business and the exercise of his profession. 6
functions. In this case the projects are an integral part of the Implied in this undertaking is the recognition by petitioner Republic
naval base which is devoted to the defense of both the that the subject shares of stock could not have been ill-gotten.
United States and the Philippines, indisputably a function of
the government of the highest order; they are not utilized In a decision dated October 2, 1992, the Sandiganbayan approved
for nor dedicated to commercial or business purposes. the Compromise Agreement and accordingly rendered judgment in
 correct test for the application of State immunity is not the accordance with its terms.
conclusion of a contract by a State but the legal nature of
the act. In the process of implementing the Compromise Agreement, either
of the parties would, from time to time, move for a ruling by the
Sandiganbayan on the proper manner of implementing or
interpreting a specific provision therein.
In Republic
of Indonesia v. Vinzon, G.R. No. 154705, June 26, 2003, it was held ISSUE: whether or not the public respondent Sandiganbayan,
that contracts entered into by a sovereign state in connection with
Second Division, gravely abused its discretion in holding that the
the establishment of a diplomatic mission, including contracts for the
upkeep or maintenance of air conditioning units, generator sets, PCGG is at fault for not paying the membership dues on the 227
electrical facilities, water heaters and water motor pumps of the sequestered NOGCCI shares of stock, a failing which eventually led
embassy and the Ambassador’s residence, are contracts in jure to the foreclosure sale thereof.
imperii. The fact that the contract contains a provision that any legal
action arising out of the agreement shall be settled according to the HELD:
laws of the Philippines and by a specified court of the Philippines
does not necessarily mean To begin with, PCGG itself does not dispute its being considered as a
a waiver of the state’s sovereign immunity from suit. receiver insofar as the sequestered 227 NOGCCI shares of stock are
concerned.12 PCGG also acknowledges that as such receiver, one of
Republic (PCGG) v. Sandiganbayan, G.R. No. its functions is to pay outstanding debts pertaining to the
129406, March 6, 2006 sequestered entity or property,13 in this case the 227 NOGCCI shares
in question. It contends, however, that membership dues owing to a
golf club cannot be considered as an outstanding debt for which
On November 3, 1990, petitioner Republic and private respondent PCGG, as receiver, must pay. It also claims to have exercised due
Benedicto entered into a Compromise Agreement in Civil Case No. diligence to prevent the loss through delinquency sale of the subject
0034. The agreement contained a general release clause 5 NOGCCI shares, specifically inviting attention to the injunctive suit,
whereunder petitioner Republic agreed and bound itself to lift the
Constitutional Law 1 based on the syllabus of Atty. Remoroza 2018-2019 70
Merzy’s Notes Prelim Exam

i.e., Civil Case No. 5348, it filed before the RTC of Bacolod City to
enjoin the foreclosure sale of the shares.
a) Such execution will require another waiver, because the power of
In fact, by entering into a Compromise Agreement with private the court ends when the judgment is rendered, since government
respondent Benedicto, petitioner Republic thereby stripped itself of funds and properties may not be seized under writs of execution or
its immunity from suit and placed itself in the same level of its garnishment, unless such disbursement is covered by the
adversary. When the State enters into contract, through its officers corresponding appropriation as required by law [Republic v. Villasor,
or agents, in furtherance of a legitimate aim and purpose and 54 SCRA 84; Department of Agriculture v. NLRC, 227 SCRA
pursuant to constitutional legislative authority, whereby mutual or 693]. Thus, in Larkins v. NLRC, 241 SCRA 598, considering that the
reciprocal benefits accrue and rights and obligations arise therefrom, employer of private respondents was not Lt. Col. Frankhauser or the
the State may be sued even without its express consent, precisely petitioner but the U.S. Government which, by right of sovereign
because by entering into a contract the sovereign descends to the power, operated and maintained the dormitories at the Clark Air
level of the citizen. Its consent to be sued is implied from the very Base for USAF members, the awards (of monetary
act of entering into such contract,26 breach of which on its part gives claims to the private respondents) will have to be satisfied by the
the corresponding right to the other party to the agreement. U.S. Government. Without its consent the properties of the U.S.
Government may not be subject to execution.
WHEREFORE, the instant petition is hereby DISMISSED
b) But funds belonging to government corporations (whose charters
provide that they can sue and be sued) that are deposited with a
bank are not exempt from garnishment [Philippine National Bank v.
Pabalan, 83 SCRA 595;
7. Scope of Consent Rizal Commercial Bank v. De Castro, 168 SCRA 49]. In National
Housing Authority v. Heirs of Quivelondo, G.R. No. 154411, June 19,
Consent to be sued does not include consent to the execution of 2003, it was held that if the funds belong to a public corporation or a
judgment against it. government- owned or controlled
corporation which is clothed with a personality of its own, then the
funds are not exempt from garnishment. This is so because when
National Housing Authority v. Heirs of the government enters into commercial business, it abandons its
Quivelondo, G.R. No. 154411, June 19, 2003 sovereign capacity and is to be treated like any other corporation.
NHA is one such corporation; thus, its funds are not exempt
Municipality of San Miguel, Bulacan v. Fernandez, from garnishment or execution.
130 SCRA 56
i) However, in Municipality of San Miguel, Bulacan v. Fernandez,
130 SCRA 56, it was held that funds of a municipality (although it is
Municipality of Makati v. Court of Appeals, 190
an
SCRA 206

Constitutional Law 1 based on the syllabus of Atty. Remoroza 2018-2019 71


Merzy’s Notes Prelim Exam

incorporated agency whose charter provides that it can sue and be


sued) are public in character and may not be garnished unless there
is a corresponding appropriation ordinance duly passed by the 8. Suability not equated with outright liability
Sangguniang Bayan. Thus, in City of Caloocan v. Allarde, G.R. No.
107271, September 10, 2003, the rule was reiterated that all Liability will have to be determined by the Court on the basis of the
government funds deposited with any official depositary bank of the evidence and the applicable law
Philippine Government by any of its agencies or instrumentalities,
whether by Merritt v. Government of the Philippine Islands,
general or special deposit, remain government funds and may not be
34 Phil 311 (1916)
subject to garnishment or levy in the absence of a corresponding
appropriation as required by law. In this case, the City of Caloocan
While consent to be sued was granted through a special law, the
had already approved and passed Ordinance No. 0134, Series of
government was held not liable for damages, because under the
1992, allocating the amount of P439.377.14 for respondent
attendant circumstances the government was not acting through a
Santiago’s back salaries plus interest. Thus, this case fell squarely
special agent
within the exception, and the amount may therefore be garnished.
ia) Be that as it may, in Municipality of Makati v. Court of
Appeals, 190 SCRA 206, it was held that where the municipality fails Fontanilla v. Maliaman, 194 SCRA 486
or refuses, without justifiable reason, to effect payment of a final
money judgment rendered
against it, the claimant may avail of the remedy of mandamus in the Supreme Court said that the National Irrigation Administration is
order to compel a government agency with a juridical personality separate and
the enactment and approval of the necessary appropriation distinct from the government; it is a corporate
ordinance and the body performing proprietary functions. Thus, the NIA may be held
corresponding disbursement of municipal funds to satisfy the money liable for
judgment. damages caused by the negligent act of its driver who was not a
special agent
c) In Pacific Products v. Ong, 181 SCRA 536, the Supreme Court said
that by the process of garnishment, the plaintiff virtually sues the
garnishee for a debt due from the defendant. The debtor-stranger II. CITIZENSHIP
becomes a forced intervenor; when served with the writ of
attachment, he becomes a party to the action. Money in the hands
of government agency (engaged in governmental functions), even if A. General Principles
due to a third party, is not liable to creditors of the third party
through garnishment. 1. Defined
To allow this would be to allow a suit against the State without the
latter’s consent Membership in a political community which is personal and more

Constitutional Law 1 based on the syllabus of Atty. Remoroza 2018-2019 72


Merzy’s Notes Prelim Exam

or less permanent in character application of the jus soli doctrine, but principally because of the
doctrine of res judicata.
Distinguished from nationality. Nationality is membership in any class CASES:
or form of political community. Thus, nationals may be citizens [if
members of a democratic community] or subjects [if members of a Roa v. Collector of Customs, 25 Phil 315
monarchical community]. Nationality does not necessarily include the
right or privilege of exercising civil or political rights. Facts:

2. Usual modes of acquiring citizenship Tranquilino Roa, was born in the town of Luculan, Mindanao,
Philippine Islands, on July 6, 1889. His father was Basilio Roa Uy
a) By birth Tiong Co, a native of China, and his mother was Basilia Rodriguez, a
i) jus sanguinis native of this country. His parents were legally married in the
ii) jus soli
Philippine Islands at the time of his birth. The father of the appellant
b) By naturalization
c) By marriage went to China about the year 1895, and died there about 1900.
Subsequent to the death of his father, in May, 1901, the appellant
was sent to China by his mother for the sole purpose of studying
3. Modes applied in the Philippines (and always with the intention of returning) and returned to the
Philippine Islands on the steamship Kaifong, arriving at the port of
a. Before the adoption of the 1935 Constitution
Cebu October 1, 1910, from Amoy, China, and sought admission to
i) Jus sanguinis. All inhabitants of the islands who were Spanish the Philippine Islands. At this time the appellant was a few days
subjects on April 11,1899, and residing in the islands who did not under 21 years and 3 months of age.
declare their intention of preserving Spanish nationality between said
date and October 11, 1900, were declared citizens of the Philippines Issue:
[Sec. 4, Philippine Bill of 1902; Sec.
2, Jones Law of 1916], and their children born after April 11, 1899. Whether or not Tranquilino Roa was a citizen of the Philippine
Islands by birth?
. ii) Jus soli. As held in Roa v. Collector of Customs, 25 Phil 315,
which Ruling:
was uniformly followed until abandoned in Tan Chong v. Secretary of
Labor, 79 Phil 249; but applied again in Talaroc v. Uy, 92 Phil 52, The nationality of the appellant having followed that of his mother,
until abandoned with finality in Teotimo Rodriguez Tio Tiam v. he was therefore a citizen of the Philippine Islands on July 1, 1902,
Republic, 101 Phil. 195. Those declared as Filipino citizens by the and never having expatriated himself, he still remains a citizen of
courts are recognized as such today, not because of the this country. The United States follow the principle of Jus Soli or
citizenship by place of birth.

Constitutional Law 1 based on the syllabus of Atty. Remoroza 2018-2019 73


Merzy’s Notes Prelim Exam

Basis: Filipino citizens by the courts are recognized as such today, not
because of the application of the jus soli doctrine, but principally
Section 1 of the fourteenth amendment to the Constitution of the because of the doctrine of res judicata
United States reads: All persons born or naturalized in the United
States, and subject to the jurisdiction thereof, are citizens of the b. After the adoption of the 1935 Constitution
United States and of the State wherein they reside Section 4 of the c.
Only the jus sanguinis doctrine.
Philippine Bill provides:

“That all inhabitants of the Philippine Islands continuing to reside 4. Natural-born citizens – Sec. 2, Art. IV
therein who were Spanish subjects on the eleventh day of April,
Those who are citizens of the Philippines from birth
eighteen hundred and ninety-nine, and then resided in said Islands,
without having to perform any act to acquire or perfect their
and their children born subsequent thereto, shall be deemed and Philippine citizenship. Those who elect Philippine citizenship shall be
held to be citizens of the Philippine Islands and as such entitled to deemed natural- born citizens
the protection of the United States, except such as shall have elected
to preserve their allegiance to the Crown of Spain in accordance with 5. Marriage by Filipino to an alien – Sec. 4, Art. IV
the provisions of the treaty of peace between the United States and
“Citizens of the Philippines who marry
Spain signed at Paris December tenth, eighteen hundred and ninety-
aliens shall retain their citizenship, unless by their act or omission
eight.” they are
deemed, under the law, to have renounced it
The cession of the Philippine Islands definitely transferred
the allegiance of the native inhabitants from Spain to the United 6. Policy against dual allegiance – Sec. 5, Art. IV
States (articles 3 and 9 of Treaty of Paris). Filipinos remaining in this
country who were not natives of the Peninsula could not, according ”Dual allegiance of citizens is inimical to
to the terms of the treaty, elect to retain their allegiance to Spain. By the national interest and shall be dealt with by law
the cession their allegiance became due to the United States and
Mercado v. Manzano, 307 SCRA 630
they became entitled to its protection. The nationality of the Islands
American instead of Spanish
Petitioner Ernesto Mercado and Eduardo Manzano were both
Teotimo Rodriguez Tio Tiam v. Republic, 101
candidates for Vice-Mayor of Makati in the May 11, 1998 elections.
Phil. 195
Based on the results of the election, Manzano garnered the highest
Those declared as
number of votes. However, his proclamation was suspended due to

Constitutional Law 1 based on the syllabus of Atty. Remoroza 2018-2019 74


Merzy’s Notes Prelim Exam

the pending petition for disqualification filed by Ernesto Mercado on Consequently, persons with mere dual citizenship do not fall under
the ground that he was not a citizen of the Philippines but of the this disqualification. Unlike those with dual allegiance, who must,
United States. therefore, be subject to strict process with respect to the termination
of their status, for candidates with dual citizenship, it should suffice
From the facts presented, it appears that Manzano is both a Filipino if, upon the filing of their certificates of candidacy, they elect
and a US citizen. Philippine citizenship to terminate their status as persons with dual
citizenship considering that their condition is the unavoidable
The Commission on Elections declared Manzano disqualified as
consequence of conflicting laws of different states.
candidate for said elective position.
When a person applying for citizenship by naturalization takes an
Manzano was eventually proclaimed as the Vice-Mayor of Makati City
oath that he renounces his loyalty to any other country or
on August 31, 1998.
government and solemnly declares that he owes his allegiance to the
Thus the present petition. Republic of the Philippines, the condition imposed by law is satisfied
and complied with. The determination whether such renunciation is
ISSUE: valid or fully complies with the provisions of our Naturalization Law
lies within the province and is an exclusive prerogative of our courts.
Whether or not a dual citizen is disqualified to hold public elective
The latter should apply the law duly enacted by the legislative
office in the philippines.
department of the Republic. No foreign law may or should interfere
RULING: with its operation and application.

The court ruled that the phrase "dual citizenship" in R.A. 7160 Sec. The court ruled that the filing of certificate of candidacy of
40 (d) and R.A. 7854 Sec. 20 must be understood as referring to respondent sufficed to renounce his American citizenship, effectively
dual allegiance. Dual citizenship is different from dual allegiance. The removing any disqualification he might have as a dual citizen. By
former arises when, as a result of the application of the different declaring in his certificate of candidacy that he is a Filipino citizen;
laws of two or more states, a person is simultaneously considered a that he is not a permanent resident or immigrant of another country;
national by the said states. Dual allegiance on the other hand, refers that he will defend and support the Constitution of the Philippines
to a situation in which a person simultaneously owes, by some and bear true faith and allegiance thereto and that he does so
positive act, loyalty to two or more states. While dual citizenship is without mental reservation, private respondent has, as far as the
involuntary, dual allegiance is a result of an individual's volition. laws of this country are concerned, effectively repudiated his
Article IV Sec. 5 of the Constitution provides "Dual allegiance of American citizenship and anything which he may have said before as
citizens is inimical to the national interest and shall be dealt with by a dual citizen.
law."

Constitutional Law 1 based on the syllabus of Atty. Remoroza 2018-2019 75


Merzy’s Notes Prelim Exam

On the other hand, private respondent’s oath of allegiance to the RULING:


Philippines, when considered with the fact that he has spent his
youth and adulthood, received his education, practiced his profession No. The Philippine law on citizenship adheres to the principle of jus
as an artist, and taken part in past elections in this country, leaves sanguinis. Thereunder, a child follows the nationality or citizenship of
no doubt of his election of Philippine citizenship. the parents regardless of the place of his/her birth, as opposed to
the doctrine of jus soli which determines nationality or citizenship on
the basis of place of birth. Private respondent was born a year
Valles v. Comelec, G.R. No. 137000, August 9, before the 1935 Constitution took into effect and at that time, what
2000 served as the Constitution of the Philippines were the principal
organic acts by which the United States governed the country and
these were the Philippine Bill of 1902 and Jones Law. Under both
FACTS: organic acts, all inhabitants of the Philippines who were Spanish
subjects on April 11, 1899 and resided therein including their
Petitioner filed a disqualification case against private respondent in
children are deemed to be Philippine citizens. Her father was born on
1998 elections for governor of Davao Oriental. Rosalind Ybasco
1879 in Daet, Camarines Sur. Therefore by virtue of the said laws,
Lopez (private respondent) was born on May 16, 1934 in Australia,
Telesforo’s daughter, herein private respondent Rosalind Ybasco
to the spouses, Telesforo Ybasco, a Filipino citizen and native of
Lopez, is likewise a citizen of the Philippines. Also, the principle of
Daet, Camarines Norte, and Theresa Marquez, an Australian. In
jus sanguinis, which confers citizenship by virtue of blood
1949, at the age of fifteen, she left Australia and came to settle in
relationship, was subsequently retained under the 1973 and 1987
the Philippines. Petitioner contends that in her application for alien
Constitutions. Thus, the herein private respondent, Rosalind Ybasco
certificate of registration and immigrant certificate of residence,
Lopez, is a Filipino citizen, having been born to a Filipino father.
private respondent expressly declared under oath that she was a
Secondly, as ruled in Aznar vs Comelec and Mercado vs Manzano
citizen or subject of Australia; and said declaration forfeited her
and Comelec, the mere fact that private respondent was a holder of
Philippine citizenship, therefore it operated to disqualify her to run
an Australian passport and had an alien certificate of registration are
for elective office. He also argued that Comelec’s finding of
not acts constituting an effective renunciation of citizenship and do
renouncing her Australian citizenship and had her Australian
not militate against her claim of Filipino citizenship. For renunciation
passport cancelled did not automatically restore the status of private
to effectively result in the loss of citizenship, the same must be
respondent as a Filipino citizen.
express.
ISSUE:
Petition is DISMISSED. COMELEC resolutions AFFIRMED
Whether or not private respondent is an Australian citizen

Constitutional Law 1 based on the syllabus of Atty. Remoroza 2018-2019 76


Merzy’s Notes Prelim Exam

Jacot v. Dal and Comeiec, G.R. No. 179848, requirement of a personal and sworn renunciation of foreign
November 27, 2008 citizenship because these are distinct requirements to be complied
with for different purposes. Section 3 of Republic Act No. 9225
requires that natural-born citizens of the Philippines, who are already
Petitioner Nestor A. Jacot assails the Resolution dated 28 September naturalized citizens of a foreign country, must take the following
2007 of the Commission on Elections (COMELEC) En Banc in SPA oath of allegiance to the Republic of the Philippines to reacquire or
No. 07-361, affirming the Resolution dated 12 June 2007 of the retain their Philippine citizenship.
COMELEC Second Division disqualifying him from running for the
position of Vice-Mayor of Catarman, Camiguin, in the 14 May By the oath dictated in the afore-quoted provision, the Filipino
2007National and Local Elections, on the ground that he failed to swears allegiance to the Philippines, but there is nothing therein on
make a personal renouncement of his United States(US) citizenship. his renunciation of foreign citizenship. Precisely, a situation might
Petitioner was a natural born citizen of the Philippines, who became arise under Republic Act No.9225 wherein said Filipino has dual
a naturalized citizen of the US on 13 December1989.Petitioner citizenship by also reacquiring or retaining his Philippine citizenship,
sought to reacquire his Philippine citizenship under Republic Act No. despite his foreign citizenship. The afore-quoted oath of allegiance is
9225, otherwise known as the Citizenship Retention and Re- substantially similar to the one contained in the Certificate of
Acquisition Act. He filed a request for the administration of his Oath Candidacy which must be executed by any person who wishes to run
of Allegiance to the Republic of the Philippines with the Philippine for public office in Philippine elections. The law categorically requires
Consulate General (PCG) of Los Angeles, California. The Los Angeles persons seeking elective public office, who either retained their
PCG issued on 19 June 2006 an Order of Approval of petitioner’s Philippine citizenship or those who reacquired it, to make a personal
request, and on the same day, petitioner took his Oath of Allegiance and sworn renunciation of any and all foreign citizenship before a
to the Republic of the Philippines before Vice Consul Edward C. Yulo. public officer authorized to administer an oath simultaneous with or
before the filing of the certificate of candidacy. Hence, Section 5(2)
ISSUE: of Republic Act No. 9225 compels natural-born Filipinos, who have
been naturalized as citizens of a foreign country, but who reacquired
whether petitioner is disqualified from running as a candidate in the
or retained their Philippine citizenship (1) to take the oath of
14 May 2007 local elections for his failure to make a personal and
allegiance under Section 3 of Republic Act No. 9225, and (2) for
sworn renunciation of his US citizenship.
those seeking elective public offices in the Philippines, to additionally
HELD: execute a personal and sworn renunciation of any and all foreign
citizenship before an authorized public officer prior or simultaneous
Contrary to the assertions made by petitioner, his oath of allegiance to the filing of their certificates of candidacy, to qualify as candidates
to the Republic of the Philippines made before the Los Angeles PCG in Philippine elections
and his Certificate of Candidacy do not substantially comply with the

Constitutional Law 1 based on the syllabus of Atty. Remoroza 2018-2019 77


Merzy’s Notes Prelim Exam

foreign citizenship. Plainly, from Section 3, Rep. Act No. 9225 stayed
Calilung v. Secretary of Justice, G.R. No. 160869, clear out of the problem of dual allegiance and shifted the burden of
May 11, 2007 confronting the issue of whether or not there is dual allegiance to
the concerned foreign country. What happens to the
Facts: other citizenship was not made a concern of Rep. Act No. 9225.

Petitioner filed a petition for prohibition to prevent Justice Secretary


Datumanong from implementing R. A. 9225 entitled " An Act Making 2. Section 5, Article IV of the Constitution is a declaration of a policy
the Citizenship of Philippine Citizens Who Acquire and it is not a self-executing provision. The legislature still has to
Foreign Citizenship Permanent, Amending for the Purpose enact the law on dual allegiance. In Sections 2 and 3 of Rep. Act No.
Commonwealth Act No. 63, As Amended, and for Other Purposes." 9225, the framers were not concerned with dual citizenship per se,
which was signed into law by President Gloria M. Arroyo on August but with the status of naturalized citizens who maintain their
29, 2003. Petitionerargued that R.A. 9225 is unconstitutional as it allegiance to their countries of origin even after their naturalization.
violates Sec. 5, Article VI of the Constitution which states that “dual Congress was given a mandate to draft a law that would set specific
allegiance of citizens is inimical to national interest and shall be dealt parameters of what really constitutes dual allegiance. Until this is
with by law.” Petitioner contends that RA 9225 cheapens done, it would be premature for the judicial department, including
Philippine citizenship this Court, to rule on issues pertaining to dual allegiance.

Issues:
the constitutionality of R.A. 9225 (An Act Making the Citizenship of
1. Whether R.A. 9225 is unconstitutional Philippine Citizens Who Acquire Foreign Citizenship Permanent,
2. Whether the court jurisdiction to pass upon the issue of dual amending for the purpose,
allegiance Com. Act No. 63) was challenged, allegedly for violating Sec. 5, Art.
IV of the Constitution. It was claimed that Sec. 2 allows all Filipinos,
whether natural-born or naturalized, who become foreign citizens, to
Held: retain their Philippine citizenship without losing their foreign
citizenship; while Sec. 3 allows former natural-bom
1. No. It is clear that the intent of the legislature in drafting Rep. Act citizens to regain their Philippine citizenship by simply taking an oath
No. 9225 is to do away with the provision in Commonwealth Act No. of allegiance without forfeiting their foreign allegiance. In upholding
635 which takes away Philippine citizenship from natural-born the validity of RA 9225, the Court said that the intent of the
Filipinos who become naturalized citizens of other countries. What legislature is to do away with the provision in CA63 which takes
Rep. Act No. 9225 does is allow dual citizenship to natural- away Philippine citizenship from natural-born Filipinos who become
born Filipino citizens who have lost Philippine citizenship by reason naturalized citizens of other countries. It allows dual citizenship; but
of their naturalization as citizens of a foreign country. On its face, it on its face, it does not recognize dual allegiance. By swearing to the
does not recognize dual allegiance. By swearing to the supreme supreme authority of the Republic, the person implicitly renounces
authority of the Republic, the personimplicitly renounces his his foreign citizenship. Plainly, Sec. 3 stays clear out of the problem

Constitutional Law 1 based on the syllabus of Atty. Remoroza 2018-2019 78


Merzy’s Notes Prelim Exam

of dual allegiance and shifts the burden of confronting the issue of Yes. In the year 1895, the private respondent’s grandfather, Ong
whether or not there is dual allegiance to the concerned foreign Te, arrived in the Philippines from China and established his
country. What happens to the other citizenship was not made a residence in the municipality of Laoang, Samar. The father of the
concern of
private respondent, Jose Ong Chuan was born in China in 1905 but
RA 9225.
was brought by Ong Te to Samar in the year 1915, he filed withthe
7. Attack on one’s citizenship may be made only through a court an application for naturalization and was declared a Filipino
direct, not a collateral proceeding citizen.In 1984, the private respondent married a Filipina named
Desiree Lim. For the elections of 1984 and1986, Jose Ong, Jr.
registered himself as a voter of Laoang, Samar, and voted there
Co v. HRET, 199 SCRA 692 during those elections.Under the 1973 Constitution, those born of
Filipino fathers and those born of Filipino mothers with analien father
were placed on equal footing. They were both considered as natural
Facts:
born citizens. Besides, privaterespondent did more than merely
The HRET declared that respondent Jose Ong, Jr. is a natural born exercise his right of suffrage. He has established his life here in the
Filipino citizen and a resident of Laoang, Northern Samar for voting Philippines.On the issue of residence, it is not required that a person
purposes. The congressional election for the second district of should have a house in order to establish hisresidence and domicile.
NorthernSamar was held. Among the candidates who vied for the It is enough that he should live in the municipality or in a rented
position of representative in the second legislativedistrict are the house or in that of afriend or relative. To require him to own
petitioners, Sixto Balinquit and Antonio Co and the private property in order to be eligible to run for Congress would be
respondent, Jose Ong, Jr. RespondentOng was proclaimed the duly tantamountto a property qualification. The Constitution only requires
elected representative of the second district of Northern Samar. The that the candidate meet the age, citizenship, voting and residence
petitioners filed election protests on the grounds that Jose Ong, Jr. is requirements..
not a natural born citizen of thePhilippines and not a resident of the
Yes. The Constitution explicitly provides that the House of
second district of Northern Samar.
Representatives Electoral Tribunal (HRET) and the Senate Electoral
Issue: Tribunal (SET) shall be the sole judges of all contests relating to the
election, returns, and qualifications of their respective members. In
Whether or not Jose Ong, Jr. is a citizen of the Philippines. the case at bar, the Court finds no improvident use of power, no
denial of due process on the part of the HRET which will necessitate
Whether the decision of HRET is appealable; the exercise of the power of judicial review by the Supreme Court.
Held:

Constitutional Law 1 based on the syllabus of Atty. Remoroza 2018-2019 79


Merzy’s Notes Prelim Exam

8. Res judicata in cases involving citizenship William Gatchalian is declared as a Filipino Citizen. Having declared
the assailed marriage as valid, respondent William Gatchalian follows
The doctrine of res judicata does not ordinarily apply to questions of the citizenship of his father, a Filipino as legitimate child. Respondent
citizenship. It does so only A person’s citizenship is resolved by a
belongs to a class of Filipinos who are citizens of the Philippines at
court or an administrative body as a material
issue in the controversy, after a full-blown hearing; (b) With the the time of the adoption of the constitution.
active participation of the Solicitor General or his representative; and
(c) The finding of his citizenship is affirmed by the Supreme Court.
Then the decision on the matter shall constitute
conclusive proof of such party’s citizenship in any other case or B. Citizens of the Philippines
proceeding [Board of Commissioners, CID v. de la Rosa 1. Those who are citizens of the Philippines at the time of
the adoption of this (1987) Constitution

Board of Commissioners, CID v. de la Rosa, 197 a) Re: 1935 Constitution


i) Sec. 4, Philippine Bill of 1902; Sec. 2, Jones Law of 1916 [including
SCRA 853
children born after April 11, 1899],

ia) In Valles v. Comelec, supra., the Supreme Court made


Facts:
reference to these organic acts and declared that private respondent
Rosalind Ybasco Lopez who was born in Australia to parents
On July 12, 1960, Santiago Gatchalian, grandfather of William
Telesforo Ybasco, a Filipino, and Theresa Marquez, an Australian, on
Gatchalian, was recognized by the Bureau of Immigration as a native May 16, 1934, before the 1935 Constitution took effect, was a
born Filipino citizen following the citizenship of natural mother Filipino citizen. Under these organic acts, inhabitants of the islands
Mariana Gatchalian. On June 27, 1961, Willian, then twelve years who were Spanish subjects on April 11, 1899, who did not opt in
old, arrives in Manila from Hongkong together with a daughter and a writing to retain Spanish nationality between April 11, 1899 to
son of Santiago. They had with them certificate of registration and October 11, 1900 — including their children — were deemed citizens
of the Philippines.
identity issued by the Philippine consulate in Hongkong based on a
Rosalind’s father was, therefore, a Filipino citizen, and under the
cablegram bearing the signature of the secretary of foreign affairs,
principle of jus sanguinis, Rosalind followed the citizenship of her
Felixberto Serrano, and sought admission as Filipino citizens. father.

ISSUE: Whether or not William Gatchalian is to be declared as a jb) A similar conclusion was reached in Maria Jeanette Tecson
Filipino citizen v. Comelec, G.R. No. 161434, March 3, 2004, on the controversy
surrounding the citizenship of Fernando Poe, Jr. (FPJ), presidential
Held: candidate. The issue of whether or not FPJ is a natural-born citizen

Constitutional Law 1 based on the syllabus of Atty. Remoroza 2018-2019 80


Merzy’s Notes Prelim Exam

would depend on whether his father, Allan F. Poe, was himself a office in the Islands. In Chiongbian v. de Leon, the Supreme Court
Filipino citizen, and if in the affirmative, whether or not held that the
the alleged illegitimacy of FPJ prevents him from taking after the right acquired by virtue of this provision is transmissible.
Filipino
citizenship of his putative father. The Court took note of the fact that vi) Those who elected Philippine citizenship.
Lorenzo Pou (father of Allan F. Poe), who died in 1954 at 84 years b) Re: 1973 Constitution. Those whose mothers are citizens of the
old, would have been born sometime in 1870, when the Philippines Philippines. Provision is prospective in application; to benefit only
was under Spanish rule, and that San those born on or after January 17, 1973
Carlos, Pangasinan, his place of residence upon his death in 1954, in
the absence of any other evidence, could have well been his place of
residence before death, such that Lorenzo Pou would have benefited Tecson v. Comelec, G.R. No. 161434, March 3,
from the “en masse Filipinization” that the Philippine Bill of 1902 2004
effected. That Filipino citizenship of Lorenzo Pou, if acquired, would
thereby extend to his son, Allan F. Poe (father of FPJ), The 1935
Constitution, during which
Facts:
regime FPJ has seen first light, confers citizenship to all persons
whose fathers are Filipino citizens regardless of whether such On 31 December 2003, Ronald Allan Kelly Poe, also known as
children are legitimate or illegitimate.
Fernando Poe, Jr. (FPJ), filed his certificate of candidacy for the
ii) Act No. 2927 [March 26,1920], then CA473, on naturalization position of President of the Republic of the Philippines under the
[including children below 21 and residing in the Philippines at the Koalisyon ng Nagkakaisang Pilipino (KNP) Party, in the 2004 national
time of elections. In his certificate of candidacy, FPJ, representing himself to
naturalization, as well as children born subsequent to naturalization], be a natural-born citizen of the Philippines, stated his name to be
"Fernando Jr.," or "Ronald Allan" Poe, his date of birth to be 20
iii) Foreign women married to Filipino citizens before or after
August 1939 and his place of birth to be Manila. Victorino X. Fornier,
November 30, 1938 [effectivity of CA 473] who might themselves be
lawfully naturalized [in view of the Supreme Court interpretation of (GR 161824) initiated, on 9 January 2004, a petition (SPA 04-003)
Sec. 15, CA473, in Moy Ya Lim Yao v. Commissioner of Immigration, before the Commission on Elections (COMELEC) to disqualify FPJ and
41 SCRA 292]. to deny due course or to cancel his certificate of candidacy upon the
thesis that FPJ made a material misrepresentation in his certificate of
iv) Those benefited by the Roa doctrine applying the jus soli candidacy by claiming to be a natural-born Filipino citizen when in
principle. truth, according to Fornier, his parents were foreigners; his mother,
Bessie Kelley Poe, was an American, and his father, Allan Poe, was a
v) Caram provision: Those born in the Philippines of foreign parents
who, before the adoption of this [1935] Constitution, had been Spanish national, being the son of Lorenzo Pou, a Spanish subject.
elected to public Granting, Fornier asseverated, that Allan F. Poe was a Filipino

Constitutional Law 1 based on the syllabus of Atty. Remoroza 2018-2019 81


Merzy’s Notes Prelim Exam

citizen, he could not have transmitted his Filipino citizenship to FPJ, the primary basis of citizenship by birth. Considering the reservations
the latter being an illegitimate child of an alien mother. Fornier made by the parties on the veracity of some of the entries on the
based the allegation of the illegitimate birth of FPJ on two birth certificate of FPJ and the marriage certificate of his parents, the
assertions: (1) Allan F. Poe contracted a prior marriage to a certain only conclusions that could be drawn with some degree of certainty
Paulita Gomez before his marriage to Bessie Kelley and, (2) even if from the documents would be that (1) The parents of FPJ were Allan
no such prior marriage had existed, Allan F. Poe, married Bessie F. Poe and Bessie Kelley; (2) FPJ was born to them on 20 August
Kelly only a year after the birth of FPJ. On 23 January 2004, the 1939; (3) Allan F. Poe and Bessie Kelley were married to each other
COMELEC dismissed SPA 04-003 for lack of merit. on 16 September, 1940; (4) The father of Allan F. Poe was Lorenzo
Poe; and (5) At the time of his death on 11 September 1954,
Issue: Lorenzo Poe was 84 years old. The marriage certificate of Allan F.
Poe and Bessie Kelley, the birth certificate of FPJ, and the death
Whether FPJ was a natural born citizen, so as to be allowed to run
certificate of Lorenzo Pou are documents of public record in the
for the offcie of the President of the Philippines.
custody of a public officer
Held:
Valles v. Comelec, G.R. No. 137000, August 9,
Section 2, Article VII, of the 1987 Constitution expresses that "No 2000
person may be elected President unless he is a natural-born citizen
of the Philippines, a registered voter, able to read and write, at least
forty years of age on the day of the election, and a resident of the 2. Those whose fathers or mothers are citizens of the
Philippines for at least ten years immediately preceding such Philippines
election." The term "natural-born citizens," is defined to include
"those who are citizens of the Philippines from birth without having 3. Those born before January 17, 1973, of Filipino mothers,
to perform any act to acquire or perfect their Philippine citizenship." who elect Philippine citizenship upon reaching the age of
Herein, the date, month and year of birth of FPJ appeared to be 20 majority
August 1939 during the regime of the 1935 Constitution. Through its
history, four modes of acquiring citizenship - naturalization, jus soli, a. Procedure for election
res judicata and jus sanguinis – had been in vogue. Only two, i.e.,
Election is expressed in a statement to be
jus soli and jus sanguinis, could qualify a person to being a “natural-
signed and sworn to by the party concerned before any official
born” citizen of the Philippines. Jus soli, per Roa vs. Collector of authorized to administer oaths. Statement to be filed with the
Customs (1912), did not last long. With the adoption of the 1935 nearest Civil Registry. The statement is to be accompanied with the
Constitution and the reversal of Roa in Tan Chong vs. Secretary of Oath of Allegiance to the Constitution and the Government of the
Labor (1947), jus sanguinis or blood relationship would now become Philippines

Constitutional Law 1 based on the syllabus of Atty. Remoroza 2018-2019 82


Merzy’s Notes Prelim Exam

b. When to elect HELD:

Within three (3) years from reaching the age of majority No. Election must be made within a reasonable period after reaching
[Opinion, Secretary of Justice, s. 1948]; except when there is a the age of majority. 3 years is a reasonable period, however, may be
justifiable reason for the delay. extended under certain circumstances as when the person
concerned has always considered himself a Filipino citizen. In the
Cuenco v. Secretary of Justice, 5 SCRA 110
case at bar the petitioner reached the aged of majority 1944, he
made his election of citizenship 1951 when he was 28 years old, 7
FACTS: years after he reached the age of majority. Petitioner cited his
reasons for the delayed election but the court stated that it was
Appeal from a decision of the Court of First Instance of Manila. insufficient excuse for the delay of the said election.

counsel for petitioner Alfonso Dy Cuenco wrote to the Commissioner WHEREFORE, the decision appealed from is hereby reversed, and
of Immigration a letter requesting the cancellation of his alien another one shall be entered dismissing the petition, with costs
certificate of registration, upon the ground that he had exercised the against petitioner.
right to elect Philippine citizenship pursuant to Article IV, section I(4)
of the Constitution and Commonwealth Act No. 625. Said election
appears in an affidavit dated May 15, 1951, stating that petitioner In Re: Ching, Bar Matter No. 914, October 1,
was born in Dapa, Surigao, on February 16, 1923; that his parents 1999
are "Benito Dy Cuenco, Chinese (now deceased)" and "Julita
Duyapit, Filipina, a native of Surigao, Philippines"; that he is "married
In 1998, Vicente Ching finished his law degree at the Saint Louis
to Rosalinda Villanueva, a Filipino," by whom he has four (4)
University in Baguio City. He eventually passed the bar but he was
legitimate children; that he renounces all allegiance to the Republic
advised that he needs to show proof that he is a Filipino citizen
of China; that he recognizes and accepts the supreme authority of
before he be allowed to take his oath. Apparently, Ching’s father was
the Republic of the Philippines and will maintain true faith and
a Chinese citizen but his mother was a Filipino citizen. His parents
allegiance thereto; and that he will obey, support and defend the
were married before he was born in 1963. Under the 1935
Constitution and laws of the Philippines. On the same date,
Constitution, a legitimate child, whose one parent is a foreigner,
petitioner, likewise, took the corresponding oath of allegiance to the
acquires the foreign citizenship of the foreign parent. Ching
Republic of the Philippines.
maintained that he has always considered himself as a Filipino; that
ISSUE: Whether or not the election of Philippine Citizenship of the he is a certified public accountant – a profession reserved for
petitioner is valid. Filipinos; that he even served as a councilor in a municipality in La
Union.
Constitutional Law 1 based on the syllabus of Atty. Remoroza 2018-2019 83
Merzy’s Notes Prelim Exam

The Solicitor-General commented on the case by saying that as a 2. No. Unfortunately, he belatedly elected Filipino citizenship. The
legitimate child of a Chinese and a Filipino, Ching should have Supreme Court cannot agree with the recommendation of the
elected Filipino citizenship upon reaching the age of majority; that Solicitor-General. Fourteen years had lapsed and it’s way beyond the
under prevailing jurisprudence, “upon reaching the age of majority” allowable 7 year period. The Supreme Court even noted that the
is construed as within 7 years after reaching the age of majority (in period is originally 3 years but it was extended to 7 years. (It seems
his case 21 years old because he was born in 1964 while the 1935 it can’t be extended any further). Ching’s special circumstances can’t
Constitution was in place). be considered. It is not enough that he considered all his life that he
is a Filipino; that he is a professional and a public officer (was)
Ching did elect Filipino citizenship but he only did so when he was serving this country. The rules for citizenship are in place. Further,
preparing for the bar in 1998 or 14 years after reaching the age of Ching didn’t give any explanation why he belatedly chose to elect
majority. Nevertheless, the Solicitor-General recommended that the Filipino citizenship (but I guess it’s simply because he never thought
rule be relaxed due to the special circumstance of Ching. he’s Chinese not until he applied to take the bar). The prescribed
procedure in electing Philippine citizenship is certainly not a tedious
ISSUE:
and painstaking process. All that is required of the elector is to
Whether or not he has elected Philippine citizenship within "a execute an affidavit of election of Philippine citizenship and,
reasonable time". thereafter, file the same with the nearest civil registry. Ching’s
unreasonable and unexplained delay in making his election cannot
Whether or not Ching should be allowed to take the lawyer’s oath. be simply glossed over.

RULING:
Republic v. Chule Lim, G.R. No. 153883,
1. No. Ching, despite the special circumstances, failed to elect
January 13, 2004
Philippine citizenship within a reasonable time. The reasonable time
means that the election should be made within 3 years from "upon
reaching the age of majority", which is 21 years old. Instead, he In 1999, Chuley Lim filed a petition for correction of entries in her
elected Philippine citizenship 14 years after reaching the age of birth certificate with the regional trial court of Lanao del Norte. Her
majority which the court considered not within the reasonable time. maiden name was Chuley Yu and that’s how it appears in all her
Ching offered no reason why he delayed his election of Philippine official records except that in her birth certificate where it appears as
citizenship, as procedure in electing Philippine citizenship is not a “Chuley Yo”. She said that it was misspelled. The Republic of the
tedious and painstaking process. All that is required is an affidavit of Philippines through the local city prosecutor raised the issue of
election of Philippine citizenship and file the same with the nearest citizenship because it appears that Lim’s birth certificate shows that
civil registry. she is a Filipino. The prosecutor contends that Lim’s father was a

Constitutional Law 1 based on the syllabus of Atty. Remoroza 2018-2019 84


Merzy’s Notes Prelim Exam

Chinese; that she acquired her father’s citizenship pursuant to the not apply in the case of respondent who was concededly an
1935 Constitution in place when she was born; that she never illegitimate child, considering that her Chinese father
elected Filipino citizenship when she reached the age of majority and Filipino mother were never married. As such, she was not
required to comply with said constitutional and
(she is already 47 years old at that time); that since she is a
statutory requirements to become a Filipinocitizen. By being an
Chinese, her birth certificate should be amended to reflect that she illegitimate child of a Filipino mother, respondentautomatically
is a Chinese citizen. Lim contends that she is an illegitimate child became a Filipino upon birth. Stated differently, she is
hence she is a Filipino. a Filipinosince birth without having to elect Filipino citizenship when
she reached the age of majority.
ISSUES:
This notwithstanding, the records show that respondent
1. Whether or not Lim complied with the legal requirement in elected Filipino citizenship when she reached the age of
electing her citizenship majority. She registered as a voter in Misamis Oriental when she
2. Whether the CA erred in allowing Lim to to use her father’s was 18 years old. The exercise of the right of suffrage and the
surname despite its finding that she is illegitimate. participation in election exercises constitute a positive act of election
of Philippine citizenship.

HELD:
1. The Republic avers that respondent did not comply with the
4. Those who are naturalized in accordance with law
constitutional requirement of electing Filipino citizenship when she
reached the age of majority. It cites Article IV, Section 1(3) of the
1935 Constitution, which provides that the citizenship of a legitimate
child born of a Filipino mother and an alien father followed the C. Naturalization
citizenship of the father, unless, upon reaching the age of majority,
the child elected Philippine citizenship. Likewise, the Republic The act of formally adopting a foreigner into the political body
invokes the provision in Section 1 of Commonwealth Act No. 625, of a nation by clothing him or her with the privileges of a citizen.
that legitimate children born of Filipinomothers may elect Philippine
citizenship by expressing such intention “in a statement to be signed 1. Modes of naturalization
and sworn to by the party concerned before any officer authorized to
administer oaths, and shall be filed with the nearest civil Direct: Citizenship is acquired by: (i) Individual, through judicial or
registry. The said party shall accompany the aforesaid statement administrative proceedings; (ii) Special act of legislature; (iii)
with the oath of allegiance to the Constitution and the Government Collective change of nationality, as a result of cession or
of the Philippines.” subjugation; or (iv) In some cases, by adoption of orphan minors as
nationals of the State where they are born.
Plainly, the above constitutional and statutory requirements of
electing Filipinocitizenship apply only to legitimate children. These do b) Derivative: Citizenship conferred on: (i) Wife of naturalized

Constitutional Law 1 based on the syllabus of Atty. Remoroza 2018-2019 85


Merzy’s Notes Prelim Exam

husband; (ii) Minor children of naturalized person; or on the (iii) instruction of persons of a particular nationality or race) or in any of
Alien woman upon marriage to a national. the branches of education or industry for a period of not less than
two years, or bom in the Philippines;
2. Doctrine of indelible allegiance [c] Good moral character; believes in the principles underlying the
Philippine Constitution; must have conducted himself in a proper and
An individual may be compelled to retain irreproachable manner during the entire period of his residence in
his original nationality even if he has already renounced or forfeited the
it under the laws of the second State whose nationality he has Philippines in his relations with the constituted government as well
acquired as the community in which he is living;

3. Direct naturalization under Philippine laws [d] Own real estate in the Philippines worth not less than P5,000.00,
or must have some known lucrative trade, profession or
Under current and existing lawful occupation;
laws, there are three (3) ways by which an alien may become a
citizen of the Philippines by naturalization: [e] Speak and write English or Spanish and any of the principal
Philippine languages;
a) judicial naturalization under Commonwealth Act No. 473, as
amended; [f] Enrolled his minor children of school age in any of the
public or private schools recognized by the Government where
b) administrativ e naturalization under Rep. Act No. 9139; and Philippine history, government and civics are taught as part of the
school curriculum, during the entire
c) legislative naturalization in the form of a law enacted by Congress, period of residence in the Philippines required of him prior to the
bestowing Philippine citizenship to an alien hearing of his petition for naturalization.

4. Naturalization under C.A. 473 b. Disqualifications


a. Qualifications
Those [a] Opposed to organized government or
[a] Not less than 21 years of age on the date of the affiliated with any association or group of persons who uphold and
hearing of the petition; teach doctrines opposing all organized governments;

[b] Resided in the Philippines for a continuous period of not less [b] Defending or teaching the necessity or propriety of violence,
than 10 years; may be reduced to 5 years if he honorably held office personal assault or assassination for the success or predominance of
in Government, established a new industry or introduced a useful their ideas;
invention in the Philippines, married to a Filipino woman, been
engaged as a teacher in the Philippines (in a public or private school [c] Polygamists or believers in polygamy;
not established for the exclusive

Constitutional Law 1 based on the syllabus of Atty. Remoroza 2018-2019 86


Merzy’s Notes Prelim Exam

[d] Convicted of a crime involving moral turpitude; [e] Suffering


from mental alienation iii) Publication of the petition. Under Sec. 9, Revised Naturalization
or incurable contagious disease; [f] Who, during the period of their Law, in order that there be a valid publication, the following
residence in the Philippines, have not mingled socially with the requisites must concur:
Filipinos, or who have not evinced a sincere desire to learn and (a) the petition and notice of hearing must be published;
embrace the customs, traditions and ideals
of the Filipinos; (b) the publication must
be made once a week for three consecutive weeks; and
[g] Citizens or subjects of nations with whom the Philippines is at
war, during the period of such war; [h] Citizens or subjects of a (c) the publication must
foreign country whose laws do not grant Filipinos the right to be in the Official Gazette and in a newspaper of general circulation in
become naturalized citizens or subjects thereof. the province where the applicant resides. In addition, copies of the
petition and notice of hearing must be posted in the office of the
c. Procedure Clek of Court or in the building where the office is located [Republic
v. Hamilton Tan Keh, G.R. No. 144742, November 11, 2004],
i) Filing of declaration of intention one year prior to the filing of the
petition with the Office of the Solicitor General. The following are The same notice must also indicate, among others, the names of the
exempt from filing declaration of intention: witnesses whom the petitioner proposes to introduce at the trial
[Republic v. Michael Hong, G.R. No. 168877 March 23 2006], ’
ia) Born in the Philippines and have received their primary and
secondary education in public or private schools recognized by the iiia) Publication is a jurisdictional requirement. Noncompliance
Government and not limited to any race or nationality. is fatal for it impairs the very root or foundation of the authority to
decide the case, regardless of whether the one to blame is the clerk
ib) Resided in the Philippines for 30 years or more before the of court or the petitioner or his counsel [Gan Tsitung v. Republic,
filing of the petition, and enrolled his children in elementary and high 122 Phil. 805; Po Yo Bi v. Republic, 205 SCRA 400].
schools recognized by the Government and not limited to any race or
nationality. iiib) This rule applies equally to the determination of the
sufficiency of the contents of the notice of hearing and of the
ic) Widow and minor children of an alien who has declared his petition itself, because an incomplete notice or petition, even if
intention to become a citizen of the Philippines and dies before he is published, is no publication at all. Thus, in Sy v. Republic, 154 Phil.
actually naturalized. 673, it was held that the copy of the petition to be posted and
published should be a textual or verbatim restatement of the petition
ii) Filing of the petition, accompanied by the affidavit of two credible filed.
persons, citizens of the Philippines, who personally know the
petitioner, as character witnesses. iiic) In the same vein, the failure to state all the required details

Constitutional Law 1 based on the syllabus of Atty. Remoroza 2018-2019 87


Merzy’s Notes Prelim Exam

in the notice of hearing, like the names of applicant’s witnesses, petition was held earlier than the scheduled date of hearing; the
constitutes a fatal defect. The publication of the affidavit of such petition was heard within 6 months from the last publication; the
witnesses did not cure the omission of their names in the notice of petitioner was allowed to take the oath of allegiance before finality
hearing. It is a settled rule that naturalization laws of the judgment, and without observing the twoyear probationary
should be rigidly enforced and strictly construed in favour of the period.]
government and against the applicant [Ong Chua v. Republic G R No
127240, March 27, 2000].
d. Effects of naturalization
iv) Actual residence in the Philippines during the entire proceedings.
i) Vests citizenship on wife if she herself may be lawfully
v) Hearing of the petition. naturalized (as interpreted by the Supreme Court in Moy Ya Lim Yao
v.
vi) Promulgation of the decision. Commissioner of Immigration, supra.).

vii) Hearing after two years. At this hearing, the applicant shall show ia) In Moy Ya Lim Yao, the Court said that the alien wife of the
that during the two-year probation period, applicant has (i) not left naturalized Filipino need not go through the formal process of
the naturalization in order to acquire Philippine citizenship. All she has to
do is to file before the Bureau of Immigration and Deportation a
petition for the cancellation of her Alien Certificate of Registration
Philippines; (ii) dedicated himself continuously to a lawful calling or (ACR). At the hearing on the petition, she does not have to prove
profession; (iii) not been convicted of any offense or violation of that she possesses all the qualifications for naturalization; she only
rules; and (iv) not committed an act prejudicial to the interest of the has to show that she does not labor under any of the
nation or contrary to any Government announced policies. disqualifications. Upon the grant
of the petition for cancellation of the ACR, she may then take the
viii) Oath taking and issuance of the Certificate of Naturalization. oath of the allegiance to the Republic of the Philippines and thus,
[In Republic v. de la Rosa, 232 SCRA 785], and companion cases, become a citizen of the Philippines.
the Supreme Court noted several irregularities which punctuated the
petition and the proceedings in the application for naturalization of ii) Minor children born in the Philippines before the naturalization
Juan C. Frivaldo, viz: the petition lacked several allegations required shall be considered citizens of the Philippines.
by Secs. 2 and 6 of the Naturalization Law; the petition and the iii) Minor child born outside the Philippines who was residing in the
order for hearing were not published once a week for three Philippines at the time of naturalization shall be considered a Filipino
consecutive weeks in the Official Gazette and in a newspaper of citizen.
general circulation; the petition was not supported by affidavits of
two credible witnesses vouching for the good moral character of the iv) Minor child born outside the Philippines before parent’s
petitioner; the actual hearing of the naturalization shall be considered Filipino citizens only during
minority, unless he begins to reside permanently in the Philippines.

Constitutional Law 1 based on the syllabus of Atty. Remoroza 2018-2019 88


Merzy’s Notes Prelim Exam

But if the ground was personal to the denaturalized Filipino, his wife
v) Child born outside the Philippines after parent’s naturalization and children shall retain their Philippine citizenship.
shall be considered a Filipino, provided that he registers as such
before any Philippine consulate within one year after attaining 5. Naturalization by direct legislative action
majority age, and takes his oath of allegiance.
This is discretionary on Congress; usually conferred on an alien who
e. Denaturalization has made outstanding contributions to the country.

i} Grounds: 6. Administrative Naturalization – R.A. 9139

ia) Naturalization certificate is obtained fraudulently or illegally. In The “Administrative


Republic v. Li Yao, 214 SCRA 748, the Supreme Court declared that Naturalization Law of 2000” would grant Philippine citizenship by
a certificate of naturalization may be cancelled if it is subsequently administrative
discovered that the applicant obtained it by misleading the court proceedings to aliens born and residing in the Philippines. In So v.
upon any material fact. Availment of a tax amnesty does not have Republic, G.R. No. 170603, January 29, 2007, the Supreme Court
the effect of obliterating his lack of good moral character. declared that CA 473 and RA 9139 are separate and distinct laws.
The former covers aliens regardless of class, while the latter covers
ib) If, within 5 years, he returns to his native country or to some native-born aliens who lived in the Philippines all their lives,
foreign country and establishes residence there; provided, that 1- who never saw any other country and all along thought that they
year stay in native country, or 2-year stay in a foreign country shall were Filipinos, who have demonstrated love and loyalty to the
be prima facie evidence of intent to take up residence in the same. Philippines and affinity to Filipino customs and traditions. The
intention of the legislature in enacting RA 9139 was to make the
ic) Petition was made on an invalid declaration of intention. process of acquiring Philippine citizenship less tedious, less technical,
id) Minor children failed to graduate through the fault of the parents and more encouraging. There is nothing in the law from which it can
either by neglecting to support them or by transferring them to be inferred that CA473 is intended to be annexed to or repealed by
another school. RA 9139. What the legislature had in mind was merely to prescribe
another mode of acquiring Philippine citizenship which may be
ie) Allowed himself to be used as a dummy. availed of by native-born aliens. The only
[In Republic v. Guy, 115 SCRA 244, although the misconduct was implication is that a native- born alien has the choice to apply for
committed after judicial or administrative naturalization, subject to the prescribed
the two-year probationary period, conviction of perjury and rape was qualifications and disqualifications.
held to be valid ground for denaturalization.]
a) Special Committee on Naturalization. Composed of the Solicitor
ii) Effects of denaturalization: If the ground for denaturalization General, as chairman, the Secretary of Foreign Affairs or his
affects the intrinsic validity of the proceedings, the denaturalization representative, and the National Security Adviser, as members, this
shall divest the wife and children of their derivative naturalization.

Constitutional Law 1 based on the syllabus of Atty. Remoroza 2018-2019 89


Merzy’s Notes Prelim Exam

Committee has the power toapprove, deny or reject applications for


naturalization under this Act c) Disqualifications: The same as those provided in C.A. 473.

b) Qualifications: Applicant must d) Procedure: Filing with the Special Committee on Naturalization of
a
[1] be born in the Philippines and petition (see Sec. 5, RA 9139, for contents of the petition);
residing therein since birth publication of pertinent portions of the petition once a week for
three consecutive weeks in a newspaper of general circulation, with
[2] not be less than 18 years of age, at the time of filingof his/her copies thereof posted in any public or conspicuous area; copies also
petition; furnished the Department of Foreign Affairs, Bureau of Immigration
and Deportation, the civil registrar of petitioner’s place of residence
[3] be of good moral character and believes in the underlying and the National Bureau of Investigation which shall post copies of
principles of the Constitutioin and must have conducted himself/ the petition in
herself in a proper and irreproachable manner during his/her entire any public or conspicuous areas in their buildings offices and
period of residence in the Philippines in his relatioins with the duly premises, and within 30 days submit to the Committee a report
constituted government as well as withthe community in which stating whether or not petitioner has any derogatory record on file or
he/she is living; any such relevant and material information which might be adverse
to petitioner’s application for citizenship; Committee shall, within 60
[4] have received his/her primary and secondary education in any days from receipt of the report of the agencies, consider and review
public school or private educational institution duly recognized by the all information received pertaining to the petition (if Committee
Department of Education, where Philippine history, government and receives any information adverse to the petition, the Committee shall
civics are taught and prescribed as part of the school curriculum and allow the petitioner to answer, explain or refute the information);
where enrolment is not limited to any race or nationality, provided Committee shall then approve or deny the petition. Within 30 days
that should he/she have minor children of school age, he/she must from approval of the petition, applicant shall pay to the Committee a
have enrolled them in similar schools; fee of P100,000, then take the oath of allegiance and a certificate of
naturalization shall issue. Within 5 days after the applicant has taken
[5] have a known trade, business, profession or lawful occupation, his oath of allegiance, the Bureau of Immigration shall forward a
from copy of the oath to the proper local civil registrar, and thereafter,
which he/she derives income sufficient for his/her support and that cancel petitioner’s alien certificate of registration.
of his/her family; provided that this shall not apply to applicants who
are college degree holders but are unable to practice their profession e) Status of Alien Wife and Minor Children. After the approval of the
because they are disqualified to do so by reason of their petition for administrative naturalization and cancellation of the
citizenship; [6] be able to read, write and speak Filipino or any of the applicant’s alien certificate of registration, applicant’s alien lawful
dialects of the Philippines; and [7] have mingled with the Filipinos wife and minor children may file
and evinced a sincere desire to learn and embrace the customs and a petition for cancellation of their alien certificates of registration
traditions and ideals of the Filipino people. with the

Constitutional Law 1 based on the syllabus of Atty. Remoroza 2018-2019 90


Merzy’s Notes Prelim Exam

Committee, subject to the payment of the required fees. But, if the as an employee, he derives an average annual income of
applicant is a married woman, the approval of her petition for around P100,000.00 with free board and lodging and other benefits;
administrative naturalization shall he is single, able to speak and write English, Chinese and Tagalog;
not benefit her alien husband, although her minor children may still he is exempt from the filing of Declaration of Intention to become a
avail of the right to seek the cancellation of their alien certificate of citizen of the Philippines pursuant to Section 6 of Commonwealth Act
registration. (C.A.) No. 473.

f) Cancellation of the Certificate of Naturalization. The Special Issue:


Committee on Naturalization may cancel certificates of naturalization W/N Edison So did meet all the qualification needed to be a
issued under naturalized Filipino citizen.
this act in the following cases
Ruling:
: [1] if the naturalized person or his duly authorized representative The petition is denied for lack of merit.
made any false statement or misrepresentation or committed any Naturalization signifies the act of formally adopting a foreigner into
violation of law, rules and regulations in connection with the the political body of a nation by clothing him or her with the
petition, or if he obtains Philippine citizenship fraudulently or privileges of a citizen.44 Under current and existing laws, there are
illegally; three ways by which an alien may become a citizen by
naturalization: (a) administrative naturalization pursuant to R.A. No.
[2] if, within five years, he shall establish permanent residence in a 9139; (b) judicial naturalization pursuant to C.A. No. 473, as
foreign country, provided that remaining for more than one year in amended; and (c) legislative naturalization in the form of a law
his country of origin or two years in any foreign country shall be enacted by Congress bestowing Philippine citizenship to an alien.
prima facie evidence of intent to permanently reside First. C.A. No. 473 and R.A. No. 9139 are separate and distinct laws
therein; – the former covers all aliens regardless of class while the latter
covers native-born aliens who lived here in the Philippines all their
[3] if allowed himself or his wife or child with acquired citizenship to
lives, who never saw any other country and all along thought that
be used as a dummy;
they were Filipinos; who have demonstrated love and loyalty to the
[4] if he, his wife or child with acquired citizenship commits any act Philippines and affinity to the customs and traditions.52 To reiterate,
inimical to national security the intention of the legislature in enacting R.A. No. 9139 was to
make the process of acquiring Philippine citizenship less tedious, less
So v. Republic, G.R. No. 170603, January 29, 2007 technical and more encouraging which is administrative rather than
judicial in nature.
Facts:
A naturalization proceeding is not a judicial adversary proceeding,
He was born on February 17, 1982, in Manila; he is a Chinese citizen and the decision rendered therein does not constitute res judicata. A
who has lived in No. 528 Lavezares St., Binondo, Manila, since birth; certificate of naturalization may be cancelled if it is subsequently

Constitutional Law 1 based on the syllabus of Atty. Remoroza 2018-2019 91


Merzy’s Notes Prelim Exam

discovered that the applicant obtained it by misleading the court citizenship upon taking the aforesaid oath [Sec. 3, R.A. 9225].
upon any material fact. Law and jurisprudence even authorize the iv) The unmarried child, whether legitimate, illegitimate or adopted,
cancellation of a certificate of naturalization upon grounds or below 18 years of age, of those who reacquire Philippine citizenship
conditions arising subsequent to the granting of the certificate.59 If upon the effectivity of this Act shall be deemed citizens of the
the government can challenge a final grant of citizenship, with more Philippines
reason can it appeal the decision of the RTC within the reglementary v) Those who retain or reacquire Phiilippine citizenship under this
period despite its failure to oppose the petition before the lower Act shall enjoy full civil and political rights and be subject to all
court. attendant liabilities
and responsibilities under existing laws of the Philippines and the
IN LIGHT OF ALL THE FOREGOING, the petition is DENIED for lack following conditions:
of merit.
va) Those intending to exercise their right of suffrage must meet
the requirements under Sec. 1, Art. V of the Constitution, R.A. 9189,
D. Loss and Reacquisition of Philippine Citizenship – C.A. otherwise known as “The Overseas Absentee Voting Act of 2003”
63 and other existing laws;

vb) Those seeking elective public office in the Philippines shall


1. Loss of citizenship meet the qualifications for holding such public office as required by
the Constitution and existing laws and, at the time of the filing of the
a. By naturalization in a foreign country certificate of candidacy, make a personal and sworn renunciation of
any and all foreign citizenship before any public officer authorized to
i) However, this is modified by R.A. 9225, entitled An Act Making administer an oath;
the Citizenship of Philippine Citizens Who Acquire Foreign
Cititzenship Permanent (which took effect September 17, 2003), vb1) In Eusebio Eugenio Lopez v. Comelec, G.R. No.
which declares the policy of the State that all Philippine citizens who 182701, July 23, 2008, reiterated in Jacotv. Dal and Comelec, G.R.
become citizens of another country shall be deemed not to have lost No. 179848,
their Philippine citizenship under the conditions of this Act. November 27, 2008, it was held that a Filipino-American, or any dual
citizen cannot
ii) Natural-born citizens of the Philippines who have lost their run for elective public office in the Philippines unless he personally
Philippine citizenship by reason of their naturalization as citizens of a swears to a renunciation of all foreign citizenship at the time of filing
foreign country are deemed to have reacquired Philippine citizenship of the certificate of candidacy. The mere filing of a certificate of
upon taking the following oath of allegiance to the Republilc: 7 candidacy is not sufficient; Sec. 5 (2) of R.A. 9225 categorically
requires the individual to state in clear and unequivocal
iii) Natural-born citizens of the Philippines who, after the effectivity terms that he is renouncing all foreign citizenship, failing which, he is
of this Act, become citizens of a foreign country shall retain their disqualified from running for an elective position. The fact that he
Philippine may have won the elections, took his oath and began discharging

Constitutional Law 1 based on the syllabus of Atty. Remoroza 2018-2019 92


Merzy’s Notes Prelim Exam

the functions of the office cannot cure the defect of his candidacy.
The doctrine laid down in Valles v. Comelec, supra., and Issue: Whether or not Frivaldo was a citizen of the Philippines at
Mercado v. Manzano, supra., does not apply. the time of his election.

vc) Those appointed to any public office shall subscribe and Held: No. Section 117 of the Omnibus Election Code provides that a
swear to an oath of allegiance to the Republic of the Philippines and qualified voter must be, among other qualifications, a citizen of the
its duly constituted authorities prior to their assumption of office; Philippines, this being an indispensable requirement for suffrage
Provided, That they renounce their oath of allegiance to the country under Article V, Section 1, of the Constitution.
where they took that oath;
Even if he did lose his naturalized American citizenship, such
forfeiture did not and could not have the effect of automatically
restoring his citizenship in the Philippines that he had earlier
vd) Those intending to practice their profession in the renounced.
Philippines shall aplly with the proper authority for a license or
permit to engage in such practice; Qualifications for public office are continuing requirements and must
be possessed not only at the time of appointment or election or
ve) The right to vote or be elected or appointed to any public assumption of office but during the officer’s entire tenure.
office in the Philippines cannot be exercised by, or extended to,
those who: (1) are candidates for or are occupying any public office Frivaldo declared not a citizen of the Philippines and therefore
in the country of which they are naturalized citizens; and/or (2) are disqualified from serving as a Governor of the Province of Sorsogon.
in active service as commissionedor non-commissioned officers in
the armed forces of the country which they are naturalized citizens modified by R.A. 9225

Eusebio Eugenio Lopez v. Comelec, G.R. No.


Frivaldo v. Comelec, 174 SCRA 245 182701, July 23, 2008

Facts: Petitioner Juan G. Frivaldo was proclaimed governor-elect FACTS:


and assume office in due time. The League of Municipalities filed
with the COMELEC a petition for annulment of Frivaldo’s election and Petitioner Lopez, a dual citizen, was a candidate for the position of
proclamation on the ground that he was not a Filipino citizen, having
Chairman of Barangay Bagacay, San Dionisio, Iloilo City held on
been naturalized in the United States. Frivaldo admitted the
October 29, 2007. He was eventually declared the
allegation but pleaded the special and affirmative defenses that his
naturalization was merely forced upon himself as a means of survival winner.respondent Villanueva filed a petition before the Provincial
against the unrelenting prosecution by the Martial Law Dictator’s Election Supervisor of the Province of Iloilo, praying for the
agent abroad.

Constitutional Law 1 based on the syllabus of Atty. Remoroza 2018-2019 93


Merzy’s Notes Prelim Exam

disqualification of Lopez because he was ineligible from running for Section 5. Civil and Political Rights and Liabilities. – Those who retain
any public office. or re-acquire Philippine citizenship under this Act shall enjoy full civil
and political rights and be subject to all attendant liabilities and
Lopez argued that he is a Filipino-American, by virtue of the responsibilities under existing laws of the Philippines and the
Citizenship Retention and Re-acquisition Act of 2003. He said, he following conditions:
possessed all the qualifications to run for Barangay Chairman.
(2) Those seeking elective public office in the Philippines shall meet
On February 6, 2008, COMELEC issued the Resolution granting the the qualification for holding such public office as required by the
petition for disqualification of Lopez from running as Barangay Constitution and existing laws and, at the time of the filing of the
Chairman. COMELEC said, to be able to qualify as a candidate in the certificate of candidacy, make a personal and sworn renunciation of
elections, Lopez should have made a personal and sworn any and all foreign citizenship before any public officer authorized to
renunciation of any and all foreign citizenship. administer an oath.
ISSUE: Whether or not there was grave abuse of discretion on the Lopez was able to regain his Filipino Citizenship by virtue of the Dual
part of the COMELEC for disqualifying petitioner. Citizenship Law when he took his oath of allegiance before the Vice
Consul of the Philippine Consulate General’s Office in Los Angeles,
RULING:
California; the same is not enough to allow him to run for a public
No. The Supreme Court dismissed the petition. The COMELEC office.
committed no grave abuse of discretion in disqualifying petitioner as
candidate for Chairman in the Barangay elections of 2007.

b. By express renunciation of citizenship

Lopez was born a Filipino but he deliberately sought American In Board of Immigration Commissioners v. Go Callano, 25 SCRA 890,
citizenship and renounced his Filipino citizenship. He later on became it was held that express renunciation means a renunciation that is
made known distinctly and explicitly, and not left to
a dual citizen by re-acquiring Filipino citizenship.
inference or implication. Thus, in Labo v. Comelec, 176 SCRA 1, it
was held that
R.A. No. 9225 expressly provides for the conditions before those
Labo lost Filipino citizenship because he expressly renounced
who re-acquired Filipino citizenship may run for a public office in the allegiance to the Philippines when he applied for Australian
Philippines. citizenship.

Section 5 of the said law states: i) In Valles v. Comelec, supra., it was held that the fact that private

Constitutional Law 1 based on the syllabus of Atty. Remoroza 2018-2019 94


Merzy’s Notes Prelim Exam

respondent was born in Australia does not mean that she is not a he was married to an Australian citizen. As a condition for such
Filipino. If Australia follows the principle of jus soli, then at most she naturalization, he formally took the Oath of Allegiance and/or made
can also claim Australian citizenship, resulting in her having dual the Affirmation of Allegiance, renouncing all other allegiance. It does
citizenship. That she was a holder of an Australian passport and had
not appear in the record, nor does the petitioner claim, that he has
an alien certificate of registration do not constitute effective
renunciation, and do not militate against her claim, of Filipino reacquired Philippine citizenship.
citizenship.
For renunciation to effectively result in the loss of citizenship, it must
be express.
c. By subscribing to an oath of allegiance to support
ii) But see Willie Yu v. Defensor-Santiago, 169 SCRA 364, where the Constitution or laws of a foreign country
obtention of a Portuguese passport and signing of commercial
documents as a Portuguese were construed as renunciation of to support the Constitution or laws of a foreign country upon
Philippine citizenship attaining 21 years of age; Provided, however, that a Filipino may not
divest himself of Philippine citizenship in any manner while the
Labo v. Comelec, 176 SCRA 1 Republic of the Philippines is at war with any country.

i) This should likewise be considered modified by R.A. 9225.


Facts:
ii) The proviso that a Filipino may not divest himself of Philippine
Petitioner Ramon Labo, elected mayor of Baguio City was citizenship in this manner while the Republic of the Philippines is at
questioned on his citizenship. He was married in the Philippines to an war with any country may be considered as an application of the
principle of indelible allegiance
Australian citizen. The marriage was declared void in the Australian
Federal Court in Sydney on the ground that the marriage had been
bigamous. According to Australian records, Labo is still an Australian d. By rendering service to or accepting commission in
citizen. the armed forces of a foreign country

Issue: Whether or not Petitioner Labo is a citizen of the Philippines. Provided, that the rendering of service to, or acceptance of
such commission in, the armed forces of a foreign country and the
Held: The petitioner’s contention that his marriage to an Australian taking of an oath of allegiance incident thereto, with consent of the
national in 1976 did not automatically divest him of Philippine Republic of the Philippines, shall not divest a Filipino of his Philippine
citizenship is irrelevant. There is no claim or finding that he citizenship if either of the following circumstances is present:
automatically ceased to be a Filipino because of that marriage. He
(i) The Republic of the Philippines has a defensive
became a citizen of Australia because he was naturalized as such
and/or offensive pact of alliance with the said foreign country;
through a formal and positive process, simplified in his case because
Constitutional Law 1 based on the syllabus of Atty. Remoroza 2018-2019 95
Merzy’s Notes Prelim Exam

or (ii) The said foreign country maintains armed forces in Philippine mode for the reacquisition of Filipino citizenship by Sorsogon
territory with the consent of the Republic of the Philippines. Governor Juan Frivaldo.
ib) The Special Committee on Naturalization created by PD 725,
e. By cancellation of the certificate of naturalization chaired by the Solicitor General with the Undersecretary of Foreign
f. By having been declared by competent authority a Affairs and the
deserted of the Philippine armed forces in time of Director of the NICA as members, was reactivated on June 8, 1995,
war and it is before this Committee that a petition for repatriation is filed
[Angat v. Republic, G.R. No. 132244, September 14, 1999].
ii) When repatriation takes effect. In Frivaldo v. Comelec, 257
2. Reacquisition of citizenship SCRA 727, it was held that repatriation of Frivaldo retroacted to the
date of
a) Under R.A. 9225, bv taking the oath of allegiance required of filing of his application on August 17, 1994. In Altarejos v. Comelec,
former G.R. No. 163256, November 10, 2004, the same principle was
natural-born Philippine citizens who may have lost their Philippine applied. Petitioner took his Oath of Allegiance on December 17,1997,
citizenship by reason of their acquisition of the citizenship of a but his Certificate of Repatriation was registered with the Civil
foreign country. Registry of Makati only after six years, or on February 18,
2004, and with the Bureau of Immigration on March 1, 2004. He
b) By naturalization, provided that the applicant possesses none of completed all the requirements for repatriation only after he filed his
the certificate of candidacy for a mayoralty position, but before the
disqualifications prescribed for naturalization. elections. But because his repatriation
i) In Republic v. Judge de la Rosa, supra., the naturalization retroacted to December 17-, 1997, he was deemed qualified to run
proceeding was so full of procedural flaws that the decision granting for mayor in the May 10, 2004 elections.
Filipino citizenship to Governor Juan Frivaldo was deemed a nullity.
iii) Effectofrepatriation. In Bengzon lllv. House of Representatives
c) By repatriation of deserters of the Army, Navy or Air Corps, Electoral Tribunal, G.R. No. 142840, May 7, 2001, the Supreme
provided Court ruled that the act of repatriation allows the person to recover,
that a woman who lost her citizenship by reason of her marriage to or return to, his original status before he lost his Philippine
an alien may be repatriated in accordance with the provisions of this citizenship. Thus, respondent Cruz, a former naturalborn Filipino
Act after the termination of the marital status. citizen who lost his Philippine citizenship when he enlisted in the
United States Marine Corps, was deemed to have recovered his
i) See P.D. 725, which allows repatriation of former natural-born natural- born status when he reacquired Filipino citizenship through
Filipino citizens who lost Filipino citizenship. repatriation.
ia) In Frivaldo v. Comelec and Lee v. Comelec, 257 SCRA 727,
the Supreme Court held that P.D. 725 was not repealed by President iv) Repatriation under R. A. 8171 (lapsed into law on October 23,
Aquino’s Memorandum of March 27, 1986, and, thus, was a valid 1995). The law governs the repatriation of Filipino women who may
have lost Filipino citizenship by reason of marriage to aliens, as well

Constitutional Law 1 based on the syllabus of Atty. Remoroza 2018-2019 96


Merzy’s Notes Prelim Exam

as the repatriation of former natural-born Filipino citizens who lost only to natural-born Filipinos who lost their citizenship on account of
Filipino citizenship on account of political or economic necessity, political or economic necessity and to their minor children. This
including their minor children, provided the applicant is not a person means that if a parent who had renounced his Philippine citizenship
due to political or economic reasons later decides to repatriate under
[a] opposed to organized government or affiliated with RA8171, his
any association or group of persons who uphold and teach doctrines repatriatioin will also benefit his minor children. Thus, to claim the
opposing organized government; benefit of RA 8171, the children must be of minor age at the time
the petition for repatriation is filed by the parent. This is so because
[b] defending or teaching the necessity or propriety of violence, a child does not have the legal capacity to undertake a political act
personal assault or assassination for the predominance of his ideas; like the election of citizenship. On their own, the minor children
cannot apply for repatriation or naturalization separately from the
[c] convicted of a crime involving moral turpitude; or [d] suffering parents. Tabasa is not qualified to avail himself of repatriation under
from mental alienation or incurable contagious disease. Repatriation RA8171.
is effected by taking the
necessary oath of allegiance to the Republic of the Philippines and Cases:
registration in
the proper Civil Registry and in the Bureau of Immigration and
Lee v. Comelec, 257 SCRA 727
Deportation.

iva) In Tabasa v. Court of Appeals, G.R. No. 125793, August 29,


2006, Joevanie Tabasa, a natural-born citizen of the Philippines, Angat v. Republic, G.R. No. 132244,
acquired American citizenship through derivative naturalization September 14, 1999
when, still a minor, his father became a naturalized citizen of the
United States. On October 3,1995, he was admitted to the FACTS:
Philippines as a “balikbayan”, but within a year, he was charged by
the Bureau of Immigration and Deportation (BID), because it Gerardo Angat, a natural born Filipino citizen, asked to regain his
appeared that the US Department of Justice had revoked his status as a Philippine citizen before the RTC Marikina. RTC allowed
passport and was the subject of an outstanding federal warrant of him to take his Oath of Allegiance on October 3, 1996 and the
arrest for possession of firearms and one count of sexual battery.
following day, the RTC declared him as citizen of the Philippines
Finding him an undocumented and undesirable alien, the BID pursuant to R.A. No. 8171.OSG filed a Manifestation and Motion in
ordered his deportation. After learning of the BID order, he then March 1997, asserting that the petition should have been dismissed
immediately executed an Affidavit of Repatriation and took an oath by the court for lack of jurisdiction.
of allegiance to the Republic of the Philippines. On the issue of
whether he validly reacquired Philippine citizenship, the Supreme ISSUE:Whether or not the RTC has jurisdiction in deciding over
Court ruled in the negative. The privilege of RA 8171 is available repatriation case.

Constitutional Law 1 based on the syllabus of Atty. Remoroza 2018-2019 97


Merzy’s Notes Prelim Exam

RULING: However, on the day that he got his citizenship, the Court had
already ruled based on his previous attempts to run as governor and
No. A petition for repatriation should be filed with the Special acquire citizenship, and had proclaimed Lee, who got the second
Committee on Naturalization and not with the RTC which has no highest number of votes, as the newly elect Governor of Sorsogon.
jurisdiction.Therefore, the court's order was null and void.
ISSUE:Whether or not Frivaldo’s repatriation was valid.
RA No. 8171, which has lapsed into law on October 23 1995, is an
act providing for repatriation of Filipino women who have lost their HELD:
Philippine citizenship by marriage to aliens and of natural-born
Filipinos who have lost the Philippine citizenship on account of The Court ruled his repatriation was valid and legal and because of
political or economic necessity. the curative nature of Presidential Decree No. 725, his repatriation
retroacted to the date of the filing of his application to run for
Moreover, petitioner was incorrect when he initially invoked RA 965 governor. The steps to reacquire Philippine Citizenship by
and RA 2630, since these laws could only apply to persons who had repatriation under Presidential Decree No. 725 are:
lost their Philippine citizenship by rendering service to, or accepting
commission in, the armed forces of an allied country or the armed (1) filing the application;
forces of the US, a factual matter not alleged in his petition.
(2) action by the committee; and
Parenthetically, under these statutes, the person desiring to
reacquire his Philippine citizenship would not even required to file a (3) taking of the oath of allegiance if the application is approved.
petition in court; all he had to do is to take an Oath of Allegiance to
the Republic of the Philippines and to register the said oath with the It is only upon taking the oath of allegiance that the applicant is
proper civil registry. deemed ipso jure to have reacquired Philippine citizenship. If the
decree had intended the oath taking to retroact to the date of the
filing of the application, then it should not have explicitly provided
Frivaldo v. Comelec, 257 SCRA 727 otherwise. He is therefore qualified to be proclaimed governor of
Sorsogon.
FACTS:
G.R. No. 132244, 14 September 1999
Juan G. Frivaldo ran for Governor of Sorsogon again and won. Raul
[Naturalization; Reacquisition; RA No. 8171
R. Lee questioned his citizenship. He then petitioned for repatriation
under Presidential Decree No. 725 and was able to take his oath of
allegiance as a Philippine citizen.
Bengzon lll v. House of Representatives Electoral

Constitutional Law 1 based on the syllabus of Atty. Remoroza 2018-2019 98


Merzy’s Notes Prelim Exam

FACTS: other hand, if he was originally a natural-born citizen before he lost


his Philippine citizenship, he will be restored to his former status as a
Respondent Cruz was a natural-born citizen of the Philippines. He natural-born Filipino.
was born in San Clemente, Tarlac, on April 27, 1960, of Filipino
parents. The fundamental law then applicable was the 1935 Having thus taken the required oath of allegiance to the Republic
Constitution. and having registered the same in the Civil Registry of Magantarem,
Pangasinan in accordance with the aforecited provision, respondent
However, respondent Cruz enlisted in the United States Marine Corps Cruz is deemed to have recovered his original status as a natural-
and without the consent of the Republic of the Philippines, took an born citizen, a status which he acquired at birth as the son of a
oath of allegiance to the United States. As a consequence, he lost his Filipino father. It bears stressing that the act of repatriation allows
Filipino citizenship for under Commonwealth Act No. 63, section him to recover, or return to, his original status before he lost his
1(4), a Filipino citizen may lose his citizenship by, among other, Philippine citizenship.
"rendering service to or accepting commission in the armed forces of
a foreign country." Therefore, Cruz has all the qualifications to be elected as a member
of the House of Representatives. The HRET did not commit any
Respondent Cruz then reacquired his Philippine citizenship through grave abuse of discretion, thus the petition was dismissed.
repatriation under Republic Act No. 2630 entitled as “An Act
Providing For Reacquisition of Philippine Citizenship By Persons Who
Lost Such Citizenship by Rendering Service To, or Accepting
Commission in, the Armed Forces of the United States.” He ran for Tribunal, G.R. No. 142840, May 7, 2001
and was elected as the Representative of the Second District of
Pangasinan in the May 11, 1998 elections. He won by a convincing
margin of 26,671 votes over petitioner Antonio Bengson III, who Tabasa v. Court of Appeals, G.R. No. 125793,
was then running for reelection. August 29, 2006

ISSUE: Whether or not respondent Cruz can still be considered a


natural-born Filipino upon his reacquisition of Philippine citizenship. FACTS:

HELD: When he was 7 years old, Joevanie A. Tabasa acquired American


citizenship when his father became a naturalized citizen of the US. In
Repatriation results in the recovery of the original nationality. This 1995, he arrived in the Philippines and was admitted as
means that a naturalized Filipino who lost his citizenship will be "balikbayan"; thereafter, he was arrested and detained by the agent
restored to his prior status as a naturalized Filipino citizen. On the of BIR. Th Consul General of the US embassy of Manila filed a

Constitutional Law 1 based on the syllabus of Atty. Remoroza 2018-2019 99


Merzy’s Notes Prelim Exam

request with the BID that his passport has been revoked and that
Tabasa had a standing warrant for several federal charges against
him.

Petitioner alleged that he acquired Filipino citizenship by repatriation


in accordance with the RA No. 8171, and that because he is now a
Filipino citizen, he cannot be deported or detained by the BID.

ISSUE: Whether or not he has validly reacquired Philippine


citizenship under RA 8171 and therefore, is not an undocumented
alien subject to deportation.

RULING:

No. Petitioner is not qualified to avail himself of repatriation under


RA 8171. The only person entitled to repatriation under RA 8171 is
either a Filipino woman who lost her Philippine citizenship by
marriage to an alien, or a natural-born Filipino, including his minor
children who lost Philippine citizenship on account of political or
economic necessity.

Petitioner was already 35 years old when he filed for repatriation.


The act cannot be applied in his case because he is no longer a
minor at the time of his repatriation in 1996. The privilege under RA
8171 only belongs to children who are of minor age at the time of
filing of the petition for repatriation.

a. By direct act of Congress

Constitutional Law 1 based on the syllabus of Atty. Remoroza 2018-2019 100

You might also like